Anda di halaman 1dari 234

Chapter 15

Solutions to exercises

15.1 Solutions for chapter 1


Solution for Exercise 1.1
Take the minimum of the four distances of the point from each side and draw a circle
of smaller radius around this point. The interiors of such circles are open sets.

Solution for Exercise 1.2


If f (x) = O(x2 ) as x 0 then f (x) < C|x2 | < C|x| and hence f (x) = O(x).

Solution for Exercise 1.3



(a) x 1 + x2 = x + 12 x3 + = O(x).

(b) x/(1 + x) = x(1 x + x2 + ) = O(x).

(c) x3/2 /[1 exp(x)] = x3/2 /[1 (1 x + x2 /2 + )] = x1/2 /[1 + O(x)] = O(x1/2 ).

Solution for Exercise 1.4


(a) x/(x 1) = (1 1/x)1 = 1 + 1/x + = O(1).
p
(b) 4x2 + x 2x = 2x 1 + 1/4x 2x = 2x(1 + 1/8x + O(x2 )) 2x = O(1).

(c) (x + b)a xa = xa (1 + b/x)a xa = xa (1 + ab/x + ) xa = O(xa1 ).

Solution for Exercise 1.5


p p
(a) Since x/ x2 + y 2 1, y/ x2 + y 2 1 it follows that fk = O(f ), k = 1, 2.

(b) Put x = r cos , y = r sin so


= a cos2 + b sin cos + c sin2 < |a| + |b| + |c| = O(1),
f

and = O(f ). If y = kx with 2kc = b b2 4ac then = 0.

397
398 CHAPTER 15. SOLUTIONS TO EXERCISES

Solution for Exercise 1.6


Since f (0) = 1, we must have A = 0 and B = 1. Since f (x) is finite as x , D = 0.
At x = a, f (x) is continuous and hence a + 1 = C/a2 .

Solution for Exercise 1.7


sin ax sin ax tan ax sin ax 1
(a) lim = a lim = a, (b) lim = lim =a
x0 x x0 ax x0 x x0 x cos ax
sin ax sin ax x a 3x + 4 1
(c) lim = lim = , (d) lim = lim (3x + 4) lim = 2.
x0 sin bx x0 x sin bx b x0 4x + 2 x0 x0 4x + 2
 z
For part (e) Take the logarithm then if E is the limit, ln E = lim w ln 1 + = z,
z
w w
so E = e .

Solution for Exercise 1.8


In these examples f (0, 0) is not defined, except possibly as a limit. This limit, if it
exists, can be found using the polar coordinates x = r cos , y = r sin .
(a) f = sin 2, which is independent of r, so the value of the function in the neighbour-
hood of the origin depends upon the direction of approach, that is , so f is not defined
at the origin and is not continuous.
(b) f = 1/ cos 2; the same remark as in part (a) applies and f is not continuous.
(c) f = r cos sin 2, so f 0 as r 0 independent of . This proves that f (r, )
is continuous at r = 0; but since the transformation between (x, y) and (r, ) is not
continuous at r = 0, this does not prove that f (x, y) is continuous at the origin. For
this we observe that
2x2 y

|f (x, y)| = 2
|2y| , that is f (x) = o(x).
x + y2

Solution for Exercise 1.9


(a) Since y = 3(a2 x2 ), y is strictly increasing on (a, a). At x = a, y = 2a3 .
(b) With x = 2a sin , || < sin1 (1/2) = /6,
y = 6a3 sin 2a3 (3 sin sin 3) = 2a3 sin 3.
Hence
  y 
1  y  1
= sin1 and x(y) = 2a sin sin1
, |y| < 2a3 .
3 2a3 3 2a3

(c) For x > a, y(x) is strictly decreasing and for x > 2a, y < 2a3 . Set x = 2a cosh
and the equation becomes
y = 6a3 cosh 2a3 (3 cosh + cosh 3) = 2a3 cosh 3
giving 
1  y 
x(y) = 2a cosh cosh1 3 , y < 2a3 .
3 2a
15.1. SOLUTIONS FOR CHAPTER 1 399

Solution for Exercise 1.10


(a) Use the product and chain rule,

d   ax b+x a b 2x
ax b+x = = p .
dx 2 b+x 2 a x 2 (b + x)(a x)

Alternatively, if y = a x b + x, then
1 1 1 dy 1 1 a b 2x
ln y = ln(a x) + ln(b + x) giving = =
2 2 y dx 2(b + x) 2(a x) 2(b + x)(a x)

which, on simplification, gives the same result.


(b) Define

dy dy (a b) sin 2x
y 2 = a sin2 x+b cos2 x to give 2y = 2(ab) sin x cos x or = p
dx dx 2 a sin2 x + b cos2 x
which can also be expressed in the form

dy (a b) sin 2x
= p .
dx 2(a + b) + 2(b a) cos 2x

(c) Use the chain and product rule

d
cos x3 cos x = 3x2 sin x3 cos x cos x3 sin x.
   
dx

dy du
(d) If y = xx = ex ln x , putting u = x ln x the chain rule gives = eu = (1 + ln x)xx .
dx dx

Solution for Exercise 1.11


dx p
cos x, but cos x = 1 sin2 x = 1 y 2 ,
p
Differentation with respect to y gives 1 =
dy
hence the result.

Solution for Exercise 1.12


(a) Since y = f (g(y)) differentiation with respect to y gives

d   df dg
1= f (g(y)) = = f (g)g (y).
dy dg dy
dy dx
Since = f (x) and = g (y), the result follows.
dx dy
(b) Differentiate again with respect to y
1 1 2 3
d2 x d2 y d2 y
   
d dy d dy dx dy dx dy
= = = 2 = 2 .
dy 2 dy dx dx dx dy dx dx dy dx dx
400 CHAPTER 15. SOLUTIONS TO EXERCISES

Solution for Exercise 1.13


Use the chain rule with u = x, so, if f (x) is even, f (u) = f (x) and differentiate
dx
with respect to u, f (u) = f (x) = f (x), that is f (x) = f (x) and f (x) is an
du
odd function. Examples of even functions and their derivatives, in brackets are cos x
2 2
( sin x), ex (2xex ). A similar analysis applies to odd functions.

Solution for Exercise 1.14


We have  
1 1 f (x + h) f (x)
=
f (x + h) f (x) f (x + h)f (x)
so that
f (x)
   
1 1 1 f (x + h) f (x) 1
lim = lim =
h0 h f (x + h) f (x) h0 h f (x + h)f (x) f (x)2

The product rule is proved by writing


   
f (x + h)g(x + h) f (x)g(x) = f (x + h) f (x) g(x + h) + f (x) g(x + h) g(x)

dividing by h and taking the limit h 0.

Solution for Exercise 1.15


The first two results follow directly by applying the product rule. Thus

h = f g + f g = h = (f g + f g ) + (f g + f g ).
     
(3) 2 2 2
The expression for h follows similarly. Since = = 1 and = 2 the
0 2 1
general result quoted is therefore true for n = 1 and 2. Suppose it to be true for n; a
further differentiation gives
n  
(n+1)
X n 
h = f (nk+1) g (k) + f (nk) g (k+1)
k
k=0
n  n+1
X n 
X n
= f (nk+1) g (k) + f (n+1s) g (s) (with s = k + 1 in second sum)
k s1
k=0 s=1
    n    
n + 1 (n+1) (0) n (0) (n+1)
X n n
= f g + f g + + f (nk+1) g (k) .
0 n k k1
k=1
   
m m
But, for all m, = = 1 and
0 m
   
n n n! n! n! (n + 1)! k
+ = + = +
k k1 k! (n k)! (k 1)! (n + 1 k)! k! (n k)! k! (n + 1 k)! n + 1
 
(n + 1)! n+1k (n + 1)! k n+1
= + = .
k! (n + 1 k)! n + 1 k! (n + 1 k)! n + 1 k
15.1. SOLUTIONS FOR CHAPTER 1 401

Hence the (n + 1) derivative can be written as


    n  
(n+1) n + 1 (n+1) n+1 (n+1)
X n + 1 (n+1k) (k)
h = f g+ fg + f g ,
0 n+1 k
k=1
n+1
X 
n + 1 (n+1k) (k)
= f g .
k
k=0

Thus, if the formula is true for n, it is true for n + 1: it is true for n = 2 and hence is
true for all n.

Solution for Exercise 1.16


d du 1 f (x)
The chain rule with u = f (x) gives ln u = = . Take the logarithm of p(x)
dx dx u f (x)
to obtain
n n
X p X f (x)
k
ln p = ln fk (x) and hence = ,
p fk (x)
k=1 k=1

which is valid provided none of the fk (x) are zero, that is p(x) 6= 0.

Solution for Exercise 1.17


Expanding the determinant gives

f (x) g(x)
D(x) = = f g giving D = (f g ) + (f g )
(x) (x)

which can be put in the form quoted.


The third-order determinant, with each element a function of x,

a b c

D(x) = d e f

g h i

can be written as a sum of three second-order determinants,



e f
b d f + c d e .

D(x) = a
h i g i g h

Now differentiate this expression using the rule just obtained for second-order determi-
nants; then recombine the 9 terms into a third-order determinant, to obtain

a b c a b c a b c
D (x) = d e f + d e f + d e f .

g h i g h i g h i

Solution for Exercise 1.18


We have F (x) = f (g(x)) and so

F (x + h) F (x) f (g(x + h)) f (g(x)) 1h    i


= = f g(x) + hg (x + h) f g(x)
h h h
402 CHAPTER 15. SOLUTIONS TO EXERCISES

where we have used the mean value theorem, equation 1.11 (page 22), to write

g(x + h) = g(x) + hg (x + h), 0 < < 1.

Now use the mean value theorem again to write

f (g + k) = f (g) + kf (g + k), k = hg (x + h), 0 < < 1,

so that
F (x + h) F (x)
= f g(x) + hg g (x + h).

h
This gives the required result on taking the limit h 0.

Solution for Exercise 1.19


1 x p
Z p
(a) dt 4 + 3t3 = 4 + 3(x)3 for 0 < < 1. Hence the limit is 2.
x 0
Z x Z z
1 2 3 1
ds ln(1+s3 ) where z = x1 and s = t1.

(b) dt ln 3t 3t + t = 3
(x 1)3 1 z 0
the Mean Value theorem gives the second integral as z 2 ln(1 + (z)3 ), 0 < < 1 and
this is zero in the limit z 0.

Solution for Exercise 1.20


u u x2
(a) We have = 2x sin(ln y), = cos(ln y).
x y y
(b) Differentiating r2 with respect to x and y gives, respectively

r r
2r = 2x and 2r = 2y,
x y
p
hence the result. Alternatively, put r = x2 + y 2 to obtain

r x x r y y
= p = and = p = .
x x2 + y 2 r y x2 + y 2 r

Solution for Exercise 1.21


Differentiating with respect to x and y gives
 2
x2
 2
2x x 2x x x2
= exp = and = 2 exp = 2 .
x y y y y y y y

A second differentation of the first result with respect to x gives

2 2 2x 2 x2 2
2
= = + 4 2
=4 .
x y y x y y y y
15.1. SOLUTIONS FOR CHAPTER 1 403

Solution for Exercise 1.22


The derivatives ux and uy are found in exercise 1.20(a); differentating uy again with
x2
respect to y gives uyy = 2 (cos(ln y) + sin(ln y)) . These expressions for ux , uy and
y
uyy satisfy the given equation.

Solution for Exercise 1.23


In this example fx = y, fy = x 2yt, ft = y 2 , dx/dt = 2t and dy/dt = 3t2 . Hence
equation 1.22 becomes
df dx dy
= y 2 + y + (x 2ty) = t4 (5 7t2 ).
dt dt dt
Alternatively, express f in terms of t,
df
f (t) = t5 t7 = t4 5 7t2 .

so
dt
Using the first expresion for df /dt we have
   
df dy dx 2 dy dx dy
= 2t 1 4t2 .

= x +y y 2ty = 2y 2t
y dt y dt dt dt dt dt

Alternatively,  
d f d dx dy
= (x 2ty) = 2y 2t .
dt y dt dt dt

Solution
for Exercise 1.24
If F = 1 + x1 x2 then the chain rule gives

dF F F x1 x + x1 x2
= x1 + x2 = 2 .
dt x1 x2 2 1 + x1 x2
dF 1 du
Alternatively, set u = x1 x2 , so = , which is a simpler method of deriving
dt 2 1 + u dt
the same result.
Differentiate this expression with respect to x1 , using the product rule,
   
dF 1 1
= (x1 x2 + x1 x2 ) + (x1 x2 + x1 x2 )
x1 dt x1 2 1 + x1 x2 2 1 + x1 x2 x1
1 x2 x2
= (x1 x2 + x1 x2 ) + .
4 (1 + x1 x2 )3/2 2 1 + x1 x2
F x2
Also = , and the chain rule gives
x1 2 1 + x1 x2

x
 
d F x2 d
= 2 3/2
(x1 x2 ),
dt x1 2 1 + x1 x2 4(1 + x1 x2 ) dt

as before.
404 CHAPTER 15. SOLUTIONS TO EXERCISES

Solution for Exercise 1.25


(a) We have using equation 1.20

d f du f dv
f (x, y) = + where u = x and v = y.
d u d v d
Now substitute f (x, y) = p f (x, y) into the left-hand side of give

d
f (x, y) = pp1 f (x, y)
d
and set = 1 to obtain the result.

(b) Differentiate both sides of the relation p f (x1 , x2 , , xn ) = f (x1 , x2 , , xn )


with respect to to obtain
n n
p1
X xk X
p f (x) = fxk (x) = xk fxk (x),

k=1 k=1

and set = 1.

(c) Differentiate both sides of the relation with respect to xk to obtain


p fxk (x) = f (x) = f (x) = fxk (x)
xk (xk )

which proves the result.

Solution for Exercise 1.26


dy
We have fx = g (x) and fy = 1 and equations 1.23 and 1.24 give = g (x) and
dx
dx
= 1/g (x), hence the result.
dy

Solution for Exercise 1.27


Here f (x, y) = ln(x2 + y 2 ) 2 tan1 (y/x) giving

2x 2 y 2(x + y)
fx = + = 2 ,
x2 + y 2 (1 + y 2 /x2 ) x2 x + y2
2y 2 1 2(y x)
fy = = 2 .
x2 + y 2 x (1 + y 2 /x2 ) x + y2

dy fx x+y
Hence = = .
dx fy xy

Solution for Exercise 1.28


Assuming y(0) is finite, putting x = 0 in the equation gives y(0) = 0. If f = x y +
dy fx 1 + y cos(xy)
sin(xy) then = = and hence y (0) = 1. Rewrite the expression
dx fy 1 x cos(xy)
15.1. SOLUTIONS FOR CHAPTER 1 405

for y (x) in the form (x cos u 1) y (x) + 1 + y cos u = 0, u = xy and differentiate to


obtain

(x cos u 1) y (x) + (cos u xu sin u) y (x) + y (x) cos u yu sin u = 0.

But u = xy + y, which is zero at x = 0. Hence at x = 0 this equation becomes


y (0) + 2y (0) = 0 and hence y (0) = 2.

Solution for Exercise 1.29


If y = xv(x) the equation for v is
dv a2 + v 2 v+1 dx
Z Z
x = or dv = .
dx v+1 v 2 + a2 x
Integration and substituting for v = y/x then gives
1  1  y 
ln a2 x2 + y 2 + tan1 =B
2 a ax
where B is a constant. Since y(1) = A we obtain the given expression for B.

Solution for Exercise 1.30


The Jacobian determinant for the functions f1 (r, ) = r cos and f2 (r, ) = r sin is
f1 f1


J = r = cos r sin = r


f2 f2 sin r cos


r
Hence, provided
p r 6= 0, J > 0 and the equations may be inverted. Squaring and adding
gives r = x2 + y 2 ; division gives = tan1 (y/x).

Solution for Exercise 1.31


We have f (x) = ieix and f (x) = i2 eix . Assuming f (n) (x) = in eix differentiating and
using induction, we see that the result holds for all n. Equation 1.30 for the Taylor
series, with a = 0 then gives

X (ix)k
f (x) = .
k!
k=0

In this example an = in /n!, so |an /an+1 | = n + 1 as n so the radius of


convergence is infinite.
Since i2k = (1)k and i2k+1 = i(1)k we can write the series as the form

X (ix)2p X (ix)2q+1 X (x2 )p X (1)q (x)2q+1
eix = + = +i .
p=0
(2p)! q=0
(2q + 1)! p=0 (2p)! q=0
(2q + 1)!

But eix = cos x + i sin x, so equating real and imaginary parts gives the quoted series.

Solution for Exercise 1.32


If f (x) = (1 + x)a then f (x) = a(1 + x)a1 , f (x) = a(a 1)(1 + x)a2 and

f (k) (x) = a(a 1)(a 2) (a k + 1)(1 + x)ak for all k provided a is not an integer.
406 CHAPTER 15. SOLUTIONS TO EXERCISES

Thus the Taylor series about the origin becomes



X a(a 1)(a 2) (a k + 1)
(1 + x)a = 1 + xk .
k!
k=1

If a is an integer, a = n, this series terminates at k = n, to give the usual binomial


expansion of (1+x)n . In this example, when a is not an integer, we see that |ak /ak+1 | =
|(k + 1)/(a k)| 1 as k , so the radius of convergence is unity.

Solution for Exercise 1.33


Since f = sin x/ cos x, is an odd function only odd powers occur in the Taylor expansion:
we have

sin2 x 1 2 sin x 2 6 sin2 x


f (x) = 1 + = , f (x) = and f (3) (x) = + ,
cos2 x cos2 x cos3 x 2
cos x cos4 x

and f (0) = f (0) = 0 (as expected) and f (0) = 1 and f (3) (0) = 2 giving the required
Taylor series.

Solution for Exercise 1.34

(a) For the first part use the solution of exercise 1.32, with a = 1 so a(a 1) (a
k + 1) = (1)k k!, giving the quoted series. Then
x Z x
1
Z
dt 1 t + t2 + + (1)n1 tn1 +

ln(1 + x) = dt=
0 1+t 0
x2 x3 (1)n xn
= x + + + .
2 3 n

(b) The series for (1 + t)1 is valid for |t| < 1, so for |x| < 1 the integral and sum may
be interchanged.

(c) Put x x and subtract this from the original series.

Solution for Exercise 1.35


The series is obtained from the solution of the previous exercise by replacing t with t2 .
Then
Z x Z x X
1 1 k 2k
X (1)k x2k+1
tan x = dt = dt (1) t = .
0 1 + t2 0 2k + 1
k=0 k=0

Solution for Exercise 1.36


Use the two Taylor expansions

z2 z3 z4 x2
 
ln(1 + z) = z + + O(z 5 ) and sinh x = x 1 + + O(x5 )
2 3 4 6
15.1. SOLUTIONS FOR CHAPTER 1 407

to give
2
x2 x2 x2 x3 x4
  
ln(1 + sinh x) = x 1+ + 1+ + + + O(x5 )
6 2 6 3 4
x3
  2
x4 x3 x4
 
x
= x+ + + + O(x5 )
6 2 6 3 4
x2 x3 5x4
= x + + O(x5 ).
2 2 12

Solution for Exercise 1.37


If y(0) = 0, putting x = 0 gives y (0) = 1. Differentiate n times using Leibnizs rule:
n
X n!
(1 + x)y (n+1) + ny (n) = xy (n) + ny (n1) + y (k) y (nk) .
k! (n k)!
k=0

With n = 1, 2, 3, and 4 this gives


n = 1 (1 + x)y (2) + y (1) = xy + y + 2yy
n = 2 (1 + x)y (3) + 2y (2) = xy + 2y + 2y 2 + 2yy
n = 3 (1 + x)y (4) + 3y (3) = xy (3) + 3y (2) + 2y (3) y + 6y (2) y (1)
 2
n=4 (1 + x)y (5) + 4y (4) = xy (4) + 4y (3) + 2y (4) y + 8y (3) y (1) + 6 y (2)
Since y(0) = 0 and y (1) (0) = 1 (from the original equation) these equations give
y (2) (0) = 1, y (3) (0) = 6, y (4) (0) = 27 and y (5) (0) = 186 and hence
1 9 31
y = x x2 + x3 x4 + x5 + O(x6 ).
2 8 20
P5
An alternative method is to assume the expansion y = x + k=2 ak xk , which au-
tomatically satisfies the conditions y(0) = 0 and y (0) = 1, to substitute this into
the differential equation, collect the powers of xk , k = 2, 3, , 5, and equate their
coefficients to zero to obtain equations for the constants ak .

Solution for Exercise 1.38


(a) The required derivatives are
fx = cos x sin y giving fx (0, 0) = 0, fy = sin x cos y giving fy (0, 0) = 0,
fxx = sin x sin y giving fxx (0, 0) = 0, fxy = cos x cos y giving fxy (0, 0) = 1,
fyy = sin x sin y giving fyy (0, 0) = 0,
and hence, to this order, sin x sin y = xy, as might be expected from the Taylor series
for each component of the product.
(b) Put u(x, y) = x + ey 1, so u(0, 0) = 0 and use the chain and product rule
to compute the derivatives, fx = ux cos u, fy = uy cos u, fxx = uxx cos u u2x sin u,
fyy = uyy cos u u2y sin u, fxy = uxy cos u uy ux sin u. Since ux = 1, uxx = uxy = 0,
uy = ey and uyy = ey we obtain,
fx (0, 0) = 1, fy (0, 0) = 1, fxx (0, 0) = 0, fxy (0, 0) = 0, fyy (0, 0) = 1,
1
and hence f = (x y) + y 2 + .
2
408 CHAPTER 15. SOLUTIONS TO EXERCISES

Solution for Exercise 1.39


Consider each term of the Taylor series in turn. The first-order term is
 

T1 = h +k +l f = hfx + kfy + lfz ,
x y z

where all derivatives are evaluated at (a, b, c). For the second-order and third-order
terms we use the identities

( + + )2 = 2 + 2 + 2 + 2 + 2 + 2,
( + + )3 = 3 + 3 + 3 + 3 2 + 3 2 + 32 + 3 2 + 32 + 3 2 + 6,

which can be derived by direct multiplication. Hence, on replacing by h/x, by


k/y, and by l/z we obtain, for the second-order term

2! T2 = h2 fxx + k 2 fyy + l2 fzz + 2hkfxy + 2klfyz + 2hlfxz .

Similarly for the third-order term

3! T3 = h3 fxxx + k 3 fyyy + l3 fzzz


+3hk 2 fxyy + 3hl2 fxzz + 3kh2 fyxx + 3kl2 fyzz + 3lh2 fzxx + 3lk 2 fzyy
+6hklfxyz .

Solution for Exercise 1.40


cosh x cosh a sinh x
(a) lim = lim = tanh a.
xa sinh x sinh a xa cosh x

(b)

sin x x cos x 1 sin x cos x 1


lim = lim = lim = lim = ,
x0 x cos x x x0 cos x x sin x 1 x0 x cos x + 2 sin x x0 3 cos x x sin x 3

3x 3x ex ln 3 ex ln 3 ln 3 ex ln 3 + ex ln 3 ln 3
(c) lim = lim = lim = .
x0 2x 2x x0 ex ln 2 ex ln 2 x0 ln 2 ex ln 2 + ex ln 2 ln 2

Solution for Exercise 1.41


f (x) g (x) g(x) f (x)
(a) If lim = then lim = 0 and hence lim = 0 so lim = .
xa g (x) xa f (x) xa f (x) xa g(x)

(b) Put F (x) = 1/f (x) and G(x) = 1/g(x) so F (a) = G(a) = 0, and
2
g (x) f (x)2 g (x)
 
f (x) G(x) f (x)
lim = lim = lim = lim lim .
xa g(x) xa F (x) xa f (x) g(x)2 xa f (x) xa g(x)

f (x) f (x)
Hence, provided all limits exist, lim = lim .
xa g(x) xa g (x)
15.1. SOLUTIONS FOR CHAPTER 1 409

Solution for Exercise 1.42


Z a Z 0 Z a
(a) dx f (x) = dx f (x) + dx f (x), put x = u in the first integral and use
a a 0
the fact that f (u) = f (u) to show that the two integrals have the same magnitude
but opposite signs.

(b) Split the integral in the same manner as in part (a), but since f (u) = f (u) the
two integrals are equal.

Solution for Exercise 1.43


sin y
Z
Assuming > 0, put y = x in the integral, which becomes I() = dy .
Z 0 y
sin x
If > 0, put = to obtain I() = dx = I().
0 x

Solution for Exercise 1.44


Z Z
(a) dx 1 ln x = x ln x dx = x(1 ln x).

x sin x
Z Z Z Z
(b) dx 2x
= x tan x dx tan x but dx tan x = dx = ln | cos x| .
cos cos x
x
Z
Hence dx = x tan x + ln | cos x|.
cos2 x
1 1 1 1 1
Z Z
(c) dx x ln x = x2 ln x dx x2 = x2 ln x x2 .
2 2 x 2 4
Z Z
(d) dx x sin x = x cos x + dx cos x = sin x x cos x.

Solution for Exercise 1.45


(a) Put cos x = u to obtain
1 i1 1 1
Z h
du ln u = u(1 ln u) = ln 2 + 1.
1/ 2 1/ 2 2 2 2

(b)

/4 /4   /4
x 1 2
Z Z 
dx x tan2 x = dx x = x tan x + ln cos x x
0 0 cos2 x 2 0
1 2
= ln 2 .
4 2 32

(c)
1 1
1 1 x3

1 3 1
Z Z
dx x2 sin1 x = x sin x dx .
0 3 0 3 0 1 x2
410 CHAPTER 15. SOLUTIONS TO EXERCISES

But on putting x = sin and using the identity sin 3 = 3 sin 4 sin3 ,
1 /2 /2
x3 1 2
Z Z Z
3
dx = d sin = d (3 sin sin 3) =
0 1 x2 0 4 0 3
1
2
Z
and hence dx x2 sin1 x = .
0 6 9

Solution for Exercise 1.46


Integrating by parts for n 1 gives
Z x  n x
t at n
In = dt tn eat = e In1
0 a 0 a

and hence aIn = xn eax nIn1 , n 1, with I0 = (eax 1)/a.


The equations for Ik , k = 1, 2, , n, are

aI1 = xeax I0 , aI2 = x2 eax 2I1 , aI3 = x3 eax 3I2 , , aIn = xn eax nIn1 .

Multiply the kth equation by Ak and add all the equations to obtain
n
X n
X n
X
a Ak Ik = eax Ak xk kAk Ik1 .
k=1 k=1 k=1

Now chose the Ak such that An = 1/a and for k = 1, 2, , n 1, the Ik cancel, that is
1
aAk = (k + 1)Ak+1 , k = 1, 2, , n 1, An = .
a
n! (1)nk
The solution of these equations is Ak = which gives the quoted expression.
ank+1 k!
Solution for Exercise 1.47
Z a Z 0 Z a
(a) dx f (x) = du f (a u) = dx f (a x).
0 a 0

(b) Since sin(/2 ) = cos and cos(/2 ) = sin we have


/2 0 /2
sin cos cos
Z Z Z
I= d = d = d .
0 sin + cos /2 cos + sin 0 cos + sin

Hence, on adding these two equivalent forms, 2I = /2.

Solution for Exercise 1.48


With t = tan(x/2) the integral becomes
Z Z Z
1 dx 1 1
dx = dt  =2 dt ,
(a b)t2

0 a + b cos x 0 dt 1t
a + b 1+t2
2
0 a + b +
15.1. SOLUTIONS FOR CHAPTER 1 411

sincerdt/dx = (1 + t2 )/2. The integral is evaluated with the further substitution


a+b
t= tan z, to give the quoted result. If b > a, a + b cos x = 0 for some x (0, ),
ab
the integrand is singular
Z and the integral does not exist.
1
Define F (a, b) = dx = then
0 a + b cos x a 2 b2
Z
F 1 a
= dx 2
= 2 ,
a 0 (a + b cos x) (a b2 )3/2

2F 1 (2a2 + b2 )
Z
= 2 dx = ,
a2 (a + b cos x)3 (a2 b2 )5/2
Z 0
F cos x b
= dx 2
= 2 .
b 0 (a + b cos x) (a b2 )3/2
For the last example define
Z
G 1
Z
G(a, b) = dx ln(a + b cos x) so = dx = .
0 a 0 a + b cos x a2 b 2
Integrating with respect to a gives
1
Z p
G = C + da = C + ln(a + a2 b2 ),
a2 b 2
where C is a constant. But if b = 0, G = ln a and hence C = ln 2 and we obtain
Z !
a + a2 b 2
dx ln(a + b cos x) = ln .
0 2

Solution for Exercise 1.49


First note that the integral expression for y(t) gives y(a) = 0. Differentiate twice with
respect to t, using the formula 1.52,
Z t Z t
dy d2 y
= dx f (x) cos (tx) and 2
= f (t) dx f (x) sin (tx) = f (t) 2 y(t).
dt a dt a

From the first of these equations we see that y (a) = 0, so the initial conditions are
satisfied. The second equation gives y (a) = f (a), which is consistent with the original
differential equation.

Solution for Exercise 1.50


(a) Since
Z a(u+h) Z a(u) Z a(u+h)
F (u + h) = dx f (x) = dx f (x) + dx f (x)
0 0 a(u)

we have
a(u+h)
F (u + h) F (u) 1 a(u + h) a(u)
Z  
= dx f (x) = f (), where a(u), a(u + h) ,
h h a(u) h
412 CHAPTER 15. SOLUTIONS TO EXERCISES

the last result being obtained from the integral form of the Mean Value Theorem.
Taking the limit h 0 gives F (u) = a (u)f (a(u)). The same result can be derived
using the Fundamental theorem of Calculus and the chain rule.

(b) We have
b
F (u + h) F (u) f (x, u + h) f (x, u)
Z
= dx
h a h
Z b
f
Assuming that the limit h 0 exists we obtain F (u) = dx .
a u

Solution for Exercise 1.51


If y = x 1/x, as x increases from to 0 and from 0 to , y increases monotonically
from to . Inverting the equation for y gives therefore gives two values for x(y),
p p
y y2 + 4 y + y2 + 4
x= < 0 and x = > 0,
2 2
and on each branch
dx 1 y dx 1 y
= p (x < 0) and = + p (x > 0).
dy 2 2 y2 + 4 dy 2 2 y2 + 4

On splitting the integral into the sum of two parts we obtain


  0   Z  
1 1 1
Z Z
dx f x = dx f x + dx f x
x x 0 x
Z ! Z !
1 y 1 y
= dy p f (y) + dy + p f (y)
2 2 y2 + 4 2 2 y2 + 4
Z
= dy f (y)

which is the required result.


Since  2
2 1 1
x + 2 = x +2
x x
the given integral becomes
  Z  2 ! Z
1 1
Z
2 2 2 y 2
dx exp x 2 = e dx exp x =e dy e = 2 .
x x e

Solution for Exercise 1.52


In the first case
Z X Z X
1 d
dx = dx ln(ln x) = ln(ln X) ln(ln 2) as X .
2 x ln x 2 dx
15.1. SOLUTIONS FOR CHAPTER 1 413

In the second case, integration by parts gives


X  X Z X
1 1 1
Z
dx = +2 dx
2 x(ln x)2 ln x 2 2 x(ln x)2

and hence
X
1 1 1 1
Z
dx 2
= as X .
2 x(ln x) ln 2 ln X ln 2

Solution for Exercise 1.53


Put y = ln x so the integral becomes
ln X
e(a1)y
Z
dy .
ln 2 yb

If a > 1 the integral converges for all b because the exponential term dominates. If
a < 1 the integral diverges for all b, for the same reason. If a = 1 the integral converges
only if b > 1.

Solution for Exercise 1.54


(a) Put x = 1 + so the ratio becomes
2
ea ln(1+) 1 ea+O( ) 1
= = a + O().

(b) Use the binomial expansion



1 + x/2 + O(x2 ) 1

1+x1 1 + O(x)
= 2
= = 1 + O(x).
1 1x 1 (1 x/2 + O(x )) 1 + O(x)

(c) Put x = a + and use the binomial expansion to give



a1/3 1 + 3a + O( 2 ) a1/3

(a + )1/3 a1/3 2
= 1/2
= 1/6 + O().
(a + )1/2 a1/2

a 1 + 2a + O( 2 ) a1/2 3a

2
(d) Put x = /2 , > 0 to give ( 2x) tan x = = 2 + O().
tan
(e) Put y = x1/x , so ln y = (1/x) ln x and lim ln y = and lim x1/x = 0.
x0 x0

(f) We have
 1/x     
1+x 1 1+x 1
2 x + O(x3 ) = e2 .

lim = lim exp ln = lim exp
x0 1x x0 x 1x x0 x
414 CHAPTER 15. SOLUTIONS TO EXERCISES

Solution for Exercise 1.55


In all cases put z = ex . In the first example this gives 2
p 2y = z + 1/z or z 2yz + 1 = 0.
This quadratic has the two solutions z = ex = y y 2 1, one of which is larger than
unity and the other smaller because they are real and their product is unity. For
x > 0, ex > 1 and so p
x = ln z = ln(y + y 2 1).
In the secondpexample the quadratic equation is z 2 2yz 1 = 0, with solutions
z = ex = y y 2 + 1. Since ex > 0 we choose the positive root to give
p
x = ln(y + y 2 + 1).
z2 1
r
1+y
In the finally example we have y = 2 or z = ex = . The positive root
z +1 1y
gives the required solution so  
1 1+y
x = ln .
2 1y

Solution for Exercise 1.56


Since y (x) = x (y)1 a second differentiation gives
d2 y x (y)
 
d 1 dy
= =
dx2 dy x (y) dx x (y)3
and since y (x) = y this gives
 3
d2 x dx dz dx
2
=y or = yz 3 if z= .
dy dy dy dy
Integration gives 1/z 2 = y 2 + c, but x (0) = 1/y (0) = 1 and y(0) = 0, so c = 1 and
dx 1
= p , x(0) = 0,
dy 1 y2

where the
Z ynegative square root is ignored because x (0) = 1. A further integration gives
1
x(y) = du .
0 1 u2
The Taylor expansion of the integrand is
1 1 3
= 1 + u2 + u4 + O(u6 )
1 u2 2 8
so integration gives
1 3
sin1 y = y + y 3 + y 5 + O(y 7 ).
6 40

1 X (2k)! u2k
More generally, we have = , |u| < 1, so
1 u2 k!2 22k
k=0

X (2k)! y 2k
sin1 y = y , |y| < 1.
k! 2 22k (2k + 1)
k=0
15.1. SOLUTIONS FOR CHAPTER 1 415

Solution for Exercise 1.57


(a) Since ln y = g ln f we have y /y = g ln f + gf /f and hence

dy
= (f g ln f + gf ) f (x)g(x)1 .
dx

(b) Since
1 1 1 1
ln y = ln(p + x) ln(p x) + ln(q + x) ln(g x)
2 2 2 2
we have
y
   
1 1 1 1 1 1 p q
= + + + = + 2
y 2 p+x px 2 q+x qx p2 x2 q x2

and  r r
dy p q p+x q+x
= + 2 .
dx p2 x2 q x2 px qx

(c) We have

dy x yn dy y
ny n1 =1+ = therefore = .
dx 1 + x2 1 + x2 dx n 1 + x2

Solution for Exercise 1.58


Differentiate using the chain rule,
dy a cos u
= , u = a sin1 x,
dx 1 x2
and
d2 y a2 sin x ax cos u a2 y x dy
= + = + ,
dx2 1 x2 (1 x2 )3/2 1 x2 1 x2 dx
which gives the required result.

Solution for Exercise 1.59


If x = cos we have
dy dy d 1 dy dx p
= = since = sin = 1 x2 .
dx d dx 1 x d
2 d

and then
d2 y 1 d2 y d x dy
= .
dx2 1x2 d 2 dx (1 x 2 )3/2 d

Hence the differential equation becomes

d2 y x dy
+ + y = 0,
d2 1 x2 d
p
which gives the required result since x/ x2 + y 2 = cot .
416 CHAPTER 15. SOLUTIONS TO EXERCISES

Solution for Exercise 1.60


Let h(x) = f (g(x)) then

h (x) = g (x)f (g),


h (x) = g (x)f (g) + g (x)2 f (g),
h (x) = g (x)f (g) + 3g (x)g (x)f (g) + g (x)3 f (g),

so that
2
g (x)f (g) + 3g (x)g (x)f (g) + g (x)3 f (g) 3 g (x) g (x)f (g)

Sh(x) = + .
g (x)f (g) 2 g (x) f (g)

On multiplying this out we see that Sh(x) = Sg(x) + g (x)2 Sf (g) < 0 since Sg(x) < 0
and Sf (g) < 0.

Solution for Exercise 1.61


Differentation gives
z 1 x
= f + g cos(x + ay) + 2 sin(x + ay),
x 2a2 2a
2z 1 x
= f + g + 2 sin(x + ay) + 2 cos(x + ay),
x2 a 2a
z x
= a(f g ) + sin(x + ay),
y 2a
2z x
= a2 (f + g ) + cos(x + ay),
y 2 2

2z 2z
and hence a2 2
2 = sin(x + ay).
x y
Solution for Exercise 1.62
f f f
Differentiation gives fx = af , fy = bf and fz = cf . So the partial
x y z
derivatves are zero at ax = by = cz = 1.

Solution for Exercise 1.63


Differentiate with respect to x,

2(uux x)f1 + 2uux f2 + 2uuxf3 = 0 or uux(f1 + f2 + f3 ) = xf1

where fk = f /xk , f = f (x1 , x2 , x3 ). Similarly, differentiation with respect to y and


z gives
uuy (f1 + f2 + f3 ) = yf2 and uuz (f1 + f2 + f3 ) = zf3 .
Adding these three results gives the required equation.

Solution for Exercise 1.64


Since fx = 2x and fy = 2y the implicit function theorem shows that y(x) and x(y)

exist if y 6= 0 and x 6= 0, respectively, and then
y (x) = f x /fy = x/y.

2 2
If f = 0 then y = 1 x , hence y = 1 x2 and y (x) = x/ 1 x2 = x/y.
15.1. SOLUTIONS FOR CHAPTER 1 417

Solution for Exercise 1.65


If f = x cos xy, fy = x2 sin u and fx = cos uu sin u, where u = xy. Thus fy (1, /2) =
1 and fx (1, /2) = /2. Hence, from the implicit function theorem, y(x) exists in
the neighbourhood of (1, /2), with
fx cos u u sin u 1
y (x) = = or x2 y = u,
fy x2 sin u tan u
hence y (1) = /2. Differentiating again gives
 
1
y x2 + 2xy = + 1 (xy + y) .
sin2 u
At x = 1, y = /2, since y (1) = /2, this gives y = . Hence the Taylor expansion
of y(x) about x = 1 is
1
y(x) = y(1) + (x 1)y (1) + (x 1)2 y (1) +
2

= (x 1) + (x 1)2 + .
2 2 2

Solution for Exercise 1.66


Differentiate the equation x3 +y 3 3axy = 0 and re-arrange to give (y 2 ax)y +x2 ay =
0, which gives the relation for y (x). Hence y is defined provided the denominator is
not zero, that is y 2 6= ax. The curve defined by f (x, y) = 0 is parallel to the x-axis if
x2 = ay, which substituted into the equation gives x3 (x3 2a3 ) = 0. At x = 0, y is
not defined; the solution at x = a21/3 gives the quoted result.

Solution for Exercise 1.67


From the graphs of y = 1/x and y = tan x, shown in figure 15.1, we see that the
equation has positive roots xk , k = 0, 1, 2, , and that k < xk < (k + 1/2) and
that for large k, xk k from above.

0 2 4 6 8 x 10 12 14 16
1

2
Figure 15.1 Graphs of y = 1/x and y = tan x.

For the nth root, put x = n + z, and since sin x = (1)n sin z and cos x = (1)n cos z
the equation becomes
(n + z) tan z = 1 with z small.
Put = 1/n so the equation becomes (1 + z) tan z = and we require the Taylor
expansion of z() about = 0. Putting = 0 we see that z(0) = 0. Differentiation gives
1 + z
(z + z) tan z + z = 1 giving z (0) = 1,
cos2 z
418 CHAPTER 15. SOLUTIONS TO EXERCISES
1
and hence x = n + .
n
Further differentiation of the same equation allows, in principle, the calculation of
z (n) (0) for n > 2; however, such calculations are extremely tedious and error prone. A
far easier method is now outlined.
First, rewrite the equation for z in the form

tan z =
1 + z
and observe that this equation defines a function z(), with z(0) = 0, that is an odd
function of to see this note that z() satisfies the same equation. Also, for small
|z| we see that to O() the equation becomes z = + O(2 ). The power series for z()
is thus
z = + z3 3 + z5 5 + O(7 ),
where z3 and z7 are coefficients to be found. Substitute this series in to the left-hand
side of the equation and use the known series for tan z to obtain
 3 3 2
tan z = + z3 3 + z5 5 + + 1 + z3 2 + + 5 +
   3  15
3 1 5 2
= + z3 + + z5 + z3 + + .
3 15
Similarly the right-hand side gives

= 1 z + 2 z 2 +

1 + z
= 3 + 5 (1 z3 ) + .
Equating the coefficients of the powers of on each side of the equation gives z3 = 4/3
and z5 = 53/15 and hence
1 4 53
x = n + 2
+ + .
n 3(n) 15(n)3

Solution for Exercise 1.68


Using the Taylor expansion of cos z the numerator becomes
(2x)2 (2x)4 (4x)2 (4x)4
   
1+a 1 + + + b 1 + + ,
2 24 2 24
which simplifies to
 
2 4 2 32
1 + a + b x (2a + 8b) + x a + b + .
3 3
Thus we need a + b + 1 = 0, a + 4b = 0, that is b = 1/3 and a = 4/3. Then the value
of the function at the origin is 2a/3 + 32b/3 = 8/3.

Solution for Exercise 1.69


There are many ways to obtain the expansions, but usually a direct use of the definition,
which requires the calculation of higher derivatives, is awkward and error prone: it is
usually easiest to use known results where possible. The methods outlined below are
not necessarily the easiest, but just the first I thought of.
15.1. SOLUTIONS FOR CHAPTER 1 419

(a) Since the Taylor series of ln(1 + z) is known we write, with u = x/2, and use the
identity cosh 2u = 1 + 2 sinh2 u,

ln(cosh x) = ln 1 + 2 sinh2 u


1 2 1 3
= 2 sinh2 u 2 sinh2 u + 2 sinh2 u + O(u8 ).
2 3
u2 u2
   
Now use sinh u = u 1 + + and sinh2 u = u2 1 + + in this expansion,
6 3
to give

u4 x2 x4
 
2
ln(cosh x) = 2u 1 + + 2u4 + = + O(x6 ).
12 2 12

(b) Similarly

1 1 1
ln(1 + sin x) = sin x sin2 x + sin3 x sin4 x + O(x5 ).
2 3 4
x2

sin x = x 1 +
6

giving

x2 x2 x2 x3 x4
   
ln(1 + sin x) = x 1 + 1 + + + O(x5 ),
6 2 3 3 4
x2 x3 x4
= x + + O(x5 ).
2 6 12

(c) Similarly

sin2 x sin3 x sin4 x


exp(sin x) = 1 + sin x + + + + O(x5 ),
2 6 24
x2 x2 x2 x3 x4
   
= 1+x 1 + + 1 + + + + O(x5 ),
6 2 3 6 24
x2 x4
= 1+x+ + O(x5 ).
2 8

(d) Use the identity sin2 x = (1 cos 2x)/2 to give

(2x)2 (2x)4 x4
 
1 1
sin2 x = 1 + + O(x6 ) = x2 + O(x6 ).
2 2 2 24 3

Solution for Exercise 1.70


The Cauchy form of the Mean Value Theorem gives f (x + 1) = f (x) + f (x + ) with
0 < < 1. Since f (x) is strictly increasing f (x) < f (x + ) < f (x + 1) and the result
follows.
420 CHAPTER 15. SOLUTIONS TO EXERCISES

Solution for Exercise 1.71


In the first case, for any x > 0 the Mean Value Theorem gives, for 0 < < 1,

x2
f1 (x) = f1 (0) + xf1 (x) = < 0.
1 + x
Hence, f1 (x) < 0 for x > 0. Similary f2 (x) > 0 for x > 0.

Solution for Exercise 1.72


sin ln x cos ln x 1 1
Using LHospitals rule, lim 5 3
= lim 4 2
= .
x1 x 7x + 6 x1 x 5x 21x 16
Solution for Exercise 1.73
2
In the first case set y = (cos x)1/ tan x and consider the limit of ln y,

sin x cos3 x
 
ln cos x 1
lim = lim = , hence lim y = e1/2 .
x0 tan2 x x0 cos x 2 sin x 2 x0

For the second case,

a sin bx b sin ax cos bx cos ax b sin bx a sin ax


lim = ab lim = ab lim
x0 x3 x0 3x2 x0 6x
ab  ab 2
= 2 2
lim b cos bx a cos ax = (a b2 ).
6 x0 6

Solution for Z Exercise 1.74 Z


tan1 (ax) 1
If I(a) = dx then I (a) = dx . Using partial
0 x(1 + x2 ) 0 (1 + x2 )(1 + a2 x2 )
fractions this becomes
Z
a2
 
1 1 1 1
I (a) = 2
dx 2
2 2
= 2
(1 a) = .
1a 0 1+x 1+a x 1a 2 2 1+a

Now integrate to obtain I(a) = ln(1 + a) + C giving I(0) = C. But, from the original
2
integral I(0) = 0, and hence C = 0.

SolutionZfor Exercise 1.75


/2 Z /2
ln(1 + z cos x) 1
If I(z) = dx then I (z) = dx . Now use the identity
0 cos x 0 1 + z cos x
2 2
cos x = (1 t )/(1 + t ), t = tan(x/2) to obtain
1
2 1 1+z
Z

I (z) = dt
2 + t2
, b2 = ,
(1 z)0 b 1 z
r
2 1z
= tan1 .
1z 2 1+z

But z = cos a and so this becomes


dI a dI 1
= or = 2 a and hence I(a) = C 2 a2 .
dz sin a da 2
2
But if cos a = 0, that is a = 1/2, I = 0. Hence C = 2 /8 and I(a) = (1 4a2 ).
8
15.1. SOLUTIONS FOR CHAPTER 1 421

Solution for Exercise 1.76


We have
/2 /2 /2  
sin x sin(/2 x) sin x cos x 1 1 1
Z Z Z
I= dx = dx = dx sin 2x + .
0 x /2 x 0 x(/2 x) 0 x /2 x

Now put y = 2x to give

1
 
1 1
Z
I = dy sin y +
0 y y
Z
1 sin y 1 sin( z) 2 sin y
Z Z
= dy + dz = dy , (z = y),
0 y 0 z 0 y

since sin( z) = sin z.

Solution for Exercise 1.77


1/z
1 1
Z Z
1
Put x = 1/z and s = 1/t to obtain tan (1/z) = dt = ds .
0 1 + t2 z 1 + s2
Hence
  Z x Z Z
1 1 1 1 1 1
tan x + tan = dt 2
+ ds 2
= dt 2
= .
x 0 1 + t x 1 + s 0 1 + t 2

Solution for Exercise 1.78


Differentiation gives g (x) = 2f (2x) f (x) so g (x) = 0 when 2f (2x) = f (x). In the
first case this gives 2e2x = ex and hence x = ln 2 is the only real solution.
sin 2x sin x
In the second case the equation becomes = . Since x 6= 0, the equation
x x
becomes 2 sin x cos x = sin x, hence sin x = 0, that is x = n, n = 1, 2, , or
cos x = 1/2, that is x = 2n /3, with n an integer.

Solution for Exercise 1.79


The definition 1.46 of an integral, with xk = a + kh/n, k = 1, 2, , n gives
n   Z a+h
hX kh
lim f a+ = dx f (x).
n n n a
k=1

(a) Put f (x) = x5 , a = 0 and h = 1


n  5 Z 1 n
1X k 1 X 5 1
lim = dx x5 hence lim k = .
n n n 0 n n6 6
k=1 k=1

(b) Put f (x) = 1/(1 + x), a = 0, h = 1 and sum from k = 1 to k = 2n

2n 2 2n 2
1X 1 1 1 1
Z X Z
lim = dx hence lim = dx = ln 3.
n n 1 + k/n 0 1+x n n+k 0 1+x
k=1 k=1
422 CHAPTER 15. SOLUTIONS TO EXERCISES

(c) Consider the complex sum, with f = eixy , a = 0 and h = 1,


n Z 1
1 X iky/n
S = lim e = dx eixy
n n 0
k=1

Hence
1 iy  sin y 2i 2  y 
S= e 1 = + sin
iy y y 2
and hence
     
1 y 2y 2 y
lim sin + sin + + sin y = sin2 .
n n n n y 2
h i1/n
1
(d) If Pn = n (n + 1)(n + 2) . . . (2n) then
n n
1X 1X
ln Pn = ln n + ln(k + n) = ln(1 + k/n).
n n
k=1 k=1

But, with f (x) = ln(1 + x), a = 0 and h = 1


n Z 1
1X
lim ln(1 + k/n) = dx ln(1 + x) = ln 4 1.
n n 0
k=1

Hence lim Pn = exp(ln 4 1) = 4/e.


n

Solution for Exercise 1.80


In these two cases the formula 1.52 does not work, because the integrand is infinite at
x = u. However, it is clear that both F (u) and G (u) exist in some cases, for instance
f = g = 1 or x, so the equivalent of expression 1.52 ought to exist.
In the first case the simplest method is to remove the singularity at x = u using the
standard change of variable x = u sin to give
Z /2
F (u) = d f (u sin ).
0

Now we can use equation 1.52 to give


Z /2 Z /2

F (u) = d f (u sin ) = d f (u sin ) sin .
0 u 0

This, second expression, may be converted back to an integral over x,


1 u xf (x)
Z

F (u) = dx .
u 0 u2 x2
In the second case we use another, more general trick. Consider the integral
Z u
g(x)
G (u) = dx , 0,
0 (u x)a
15.1. SOLUTIONS FOR CHAPTER 1 423

for which equation 1.52 is valid, when > 0. This gives


Z u  
g(u ) 1
G (u) = + dx g(x) , > 0.
a 0 u (u x)a
   
1 1
Now write = and integrate by parts
u (u x)a x (u x)a
u Z u
g (x)

g(u ) g(x)
G (u) = a
a
+ dx ,
(u x) 0 0 (u x)a
Z u
g(0) g (x)
= + dx ,
ua 0 (u x)a

and take the limit 0 to obtain


u
g(0) g (x)
Z
G (u) = + dx .
ua 0 (u x)a
424 CHAPTER 15. SOLUTIONS TO EXERCISES

15.2 Solutions for chapter 2


Solution for Exercise 2.1
A3 x
(a) Differentiating the given solution gives y = = xy 3 . This solution
(1 A2 x2 )3/2
is singular at x = 1/A, which depends upon the initial condition, y(0) = A. The
equation is nonlinear.

(b) Differentiating the given solution gives

3A 1 2 y 2 3A 1
y = + x but also x = x = y .
3x2 3 x 3 3x2
This solution is singular at x = 0, which is independent of the initial condition, y(1) =
A. The equation is linear.

Solution for Exercise 2.2


Equations a, e, g and i are linear, the rest are nonlinear. Exercise (h) is actually two
equations y = 1, y(1) = 1, with the solutions y = x and y = x 2.

Solution for Exercise 2.3


Equations f and g are initial value problems: equations a, b, and e are boundary value
problems: equations c and d are neither initial nor boundary value problems: equation h
is not a differential equation.

Solution for Exercise 2.4


(a) Defining x = y gives y +f (x)y +g(x) = 0, and these are the two coupled equations.

(b) Since xf (x) = dF (x)/dt, Lienards equation can be written as


 
d x
+ F (x) + g(x) = 0,
dt

suggesting that we set z = x/ + F (x), which gives the required coupled equations.
Note we could also set z = x + F (x), to obtain a different pair of equations, but the
former are more useful when 1.

Solution for Exercise 2.5


Differentiation gives

p = p x + p + f (p)p , that is (x + f (p))p = 0.

One solution is p = 0, that is, p = c, a constant, so the general solution is y = cx+f (c).
A particular solution is given by the other solution of this equation, x = f (p),
which gives p in terms of x, and by substituting this into the original equation we obtain
one, or more, particular solutions. The geometric interpretation of this is discussed in
section 2.6.
15.2. SOLUTIONS FOR CHAPTER 2 425

Solution for Exercise 2.6


From exercise 2.5 we see that the general solution is y = cx ec . The singular solution
is obtained by eliminating c from the equations

y = cx ec and x = ec , which gives y = x(ln x 1).

Solution for Exercise 2.7


If y = p, then y = p and the equation becomes the first-order equation F (x, p, p ) = 0.

Solution for Exercise 2.8


dp dp dy dp
Use the chain rule: = = p , hence the result.
dx dy dx dy

Solution for Exercise 2.9


In this example equation 2.12 becomes, since F (x, y) = y,
Z x
yn+1 (x) = 1 + dt yn (t), y0 (x) = 1.
0

Thus the first, second and third iterates are

1 1 1
y1 = 1 + x, y2 = 1 + x + x2 , y3 = 1 + x + x2 + x3 .
2 2 3!

The kth iterate adds the term xk /k!, so the nth iterate should be

1 1
yn (x) = 1 + x + x2 + + xn .
2 n!
This expression is correct for n = 1 and 2 and by substituting into the iterative formula
we see that yn+1 has the same form, hence it is true for all n.
The series for y(x) is a power series with coefficients un = 1/n!. The radius of
un
convergence is given by, see equation 1.32 (page 32), lim = , so the series is
n un+1
valid for all x.

Solution for Exercise 2.10


(a) In this example equation 2.12 becomes, since F (x, y) = 1 + xy 2 ,
Z x
yn+1 (x) = A + x + dt tyn (t)2 , y0 (x) = A.
0

Thus the first two iterates are


1
y1 = A + x + A2 x2 ,
2
1 2 2 2 3 1 1 1
y2 = A + x + A x + Ax + (1 + A3 )x4 + A2 x5 + A4 x6 .
2 3 4 5 24
426 CHAPTER 15. SOLUTIONS TO EXERCISES

(b) From the equation y (0) = 1. Differentiating the equation gives y (x) = y 2 + 2xyy ,
so y (0) = A2 . Another differentiation gives y (x) = 4yy + 2xy 2 + 2xyy , so y (0) =
4A. Thus the third-order Taylor series is
1 2
y(x) = A + x + A2 x2 + Ax3 .
2 3
This example shows that the iterative scheme, to a given order, usually produces more
terms of the series solution than the equivalent order Taylor series.
(c) Since 1 + xy 2 for x 0, y > 0 and hence y(x) is monotonic increasing.
For sufficiently large x, xy 2 1, so y z where z = xz 2 and this equation has the
solution 2/z = c2 x2 , where c is a constant. As x c, z(x) . But y(x) > z(x)
for all x > 0, thus y(x) also tends to infinity at some finite x: the actual position of
this singularity depends upon A, but is difficult to determine, but see exercise 2.21
(page 68).

Solution for Exercise 2.11


Throughout this solution C and other captial letters denote arbitrary constants.
(a) The equation separates directly to give
 
dy x 1+y
Z Z
2
= dx giving ln = C + ln(1 + x2 )
1y 1 + x2 1y

A(1 + x2 ) 1
which rearranges to y = .
A(1 + x2 ) + 1
(b) Write the equation in the form (1 + x)y = x(1 + y) which separates to
dy x Aex
Z Z
= dx giving ln(1 + y) = C + x ln(1 + x) that is y = 1.
1+y 1+x 1+x

(c) This equation separates directly and, using the initial conditions, gives
Z y Z x  2
dy x 1 1
= dx hence y = 1 + x ln(1 + x) 1.
0 1+y 0 1+x 2 2

(d) If z = x + y the equation becomes


dz 1 + 2z dz 2
1= that is = .
dx 1 2z dx 1 2z
The last equation separates to give
Z
dz (1 2z) = 2x + C that is z z 2 = 2x + C.

This is a quadratic equation for z, and hence y, with the two solutions
1 
y= 1 2x 1 4C 8x .
2
15.2. SOLUTIONS FOR CHAPTER 2 427

Solution for Exercise 2.12


Throughout this solution C and other captial letters denote arbitrary constants.
(a) If y = xv the equation becomes

dv dv F (v) v
x + v = F (v) which rearranges to = .
dx dx x

(b) In this example F (v) = v + ev , so the separable form is

dv ev
= which gives ev = C + ln x that is y = x ln (C + ln x) .
dx x

(c) If y = xv the equation becomes

dv 1 + 3v dv 1 v2
x +v = , that is x = .
dx 3+v dx v+3
One solution is v 2 = 1, that is, y = x. The equation for v separates to
 
v+3 dx 2 1
Z Z Z
dv = or dv + = C + ln x.
1 v2 x 1v 1+v
1+v
Hence = Ax and this becomes
(1 v)2
x+y 1  
2
= A with solutions y = 1 + Bx 1 + 4Bx , (B = 2A).
(x y) B

The initial condition, y(1) = 0, gives B + 1 4B + 1 = 0; only the minus sign gives
real roots and these are B = 0 and 2. The solution B = 2 gives
1 
y= 1 + 2x 1 + 8x .
2
If B = 0 we need to take the limit as B 0, or expand in B for small B: using
LHospitals rule we obtain y = x, which is one of the particular solutions found
above.

1 1 + 4Bx
The other solution, y = x + + x as B , and gives the other
B B
particular solution found above.
(d) If y = xv the equation becomes

dv 1 + v2 dv 1v
x +v = which rearranges to x = .
dx 1+v dx 1+v
Hence  
2 dx
Z Z
dv 1 = that is ev = A|x|(1 v)2 .
1v x
This gives |x|ey/x = A(x y)2 ; here y cannot be expressed as a simple formula in x.
428 CHAPTER 15. SOLUTIONS TO EXERCISES

(e) If y = xv the equation becomes

dv 3 v + 3v 2 dv 3(1 v)
x +v = which rearranges to x = .
dx 2 + 3v dx 2 + 3v
Hence
 
5 dx
Z Z
dv 3 =3 that is e3v |1v|5 = Ax3 or x2 e3y/x = B|xy|5 .
1v x
As in the previous example y cannot be expressed as a simple formula in x.
(f) If the two straight lines intersect at (a, b) we set = x a and = y b, that is we
move the point of intersection to the origin. The equation becomes

d 4 3 dv 2v 2 + 3v 2
= and with = v(), = 2
d 3 + 4 d 3 + 4v
that is
3 + 4v d
Z Z
dv = 2 .
2v 2 + 3v 2
Integration gives ln(2v 2 + 3v 2) = C 2 ln or (2v 1)(v + 2) = A/ 2 , where A is an
arbitrary constant. The lines intersect at (1, 1) hence the solution is (2y x 1)(y +
2x 3) = A.
If we set A = 0 in this general solution we obtain the two particular solutions 2y x = 1
and y + 2x = 3, which can also be derived from the equation for () by setting = k
and solving the resultant quadratic to give k = 1/2 and k 2.

Solution for Exercise 2.13


(a) The integrating factor is
Z 
x+3
p(x) = exp dx = exp (x + ln(x + 2)) = (x + 2)ex ,
x+2
so the equation can be written as
d 4x + A x
((x + 2)ex y) = 4 with the general solution y = e .
dx x+2
This solution is singular at x = 2, where the coefficient of y is zero.
(b) In this case the formula for the integrating factor, equation 2.17, is not convenient
because P (x) = tan x so p(x) = 1/| cos x|, which is messy. It is better to simply write
the original equation in the form of 2.16, that is
d  y 
cos2 x = 2 cos2 x sin x hence y = C cos x 2 cos2 x.
dx cos x
This is the general solution. If y(0) = 0, C = 2 and the solution is y = 2(1 cos x) cos x.
In this example the coefficient of y is zero where cos x = 0, but the solution is well
behaved everywhere. If we set x = z /2 and expand about z = 0 the equation
15.2. SOLUTIONS FOR CHAPTER 2 429

becomes zy (z) y = 2z 2, which has the general solution y = Az 2z 2. This equation


is similar to the equation considered in exercise 2.1, part (b), the main difference being
that the coefficient of y has a different sign: the solution of this equation is singular at
x = 0.
This example shows that solutions of first-order linear equations need not be singular
at the points where the coefficient of y is zero.

(c) The integrating factor is


Z  
1 2
p(x) = exp dx = x2 e1/x ,
x2 x

so the equation becomes

e1/x 1
Z

(yp) = 4 which gives yp = A dz z 2 ez , z= .
x x

Hence the general solution is y = Ax2 e1/x 1 2x 2x2 . This linear equation has an
essential singularity at x = 0, where the coefficient if y has a repeated zero.

(d) On substituting x = 0 into the equation we see that y (0) = 0, so the solution is
stationary at x = 0.
The equation can be written in the form

d
cos x (y cos x) = (1 + cos2 x) tan x, y(0) = 2, and hence
dx
Z x Z c
1 + cos2 x
 
1
y cos x 2 = dx sin x = dc 1 + , (c = cos x)
0 cos2 x 1 c2
1 2 1
= cos x , hence y = 1 + .
cos x cos x cos2 x
This solution can be written as y = 2 (1/ cos x 1)2 , which, by inspection, has
a local maximum at x = 0 (because 1/ cos x has a minimum here). In fact since
1/ cos x = 1 + x2 /2 + O(x4 ), y = 2 x4 /4 + , so y (0) = y (0) = 0, and the solution
has a very flat maximum at x = 0.
The solution is singular where cos x = 0, which is where the coefficients of y are zero.

Solution for Exercise 2.14


(a) Equation 2.20 can be integrated directly by writing it in the form
Z x  Z x 
dy
Z Z
= dx P (x) which gives ln y = dt P (t), that is y = exp dt P (t) .
y a a

(b) If y = vf , equation 2.16 becomes

Q
v f + v (f + f P )) = Q, but f + f P = 0 and hence v = .
f
430 CHAPTER 15. SOLUTIONS TO EXERCISES
Rx
Because f (a) = 1, this gives v = A + adt Q(t)/f (t), and hence the general solution is
 Z x 
Q(t)
y(x) = f (x) A + dt .
a f (t)

Since f (x) = 1/p(x), where p(x) is defined in equation 2.17 (page 63), we see that this
solution is identical to that in equation 2.19.

Solution for Exercise 2.15


If p = y (x) the equation becomes the first-order equation
dp p d p
= 3x, that is = 3,
dx x dx x
which integrates to
dy 1
p= = c1 x + 3x2 and hence y = c2 + c1 x2 + x3 .
dx 2
1
The initial conditions give A = c2 + 21 c1 +1, A = c1 +3, and hence y = (2A A + 1)+
2
1
(A 3)x2 + x3 .
2
Solution for Exercise 2.16
If p = y (x) the equation becomes pp (y) + 2 y = 0, which can be written as
d dy p
p2 + 2 y 2 = 0 giving

= C 2y2,
dy dx

and since y (0) = 0 and y(0) = A, C = 2 A2 and


Z y Z x
dy
p = dx and hence y = A cos x.
A A2 y 2 0

Solution for Exercise 2.17


(a) Here n = 2 so z = 1/y and the equation for z is
d 4
ze2x = xe2x

giving y = .
dx 2x 1 + 5e2x

(b) Here n = 3 so z = 1/y 2 and the equation for z is

dz 2(2x2 1) 2x
z= .
dx x(1 x2 ) 1 x2

The integrating factor is p(x) = x2 (1 x2 ) and the equation for z becomes



d x 2 1 x2
x2 (1 x2 )z = 2x3 which gives y =

.
dx A x4
15.2. SOLUTIONS FOR CHAPTER 2 431

(c) Here n = 3 so z = 1/y 2 and the equation for z is


dz d  z 
cos x + 2z sin x = 2 cos2 x or cos3 x = 2 cos2 x,
dx dx cos2 x
which integrates to
 
z 1 + sin x 1
= A 2 ln giving, since y(0) = 1, y = .
cos2 x
q
cos x cos x 1 2 ln 1+sin x cos x

(d) By writing the equation in the form y y/x = y 2 /x3 we see that n = 2. Putting
z = 1/y gives xz + z = x2 , that is (xz) = x2 , which integrates to give
y = x2 /(1 + Ax).

Solution for Exercise 2.18


The easiest way of doing this problem is to express c in terms of x and then differentiate.
G yF
This gives c = and differentiation gives
yf g
G y F yF G yF
0= (f y + f y g ) ,
fy g (f y g)2
which rearranges to
(F g f G)y = (gG Gg ) + (F g F g + f G f G ) y + (f F f F ) y 2 ,
which is Riccatis equation.

Solution for Exercise 2.19


Differentiating equation 2.25 gives
u u 2 u R Qu u 2
y = + 2 + = P + from equation 2.23 for y .
uR u R uR2 uR u2 R
Notice that the nonlinear terms, u 2 , cancel. Rearranging this gives u (Q+R /R)u +
P Ru = 0.

Solution for Exercise 2.20


R
(a) We assume that u(x) 6= 0, so ln u = dx y and
du d2 u
= yu and = y u + yu = (y + y 2 )u.
dx dx2
Substituting this into the second-order equation gives p2 (u(y + y 2 ) + p1 yu + p0 u = 0.
Division by u gives the required result.
(b) The general solution of Riccatis equation is y(x, c), where c is the arbitrary con-
stant Z
of integration. The general solution of the second-order equation is u(x) =
x
A exp dt y(t, c) which contains two arbitrary constants, A and c.
a
Thus if u(a) = A and u (a) = A , then the equations relating A, ! A and c are
Z b
A = Ay(a, c). If u(a) = A and u(b) = B, then B = A exp dx y(x, c) .
a
432 CHAPTER 15. SOLUTIONS TO EXERCISES

Solution for Exercise 2.21


(a) In this example P = 1, Q = 0 and R = x, so y and u are related by y = u /(xu)
and u satisfies the equation xu u + x2 u = 0.

(b) If
u = a0 + a1 x + a2 x2 + + an xn + ,

then

xu = +2a2 x +6a3 x2 + +n(n + 1)an+1 xn ,


u = a1 2a2 x 3a3 x2 (n + 1)an+1 xn ,
x2 u = +a0 x2 + +an2 xn .

Substituting these series into the equation for u and collecting the powers of x gives
x0 : a1 = 0,
x1 : 0 = 0,
x2 : 3a3 + a0 = 0,
xn : (n2 1)an+1 + an2 = 0, n 2.
Thus the values of a0 and a2 are undetermined and a1 = 0. We obtain two independent
solutions by setting (a0 , a2 ) = (1, 0) and (0, 1). The first of these gives a5 = a8 =
a3k+2 = 0 and the second gives a3 = a6 = a3k = 0, k = 1, 2, , so the two solutions
are

u1 (x) = 1 + a3 x3 + a6 x6 + a3k x3k + , a0 = 1,


2 3 6 3k

u2 (x) = x 1 + b5 x + b8 x + + b3k+2 x + , a0 = 0.

The recurrence relation for a3k is given by


an3 a3(k1)
an = that is a3k = , a0 = 1.
(n 1)2 1 (3k 1)2 1

The first few terms are

1 1 (1)k
a3 = 2
, a6 = suggesting that a3k = .
2 1 (2 1)(52 1)
2 (22 1)(52 1) ((3k 1)2 1)

The latter expression satisfies the recurrence relation, and is true for k = 1 and 2, so
by induction it is true for all k 1. Similarly

(1)k
b3k+2 = .
(42 1)(72 1) ((3k + 1)2 1)

The series for u1 and u2 are power series in x3 and using equation 1.32 (page 32) the
radii of convergence are given by

a3k
r13 = lim = and r3 = lim b3k+2 = .

2
k a3k3 k b3k1

15.2. SOLUTIONS FOR CHAPTER 2 433

(c) The general solution is u(x) = c1 u1 (x+ c2 u2 (x), for some constants c1 and c2 , so

c1 u1 (x) + c2 u2 (x)
y =
x (c1 u1 (x) + c2 u2 (x))
c1 (3a3 x + ) + c2 (2 + 5b5 x3 + )
= .
c1 (1 + a3 x3 + ) + c2 (x2 + )
Thus the initial condition gives
2c2 2u1 (x) Au2 (x)
A= and hence y(x) = .
c1 x (2u1 (x) Au2 (x))
The denominator of y is

x6
 
A 1 A
d(x) = x 1 x2 x3 + x5 + + .
2 3 30 72
p
If A 1 then the first two terms show that d(x) is zero
close to x = 2/A. At this
point the 3 rd and 4 th terms of the bracketed term are 2 3 2 A3/2 and 2152 A3/2 , so we
p
deduce that y(x) has a pole near x = 2/A.
2A
Note that the similar equation y = xy 2 , y(0) = A, has the solution y = , which
p 2 Ax2
has a pole at x = 2/A.

Solution for Exercise 2.22


(a) With v = a the right-hand side is x(a2 2a + 1) + a 1, which is zero only if a = 1.
Then P1 = 1 and the equation for z becomes
d
ex (zex) = x giving z = Aex + 1 x.
dx
Hence the general solution is
1 1 1
y =1+ so y(0) = 1 + = giving A = 3.
1 x + Aex A+1 2

(b) With v = ax + b the equation becomes

a = 1 (b2 1)x 2b(a 1)x2 (a 1)2 x3

so there are two particular solutions, v1 = x + 1 and v2 = x 1. Using equation 2.28


we see that the general solution is given by

x + 1 A(x 1) exp(x2 )
 
yx1
Z
ln = dx 2x that is y = .
yx+1 1 A exp(x2 )

If y(1) = 1, A = e so
x + 1 + (x 1) exp(1 x2 )
y= .
1 + exp(1 x2 )
434 CHAPTER 15. SOLUTIONS TO EXERCISES

(c) Inspection shows that v = x is a particular solution. This satisfies the condition
v(1) = 1, so is the required solution.

(d) Substituting v = aebx into the equation gives

a(2b 1)ebx + ex ae(b+1)x a2 e2bx = 0.

Inspection shows that this equation is satisfied for all x if b = 1 and a = 1 so a


particular solution is y = ex ; this satisfies the condition at x = 0, and is hence is the
unique solution.
A general solution can be expressed in the form y = 1/z ex , but this initial condition
gives 1/z(0) = 0, so there is no solution of this form.

Solution for Exercise 2.23


In this example P1 = f (x) + 2a and the integrating factor, equation 2.17, for the linear
equation defining z(x) is p(x) = exp 2ax + dx f (x) , and since (pz) = p the general
R 

solution for y is
p(x)
y(x) = a + R .
C dx p(x)

Solution for Exercise 2.24


In this case P = 1, Q = x and R = 1 and this corresponds to case 4 with f = a = n =
1, so the particular solution is v = x. Then P1 = x, equation 2.27, and the integrating
factor, equation 2.17, is p = exp(x2 /2), and the equation for z is (zp) = p and the
general solution for y is
exp(x2 /2)
y =x+ R .
C dx exp(x2 /2)
Writing this in the form

exp(x2 /2) 1
y =x+ Rx gives y(0) = = a,
C 0 dt exp(t2 /2) C

and hence
a exp(x2 /2)
y =x+ Rx .
1 a 0 dt exp(t2 /2)
Rx
This solution is singular when a 0 dt exp(t2 /2) = 1.

Solution for Exercise 2.25


(a) With y = Axa the equation becomes Aaxa+1 + 2 + A2 x2(a+1) = 2Axa+1 , so we need
to put a = 1 and then the equation becomes (A 1)(A 2) = 0, so the two solutions
are v1 = 2/x and v2 = 1/x. Thus, since R(x) = 1 the general solution is given by
 
y v1 dx xy 2 B 2x B
Z
ln = that is = or y = .
y v2 x xy 1 x x(x B)
15.2. SOLUTIONS FOR CHAPTER 2 435

(b) In this example v = x and v = 1/x are particular solutions, and with y1 = x,
y2 = 1/x and R = 1/(1 + x), equation 2.28 becomes
   
yx x 1/x 1
Z Z
ln = dx = dx 1 .
y 1/x x+1 x

Integrating this and rearranging gives

x Bex
y= .
1 Bxex

(c) With y = Axa the equation becomes Aaxa+1 = 2 A2 x2a+2 , so a = 1 and


A2 A 2 = 0. Thus the two solutions are v1 = 2/x and v2 = 1/x. Since R = 1
equation 2.28 gives  
xy 2 dx
Z
ln = 3 = B 3 ln x
xy + 1 x
2x3 + b
which rearranges to y = .
x(x3 b)

Solution for Exercise 2.26


By comparing the coefficients of y k , k = 0, 1 and 2 we see that f (x) = 1/(1 + x2 ),
n = 2, a = 0 and b is replaced by b2 . Putting y = x2 z gives

dz x
= (b2 + z 2 )
dx 1 + x2
which is separable, and gives
   
b 2 b b 2
z = b tan A + ln(1 + x ) that is y = 2 tan A + ln(1 + x ) ,
2 x 2

where A is an arbitrary constant.

Solution for Exercise 2.27


Two particular integrals are y = Axn : using equation 2.28 with v1 = Axn and
v2 = Axn gives
y Axn
  Z
ln = 2A dx xn1 f (x) = F (x)
y + Axn
y Axn
hence = BeF (x) , which gives the quoted result.
y + Axn

Solution for Exercise 2.28


w w w 2
 
1
(a) If y = , then y = 2 and the equation y = bxn + ay 2 becomes
aw a w w

d2 w
+ abxn w = 0.
dx2
436 CHAPTER 15. SOLUTIONS TO EXERCISES

Now set z = x and w = z u(z) so that


dw d  dz 
= z u = z u x1 ,
dx dz dx
d2 w  
= 2 z u x22 + ( 1) z u x2
dx2
 
= 2 z u z 22/ + ( 1) z u z 12/ .

Since
 
z u = z u + z 1 u and z u = z u + 2z 1 u + ( 1)z 2 u,

the equation for u(z) becomes


   
2 z 22/ z u + 2z 1 u + ( 1)z 2u +(1)z 12/ z u + z 1 u +abz +n/u = 0.

Expanding this gives


   
1 ab n+2
z 2 u + zu 2 + 1 + 2 + 2 z u = 0.

This equation can be converted to a form similar to Eulers equation, see section 2.4.6,
by setting 2 + 1 1/ = 1, that is, 2 = 1; then cast it into a form similar to Bessels
equation by setting n + 2 = 2. This gives
 
4ab 1
z 2 u + zu + z 2
u = 0.
(n + 2)2 (n + 2)2

2 ab
Finally define = z to give Bessels equation
n+2
d2 u du 1
2 + + ( 2 2 )u = 0, = ,
d 2 d n+2
with the general solution
1 n+2 1
u = AJ () + BY (), = , = , = , n 6= 2,
n+2 2 n+2
where A and B are arbitrary constants.
(b) If n = 2 the equation for w(x) is x2 w (x) + abw = 0, which has solutions of
the form w = Ax , for some constants A and . Substituting into the equation gives
2 + ab = 0, to give the two solutions
1+d
w = Ax1 , 1 = , d= 1 4ab,
2
1d
w = Ax2 , 2 = .
2
These give two particular solutions for y,
1 2
v1 = and v2 = .
ax ax
15.2. SOLUTIONS FOR CHAPTER 2 437

The general solution is then obtained using equation 2.28,


  Z
axy + 1 2 1 axy + 1
ln = dx , that is, = Axd ,
axy + 2 x axy + 2

where A is an arbitrary constant. This rearranges to

A2 1 xd 1d
y= , 1, 2 = , d= 1 4ab.
ax(xd A) 2

Solution for Exercise 2.29


Suppose y() = y () = 0, then a solution is y(x) = 0 for all x, which, since by P2
this is the only solution, contradicts the assumption that y(x) is nontrivial. Hence if
y() = 0, y () 6= 0 and the zero is simple.

Solution for Exercise 2.30


The vectors are dependent only if parallel,  that is x y = |x||y|. The vectors are not
2
parallel if x y 6= |x||y|, that is x21 + x 22 y12 + y22 6= (x1 y1 + x2 y2 ) ; this rearranges
2 x x2
to (x1 y2 x2 y1 ) 6= 0 that is 1 6= 0.
y1 y2

Solution for Exercise 2.31


(a) By expanding equation 2.31, dividing by p(x) and comparing the coefficients of y
p p1 q p0
and y (after division by p2 (x)) we obtain = and = . Integrating the first
p p2 p p2
equation gives Z 
p1 p1
Z
ln p = dx that is p = exp dx .
p2 p2

(b) Putting y = uv in equation 2.31 gives


   
p u v + 2u v + uv + p u v + uv + quv = 0

which can be rearranged to


 
pvu + u (2pv + p v) + u qv + p v + pv = 0.

Choose v to make the coefficient of u zero, that is pv 2 = 1, to give

p 2 p
 
q
u p + + 3/2 u = 0.
p 4p 2 p

Dividing by p gives the quoted result.

Solution for Exercise 2.32


Since g (x) = F (f )f (x), W (f, g) = [f F (f )F (f )]f (x), so W = 0 only if F /F = 1/f,
that is g(x) = cf (x).
438 CHAPTER 15. SOLUTIONS TO EXERCISES

Solution for Exercise 2.33


The Wronskian is
  
W = a1 cos x a2 sin x b1 sin x + b2 cos x
  
a1 sin x + a2 cos x b1 cos x b2 sin x = a1 b2 a2 b1 .

The functions are linearly independent if W 6= 0, that is if a1 b2 6= a2 b1 .

Solution for Exercise 2.34


In this example p2 = 1 and p1 = 0 so equation 2.35 becomes
 Z x 
1
g1 (x) = f (x) C + W (a) ds .
a f (s)2

Putting C =Z 0 and g1 = W (a)g(x) (permissible because the equation is linear) gives


x
1
g(x) = f (x) ds .
a f (s)2
Differentiating this twice gives
Z x Z x
1 1 1
g (x) = f (x) ds 2
+ and g
(x) = f
(x) ds ,
a f (s) f (x) a f (s)2

and hence g + qg = 0.

Solution for Exercise 2.35


(a) The functions f and g satisfy the equations

f + p1 f + p0 f = 0 and g + p1 g + p0 g = 0.

Multiply the first by g, the second by f and subtract to obtain


f g f g
f g g f + (f g f g ) p1 = 0 that is p1 = .
W (f, g; x)

Multiply the first by g , the second by f and subtract to obtain


f g g f
f g g f + (f g gf ) p0 = 0 that is p0 = .
W (f, g; x)

(b) (i) If f = x and g = sin x then W = x cos x sin x and


x sin x sin x
p1 = and p0 =
x cos x sin x x cos x sin x
giving the equation (x cos x sin x)y + y x sin x y sin x = 0 which has singular points
at the roots of tan x = x.
(ii) If f = xa and g = xa+b then,

2a + b 1 a(a + b)
W = bx2a+b1 , p1 = and p0 =
x x2
15.2. SOLUTIONS FOR CHAPTER 2 439

giving the equation x2 y (2a + b 1)xy + a(a + b)y = 0 which has a singular point at
x = 0. The solution of this linear equation can be found in a variety of ways: one is to
put y = Ax to form a quadratic in , with solutions = a, a + b, giving the functions
f and g, as expected, and the general solution y = (C + Dxb )xa .
If b = 0, f = g and W (f, g) = 0. Nevertheless, the equation for y(x) exists and becomes
x2 y (2a1)xy +a2 y = 0; putting y = Ax gives (a)2 = 0, and hence the solution
y = xa . Now substitute y = xa v(x) into the equation to obtain x2 v + xv = 0, having
the solution v = A + B ln x. Thus when b = 0 the general solution is y = (A + B ln x)xa ,
which cannot be obtained from the general solution valid when b 6= 0.

(iii) If f = x and g = eax then W = (ax 1)eax and

xa2 a2
p1 = and p0 =
1 ax 1 ax

giving the equation (1 ax)y + xa2 y a2 y = 0 which has a singular point at x = 1/a.

Solution for Exercise 2.36


Differentiate the Wronskian and use the fact that f and g satisfy equation 2.30,

dW g   f  
= f g f g = p1 f + p0 f p1 g + p0 g
dx p2 p2
p1   p 1
= gf g f = W.
p2 p2

Integrate this equation,


ix Z x  Z x 
h p1 (t) p1 (t)
ln W = dt . Hence W (x) = W (a) exp dt .
a a p2 (t) a p2 (t)

Solution for Exercise 2.37


The following table gives the values of and the general solution for each case.
general solution
a) (3, 2) c1 exp(3x) + c2 exp(2x);
3 1
b) , ) c1 exp(3x/2) + c2 exp(x/2);
2 2
1 3  
c) i c1 cos( 3x/2) + c2 sin( 3x/2) ex/2 ;
2 2
d) 2 i e2x (c1 cos x + c2 sin x);
e) 3 2i e3x (c1 cos 2x + c2 sin 2x);
f) i c1 cos x + c2 sin x or c1 eix + c2 eix ;
g) c1 cosh x + c2 sinh x or c1 ex + c2 ex ;
h) k i ekx (c1 cos x + c2 sin x).
 
7
Applying the boundary conditions gives (d) 2e2x sin x, (e) e3x 2 cos 2x + sin 2x ,
2
b b
(f) a cos x + sin x and (g) a cosh x + sinh x.

440 CHAPTER 15. SOLUTIONS TO EXERCISES

Solution for Exercise 2.38


Substituting y = vex into equation 2.37 gives
a2 v + (2a2 + a1 ) v + a2 2 + a1 + a0 v = 0.


If a2 2 + a1 + a0 = 0 has only one solution, = a1 /2a2 , then v = 0.

Solution for Exercise 2.39


(a) The equation 2 6 + 9 = ( 3)2 has the repeated root = 3, so the general
solution is y = (c1 + c2 x)e3x . The first initial condition gives y(0) = c1 = 0; and since
y = c2 (1 + 3x)e3x, the second initial condition gives c2 = b, so the solution is y = bxe3x .
(b) The equation 2 + 2 + 1 = ( + 1)2 has the repeated root = 1, so the general
solution is y = (c1 + c2 x)ex . The boundary condition
 at x = 0 gives c1 = a; at x = X
X X x  x
we have (a + c2 X)e = b giving the solution y = a + (be a) e .
X

Solution for Exercise 2.40


The general solution of the homogeneous equation, y + 2 y = 0, is yg = A cos x +
B sin x. For the particular integral, we assume that yp = c0 + c1 x + c2 x2 ; substituting
this into the equation gives
2c2 + (c0 + c1 x + c2 x2 ) 2 = x2 .
Equating the coefficients of the powers of x gives 2c2 +c0 2 = 0, c1 2 = 0 and c2 2 = 1,
giving the general solution
2 x2
y = A cos x + B sin x + .
4 2
2
The initial conditions give y(0) = A = a, y (0) = B = b hence
4
x2
 
2 b 2
y = a + 4 cos x + sin x 4 + 2 .

Solution for Exercise 2.41


(a) If y = Aex , the equation becomes A(a2 2 + a1 + a0 ) = 1, hence the solution is
ex
y= provided a2 2 + a1 + a0 6= 0.
a2 2 + a1 + a0

(b) Put y = v(x)ex , so the equation becomes


a2 v + 2v + 2 v + a1 (v + v) + a0 v = 1.


Rearranging this gives a2 v + v (2a2 + a1 ) + v a2 2 + a1 + a0 = 1, and if a2 2 +



a1 + a0 = 0 this becomes v + (2 + a1 /a2 )v = 1/a2 , which is a first-order, linear
equation in v and hence
dv 1
= + Aex(2+a1 /a2 ) .
dx 2a2 + a1
15.2. SOLUTIONS FOR CHAPTER 2 441

A further integration gives

x Aa2
v= +B ex(2+a1 /a2 ) ,
2a2 + a1 2a2 + a1

where A and B are arbitrary constants. A particular integral is obtained by setting


A = B = 0 and then
xex
y= .
2a2 + a1
The general solution for y is obtained by retaining A and B.

Solution for Exercise 2.42


(a) In this example the equivalent of equation 2.38 is 2 + 1 = 0, so we expect a
particular solution of the form y = Cxeix . Substituting this into the equation gives
2Ci = 1, hence a particular solution is y = ixeix /2.
The general solution of the homogeneous equation is Aeix + Beix , so the general
solution is
1
y = Aeix + Beix ixex .
2
The initial conditions give a = A + B and b = (A B)i i/2 and hence
   
a 1 ib ix a 1 ib ix 1 ix
y= + e + + e ixe .
2 4 2 2 4 2 2

(b) The general solution of the homogeneous equation is A cosh x + B sinh x, so we


expect a particular solution of the inhomogeneous equation to be y = C sin x, and
substituting this into the equation gives C = 1/2. Thus the general solution is

1
y = A cosh x + B sinh x sin x.
2
The initial conditions give a = A and b = B 1/2, hence the solution is
 
1 1
y = a cosh x + b + sinh x sin x.
2 2

(c) The general solution of the homogeneous equation is A cosh 2x+B sinh 2x, so we ex-
pect a particular solution of the inhomogeneous equation to be y = C, and substituting
this into the equation gives 4C = 6. Thus the general solution is
3
y = A cosh 2x + B sinh 2x .
2
The initial conditions give A 3/2 = a and 2B = b, hence the solution is
 
3 1 3
y = a+ cosh 2x + b sinh 2x .
2 2 2
442 CHAPTER 15. SOLUTIONS TO EXERCISES

(d) The general solution of the homogeneous equation is A cos 3x + B sin 3x, so we
expect a particular solution of the inhomogeneous equation to be y = C + Dx, and
substituting this into the equation gives 9C + 9Dx 1 + 2x. Thus the general solution
is
1
y = A cos 3x + B sin 3x + (1 + 2x).
9
The initial conditions give A + 1/9 = a and 2/9 + 3B = b, hence the solution is
   
1 1 2 1
y= a cos 3x + b sin 3x + (1 + 2x).
9 3 9 9

(e) To find the general solution of the homogeneous equation we set y = ex to obtain
2 6 = 0, that is = 3 and 2: thus the general solution is Ae3x + Be2x . We
expect the particular integral to have the form y = C cos 2x + D sin 2x and substituting
this into the equation and equating the coefficients of cos 2x and sin 2x to zero gives
the linear equations D + 5C = 9 and 5D C = 7, with solutions D = 22/13,
C = 19/13. Thus the general solution is

19 22
y = Ae3x + Be2x cos 2x sin 2x.
13 13
The initial conditions give the equations

19 44
A+B =a+ , 3A 2B = b +
13 19
 
1 82 1
with solution A = + 2a + b and B = (1 + 3a b); hence the solution is
5 13 5
 
1 82 1 19 22
y= + 2a + b e3x + (1 + 3a b) e2x cos 2x sin 2x.
5 13 5 13 13

Solution for Exercise 2.43


The homogeneous equation is the same as in the text example, but with = 1, so
f = cos x, g = sin x and W (f, g) = 1. The equations for c1 (x) and c2 (x) are

1
c1 = tan x sin x = + cos x and c2 = tan x cos x = sin x,
cos x
with solutions
  x 
c1 (x) = sin x ln tan + and c2 (x) = cos x.
2 4
  x 
Thus y = c1 (x)f + c2 (x)g = cos x ln tan + . The general solution is
2 4
  x 
y = A cos x + B sin x ln tan + cos x.
2 4
15.2. SOLUTIONS FOR CHAPTER 2 443

Solution for Exercise 2.44


Using the new independent variable t, where x = et , the equation becomes
 2 
d y dy dy
2
+2 6y = 0.
dt dt dt

Set y = et to obtain 2 + 6 = 0, which has solutions = 3, 2. Hence

y = c1 e2t + c2 e3t = c1 x2 + c2 x3 ,

where c1 and c2 are arbitrary constants. The initial conditions give 1 = c1 + c2 and
0 = 2c1 3c2 , hence c1 = 3/5 and c2 = 2/5.

Solution for Exercise 2.45


Multiply the equation by x and set x = et , t 0, to give y (t) = 0. The general
solution is y(t) = c1 + c2 t, and the initial conditions give y(x) = A + A ln x.

Solution for Exercise 2.46


Using the results derived in the text, we have

d2 y d2 y
 
dy dy d dy dy
x = , x x = 2 = x2 2 + x
dx dt dx dx dt dx dx
and
d3 y d2 y d3 y d2 y
    
d d dy d dy dy
x x x = 3 =x x2 2 + x = 3 3 2 +2 .
dx dx dx dt dx dx dx dt dt dy
In terms of t the given equation becomes

d3 y d2 y dy
3
6 2
+ 11 6y = et/2 .
dt dt dx
The general solution of the homogeneous equation is found by substituting y = et into
the equation, to obtain 3 62 + 11 6 = ( 1)( 2)( 3) = 0, so the general
solution is y = A exp(t) + B exp(2t) + C exp(3t). For the inhomogeneous equation, put
y = det/2 and we find that d = 8/15, so the general solution is
8 1/2
y = Ax + Bx2 + Cx3 x .
15

Solution for Exercise 2.47


(a) The family of straight lines is,
x y x y
f (x, y, C) = + 1 = 0 giving fC (x, y, C) = + = 0.
C dC C2 (d C)2
Then fC = 0 when
r p
y d d y/x
d C = C or C = p , dC = p .
x 1 y/x 1 y/x
444 CHAPTER 15. SOLUTIONS TO EXERCISES

Substituting these relations into the equation f = 0 gives


 r   r 
y y y 2
x 1 p 1 = d, that is x y = d,
x y/x x

which
gives the four equations x y = d, x y = d, x + y = d and

x + y = d. The last of these cannot be satisfied for real x and y, thus there are
three real solutions:

y = x + d, x 0, y d, (A)

y = x d, x d, y 0, (B)

y+ x = d, 0 x, y d (C)

These three solutions are shown in figure 15.2.

Curve A Curve B Curve C


6 y 1
y y
5 0.5 0.8
4 0.4
0.6
3 0.3
0.4
2 0.2

1 0.1 0.2

0 0 0
0 2 2
0.5 1
x
1.5 1 1.5 2.5
x 3 0 0.2 0.4 0.6
x
0.8 1

Figure 15.2 Graphs of the three solutions defined above, for d = 1

(b) If ab = d2 the family of lines is

x Cy x y
+ 2 1 = 0 giving fC (x, y, C) = 2 + 2 .
f (x, y, C) =
C d C d
x
r
Hence fC = 0 gives C = d , so the envelope is given by
y

x 1 x 1 2
r
p + y = 1, that is xy = d , a hyperbola.
d x/y d y 4

Solution for Exercise 2.48


(a) This is a Bernoulli equation with P = Q = 1 and n = 1/2, so we set z = y 1/2 to
obtain
dz 1 1 d  x/2 1 x/2
+ z= that is ze = e .
dx 2 2 dx 2
 2
The general solution is z = aex/2 + 1 and hence y = 1 + ( A 1)e(1x)/2 .
15.2. SOLUTIONS FOR CHAPTER 2 445

(b) This is a Bernoulli equation with P = Q = 1 and n = 1/5, so we set z = y 4/5 to


obtain
dz 4 4 d  4x/5 4 4x/5
z= that is ze = e .
dx 5 5 dx 5
   5/4
4x/5 4/5 4
The general solution is z = ae 1 and hence y = (1 + A ) exp (x 1) 1 .
5
(c) This is a Bernoulli equation with P = Q = x and n = 2, so we set z = 1/y to
obtain
dz d  x2 /2  2
+ xz = x that is ze = xex /2 .
dx dx
2
/2
The general solution is z = aex 1 and hence
1
y= .
3 exp(x2 /2) 1

(d) Set z = y x to give z = (1 + sin z) which is separable and hence


Z z
1 h  z iz
dz = x that is tan = x
0 1 + sin z 4 2 0
which gives

z= 2 tan1 (1 + x) that is y =x+ 2 tan1 (1 + x).
2 2

(e) This is a homogeneous equation, so we set y = vx to obtain


dv dv dx
p Z Z p
x = 1 + v 2 and hence = giving ln( 1 + v 2 v) = ln(ax).
dx 1 + v2 x
p
Thus the general solution
p is x2 + y 2 y = ax2 . The only solution that satisfies the
initial condition is x + y 2 y = ax2 , and rearranging this gives
2

1 1 x2
y= ax2 and the initial condition gives y = A .
2a 2 4A

(f) Writing the equation in the form


dy 2x + 4y 2
2 =
dx 2x + 4y + 3
suggests defining z = 2x + 4y, which gives the separable equation
1 dz 2z + 1 1 5
= giving z + ln(1 + 2z) = 2x + a.
2 dx z+3 2 4
Replacing z by 2x + 4y and using the initial condition gives
 
5 1 + 4x + 8y
2y + ln = x 1.
4 5
446 CHAPTER 15. SOLUTIONS TO EXERCISES

(g) Write the equation in the form


dy xy v dx
Z Z
= giving dv 2 = , where y = xv.
dx y v +v1 x
With u = v + 1/2 the integral on the left-hand side becomes
!
u 12 1 51 5+1
Z Z
du 2 5 = + ,
u 4 2 5 u 5/2 u + 5/2

hence the solution becomes


( ! !)
1 5 5
( 5 1) ln u + ( 5 + 1) ln u + = a ln x.
2 5 2 2

But u = v + 1/2 = y/x + 1/2, so this expression simplifies to


! !
51 5+1
( 5 1) ln y x + ( 5 + 1) ln y + x = a = constant.
2 2

But y(1) = 1, so
! ! !
3 5 3 5 7 3 5
a = ( 5 1) ln + ( 5 + 1) ln + = ln + .
2 2 2 2 2 2

The solution can also be written in the form


!51 !5+1
51 5+1 7+3 5
y x y+ x = .
2 2 2

(h) This is a linear, first-order equation which can be written in the form
d
exp( cosh x) (y exp(cosh x)) = sinh 2x,
dx
which can be integrated,
Z
y exp(cosh x) = a+2 dx sinh x cosh x exp(cosh x)
Z
= a+2 dc ce2 = a + 2(c 1)ec , c = cosh x,

y = a exp( cosh x) + 2(cosh x 1).

(i) This is a linear, first-order equation which can be written in the form
1 d 1
yx2 = x2 x2 y = a + x5 .

2
hence
x dx 5
 
1 1
The initial condition gives a = 1/5, and y = x3 2 .
5 x
15.2. SOLUTIONS FOR CHAPTER 2 447

(j) This is a Bernoulli equation with P = tan x, Q = tan3 x and n = 2, so if z = 1/y 2 ,

dz d  z  sin3 x
cos x +2z sin x = 2 cos x tan3 x which can be written in the form = 2
dx dx cos2 x cos5 x
which has the general solution

z sin3 x 1 c2
Z Z
= a2 dx =a=2 dc , c = cos x
cos2 x cos5 x c5
1 1
= a 4 + 2.
2c c

If y(0) = 2, then z(0) = 1/2 and hence a = 0, so that

1 2 cos x
z =1 that is y = , 0x< .
2 cos2 x cos 2x 4

(k) This is a Bernoulli equation with P = 1/x, Q = 1/x3 and n = 2, so if z = 1/y,

dz z 1 d 1
+ = 3 which can be written in the form (xz) = 2
dx x x dx x
which has the general solution

a 1 x2
z= + 2 and hence y = .
x x 1 + ax

Solution for Exercise 2.49


(a) With p = y , p /p2 = 2x so 1/p = x2 1 giving
 
dy 1 1 1+x
= that is y = ln .
dx 1 x2 2 1x

(b) For the homogeneous equation, set y = xa , so (a 1)2 = 0. This gives one solution
y = x. Now set y = xv(x) to obtain xv + v = 0, so the second solution is y = x ln x,
and the general solution of the homogeneous equation is y = x(A + B ln x).
For the inhomogeneous equation, set x = et to write it in the form y 2y + y = te3t ,
which suggests y = (at + b)e3t . Substituting this into the equation gives

1 1
4ate3t + 4(a + b)e3t = te3t hence a = , b= ,
4 4
1
so the general solution is y = x(A + B ln x) x3 (1 ln x).
4
(c) Write the equation in the form xy (x) = yy (x), and set x = et to express this as
y (t) = (1 + y)y (t). Now put p = y (t) and express p in terms of y, so p (t) = pp (y)
and the equation becomes pp (t) = (1 + y)p.
448 CHAPTER 15. SOLUTIONS TO EXERCISES

One solution of this is p = 0, giving y = constant. Assuming p 6= 0, integration gives


p = 21 a + 21 (1 + y)2 , that is

dy
Z
b+t=2 .
a + (1 + y)2

The form of this integral depends upon the sign of a. For a = 2 > 0, we obtain
   
1 1 1 1+y 1 1
(b + t) = tan hence y = 1 + tan b + ln x .
2 2 2

For a = 2 < 0, we obtain


   
1 1 1+y 1 1
(b + t) = tanh1 hence y = 1 tanh b + ln x .
2 2 2

(d) This is a first-order equation in p = y (x)


dp p d p
= 3x that is x = 3x
dx x dx x
with general solution p = ax + 3x2 and hence y = b + ax2 /2 + x3 .
(e) With p =Z y this equation is xp = p2 , having the general solution p = 1/(a ln x),
1
so y = b + dx . This integral cannot be expressed in terms of elementary
a ln x
functions.
dy 2
(f) If p = y (x) then p /p2 = (x + a) and hence = . If y (0) =
dx b + (x + a)2
2/(b + a2 ) = B then b = 2/B a2 > 0. Setting 2 = b = 2/B a2 and integrating, we
obtain
Z x     
1 2 1 a+x 1 a
yA=2 dx 2 hence y = A + tan tan .
0 + (x + a)2

(g) If p = y (x) the equation becomes (y a)p (x) + p2 = 0. Now consider p as a


function of y, so p (x) = pp (y) and (y a)p (y) = p, which has the general solution
p = A/(y a). A p further integration gives (y a)2 = (2(Ax + B), which gives the two
solutions y = a 2(Ax + B).

Solution for Exercise 2.50


(a) If y = 1 x, then y = 1 and y = 0, so the equation is satisfied. Now set
y = (1 x)v(x) and the equation reduces to xv + v = 0, and this has the general
solution v = A + B ln x. Thus the general solution of the original equation is y =
(A + B ln x)(1 x).
(b) If y = (cos x)/x, then
sin x cos x cos x 2 sin x 2 cos x
y = 2 , y = + +
x x x x2 x3
15.2. SOLUTIONS FOR CHAPTER 2 449

and the equation becomes


   
2 sin x 2 cos x 2 sin x 2 cos x
cos x + + + , + cos x = 0.
x x2 x x2

Now set y = v(cos x)/x and the equation becomes


    
cos x sin x cos x cos x 2 sin x 2 cos x
x v 2v + 2 +v + +
x x x x x2 x3
  
cos x sin x cos x
+2 v v + 2 + v cos x = 0,
x x x

which redues to v cos x 2v sin x = 0. This equation integrates directly to v =


A + B tan x, and hence the general solution is y = (A cos x + B sin x)/x).

(c) Differentiation gives

v (x) = (cos x sin x) ex and v (x) = 2ex sin x.

Substituting these into the equation shows that v = ex cos x is a particular solution.
Now put y = f (x)v(x), to give

y = (f cos x + (cos x sin x)f ) ex ,


y = (f cos x + 2(cos x sin x)f 2f sin x) ex ,

and the equation becomes f + 2f = 0. This has the general solution f= c1 + c2 e2x ,
hence the general solution of the original equation is y = c1 ex + c2 ex cos x.

Solution for Exercise 2.51


2
4ac b2

dy b
(a) Write the equation in the form =c y+ + so with z = y + b/(2c)
dx 2c 4c
4ac b2
the equation becomes z = cz 2 + , and the nature of the solution depends upon
4c
2
the sign of 4ac b .

If 4ac b2 > 0 we set = 4ac b2 /2 > 0, so the equation becomes z = (c2 z 2 + 2 )/c
which is separable and gives
dz x b
Z
2 2 2
= + A and hence y = + tan(x + B).
c z + c 2c c

If 4ac b2 < 0 we set = b2 4ac/2 > 0, so the equation becomes z = (c2 z 2 2 )/c
which is separable and gives
dz x b
Z
= + A which hence y = tanh(x + B).
c2 z 2 2 c 2c c

If 4ac b2 = 0 the equation becomes z = cz 2 , which integrates directly to give


y = 1/(A cx) b/(2c).
450 CHAPTER 15. SOLUTIONS TO EXERCISES

(b) With these values of a, b and c the values of or , and the solutions are as shown
in the table.
a b c b2 4ac general solution
i) 2 3 1 1 3/2 tanh(x/2 + )/2
ii) 9 0 4 122 3 tanh(6x + )/2
iii) 1 2 1 0 1/( x) + 1
iv) 1 4 5 22 2/5 + tan(x + )/5

Solution for Exercise 2.52


(a) Differentiating the relation y = vy gives v y +y v+a1 vy +a0 y = 0, hence the quoted
result. The two coupled, first-order equations are obtained simply by rearranging this
equation.

(b) In this example a1 = 0 and a0 = 2 , so v + v 2 + 2 = 0. The general solution for y


is y = A cos(x + c), so v = tan(x + c) is the general solution of the associated
Riccati equation.

Solution for Exercise 2.53


(a) Differentiation gives

y x y y
z = 2 = (c + dz) (ax + by) 2 = c + (d a)z bz 2 .
x x x

(b) Put x = Aet and y = Bet in the equations for x and y, to obtain

(a )A + bB = 0 and cA + (d )B = 0.

These equation have solutions only if satisfies,



a b p

c = 0 that is 2 = 21,2 = a + d (a d)2 + 4bc.
d

The general solution for x(t) is therefore x = A1 e1 t + A2 e2 t , for some constants A1


and A2 . This gives
1 1
(1 a)A1 e1 t + (2 a)A2 e2 t .

y= (x ax) =
b b
Thus
(1 a)A1 e1 t + (2 a)A2 e2 t
z= .
A1 e1 t + A2 e2 t
But
da 1 da 1 p
1 a = 1 = + , 1 a = 2 = where = (a d)2 + 4bc.
2 2 2 2
1 e1 t + C2 e2 t
Thus, if we put C = A2 /A1 we obtain the general solution z = .
b (e1 t + Ce2 t )
15.2. SOLUTIONS FOR CHAPTER 2 451

Solution for Exercise 2.54


(a) If z = yxa the equation becomes
y xa+1 + ayxa = ayxa bx2a y 2 + cxn , that is, y = bxa1 y 2 + cxna1 .
dy dy a1
If = xa , = ax , and this equation simplifies to
dx d
dy b c
= y 2 + (n2a)/a .
d a a
If n = 2a, y () = c/a by 2 /a: depending upon the relative signs of c/a and b/a this
can be integrated in terms of tan or tanh functions.
(b) With z = a/b + xn /u the original equation becomes
 n1 2
xn u a xn a xn
   
nx
x 2 =a + b + + cxn
u u b u b u
which simplifies to xu = (n + a)u cu2 + bxn , which is the original equation with
(a, b, c) replaced by (n + a, c, b). It follows that if n = 2(n + a), the original equation
has solutions that can be expressed in terms of simple functions.
Repeating this process one more time n + a is replaced by 2n + a, and so on: hence for
all n satisfying n = 2(ns + a), s = 0, 1, 2, , the equation has solutions that can be
expressed in terms of simple functions.

Solution for Exercise 2.55


nxn xn+1 axn bx2n
Putting z = xn /u gives u = + cxn which rearranges to the
u u2 u u2
quoted equation.
Using the result derived in exercise 2.54, it follows that z(x) can be expressed in
terms of simple functions if 2(n a) = n.
By defining the new variable v, as in exercise 2.54, namely u = (n a)/c + xn /v, we
see that the coefficient of v becomes 2na, so we can deduce that z(x) can be expressed
in terms of simple functions if 2(2n a) = n. Repeating this analysis shows that if
n = 2(ns a), s = 1, 2, , then z(x) can be expressed in terms of simple functions.

Solution for Exercise 2.56


In this solution we assume that the infinite series is uniformly convergent, so can be
differentiated term by term: this assumption is proved in Whittaker and Watson (1965,
page 195).
First differentiate the recurrence relation,
Z x

yn (x) = dt G(t)yn1 (t) and yn (x) = G(x)yn1 (x),
a

so that yn (a) = yn (a)


= 0, n 1, hence y(a) = y0 (a) = A and y (a) = y0 (a) = A . The
second derivative of y is

X
X
y (x) = yk (x) = G(x) yk1 (x) = G(x)y,
k=1 k=1

thus y + G(x)y = 0, y(a) = A, y (a) = A .


452 CHAPTER 15. SOLUTIONS TO EXERCISES

Solution for Exercise 2.57


(a) A solution of the equation y = y, y(a) = 0, is y(x) = 0 and by the uniqueness
theorem this is the only solution. Hence if E(x) = 0 for any x it is zero for all x, so
cannot satisfy the condition E(0) = 1.

(b) Differentiation gives W (x) = E (x)/E(x)2 = W (x). Putting y = x, so


W (y) = W (x), gives W (y) = W (y) with W (0) = 1. This is the same as the
equation that defines E(y) and hence W (y) = E(y) = E(x): hence E(x)E(x) = 1.

(c) Differentiation gives Z (x) = E (x + y) = E(x + y) = Z(x) and the initial condition
is Z(0) = E(y). But the equation for Z(x) is linear, so the the solution of this equation
with the initial condition Z(0) = A, for any constant A, is Z(x) = AE(x). Now put
A = E(y) to obtain E(x + y) = E(x)E(y).

(d) Differentiating the equation L(y) = x with respect to y gives

dx 1
L (y) = = .
dy dy/dx

But dy/dx = E(x) = y and hence L (y) = 1/y.


If E(xk ) = yk , k = 1, 2, then E(x1 + x2 ) = y1 y2 , so x1 + x2 = L(y1 y2 ). But xk = L(yk ),
hence the result.
Since E(0) = 1, L(1) = 0, putting y1 = y and y2 = 1/y gives 0 = L(y) + L(1/y).

(e) Differentiate the original equation n 1 times to give E (n) (x) = E (n1) (x), so
E (n) (0) = 1 for all n. This gives the Taylor series for E(x).
Differentiating the equation for L(y) n times gives L(n+1) (y) = (1)n y n1 , so L(n+1) (1) = (1)n1
which gives the Taylor series about y = 1. The relation
   
1+z 1
L = L(1 + z) + L = L(1 + z) L(1 z)
1z 1z

gives the required Taylor series.

Solution for Exercise 2.58


(a) Since C = S and S = C, C = C and S = S. Thus if we define z(x) =
C(z) + S(x), then z(0) = and z (0) = and z + z = 0.

(b) From the equations, CC + SS = 0 and hence C 2 + S 2 = constant. The initial


conditions give C 2 + S 2 = 1.

(c) We have

f (x) = C (x + a) = S(x + a) = g (x) and f (x) = S (x + a) = C(x + a) = f (x)

so that
f
   
f
=A with f (0) = C(a), g(0) = S(a).
g g
15.2. SOLUTIONS FOR CHAPTER 2 453

The functions C(x + a) and S(x + a) satisfy the same linear equations as C(x) and
S(x), but with different initial conditions. Thus C(x + a) and S(x + a) must be linear
combinations of C(x) and S(x):
C(x + a) = AC(x) + BS(x), S(x + a) = CS(x) + DC(x).
Putting x = 0 gives C(a) = A and S(a) = D. Differentiation gives
C (x + a) = S(x + a) = AC (x) + BS (x)
= AS(x) + BC(x).
Putting x = 0 gives S(a) = B and similarly, by considering S (x + a) we obtain
B = S(a), and hence
C(x + a) = C(x)C(a) S(x)S(a) and S(x + a) = S(x)C(a) + C(x)S(a).

(d) Represent the solution as a point in the Cartesian space with coordinates (C(x), S(x)).
At x = 0 the coordinates are (1, 0) and as x increases the point traces out the cir-
cle C(x)2 + S(x)2 = 1. From the initial point the point moves anticlockwise, be-
cause C (0) = 0 and S (0) = 1. At no value of x is C = S = 0: if this were
the case a solution would be C = constant and S = constant, which would contradict
the uniqueness theorem. Thus as x increases the point moves continuously, anticlock-
wise round the circle. At some value of x = X > 0 it must reach the initial point
(C(X), S(X)) = (1, 0). Because the equations are autonomous we may use this as an
initial condition to see that (C(2X), S(2X)) = (1, 0), and hence using induction we see
that (C(nX), S(nX)) = (1, 0), n = 0, 1, 2, .
Alternatively we can use the result found in part (c) with x = a = X, to obtain
C(2X) = C(X)2 S(X)2 = 1 and S(2X) = 2S(X)C(X) = 0, and use induction to
prove the general result.
It follows, using the result found in part (c), that C(x) and S(x) are periodic functions
with period X, C(x + X) = C(x) and S(x + X) = S(x) for all x.
(e) Consider the vales of x for which the points on the circle (0, 1), (1, 0) and (0, 1)
are first reached. If these are X1 , X2 and X3 we have
Z X1 Z 0 Z 1 Z 1
dC dC dC p
X1 = dx =
= = , (C = 1 C 2 < 0).
0 1 C 0 S 0 1 C2
For the second point
Z X2 1 0 1
dC dC dz
Z Z Z
X2 = X1 + dx = X1 + = X1 + = X1 + = 2X1 ,
X1 0 C 1 1 C2 0 1 z2

where C = 1 C 2 < 0. For the third point
Z X3 Z 1 Z 0
dC dC p
X3 = X2 + dx = X3 +
= X2 + = 3X1 , (C = 1 C 2 < 0).
X2 0 C 1 1 C2
Finally, for the last quarter of the circle
Z XX 1 1
dC dC
Z Z
X = X3 + dx = X3 + = X3 + = 4X1 ,
X3 0 C 0 1 C2
which proves the required result.
454 CHAPTER 15. SOLUTIONS TO EXERCISES

(f) Multiplication gives


 
1 0
A2 = = I, A3 = AA2 = A, and A4 = I,
0 1
and hence A2n = (1)n I and A2n+1 = (1)n A. Now differentiate the equations n 1
times  (n)     (n)   
C n C C (0) n 1
=A , =A .
S (n) S S (n) (0) 0
Thus C (2n+1) (0) = S (2n) (0) = 0 and C (2n) (0) = S (2n+1) (0) = (1)n and the Taylor
series are

X C (n) (0) n X (1)m 2m
C(x) = x = x ,
n=0
n! m=0
(2m)!

X S (n) (0) n X (1)m 2m+1
S(x) = x = x .
n=0
n! m=0
(2m + 1)!

Solution for Exercise 2.59


The equation is (1 x2 )y 2xy + y = 0. Put y = uv to give
(1 x2 )vu + 2(1 x2 )v 2xv u + v + (1 x2 )v 2xv u = 0
 

so if v /v = x/(1 x2 ), that is v = 1/ 1 x2 the equation becomes
d2 u
 
1 u
2
+ + 2
= 0.
dx 1x 1 x2

Solution for Exercise 2.60


The chain rule gives
dy dy dt dy p d2 y q (x) dy d2 y
= = q(x) and 2
= + q(x) 2 .
dx dt dx dt dx 2 q dt dt
d2 y q (x) dy dy
Hence the equation becomes q + + p1 q + qy = 0, which is the required
dt2 2 q dt dt
result

Solution for Exercise 2.61


Because the second derivative of each function is a linear combination of the function
and its derivative, the third column is a linear combination of the first two and the
determinant is zero.

Solution for Exercise 2.62


(a) The Wronskian is W = sinh x cos x cosh x sin x and then
f g g f cosh x sin x + sinh x cos x
p0 = =
W (f, g; x) cosh x sin x sinh x cos x

f g gf 2 sinh x sin x
p1 = =
W (f, g; x) sinh x cos x cosh x sin x
15.2. SOLUTIONS FOR CHAPTER 2 455

so that the equation is

(sinh x cos x cosh x sin x) y + 2 sinh x sin x y (cosh x sin x + sinh x cos x )y = 0

(b) The Wronskian is W = 4/ sin 2x and

cos 2x
p0 = 4/ sin2 2x, p1 = 2 giving the equation sin2 2x y + sin 4x y 4y = 0.
sin 2x

Solution for Exercise 2.63


If g = 1/f the Wronkian is W = 2f /f = 2u. Also
     3
1 1 f
p0 W = f

f =2 , hence p0 = u2 ,
f f f

and    
1 1 f u
p1 W = f f =2 , hence p1 = .
f f f u
u f
Thus the differential equation with solutions f and 1/f is y y u2 y = 0, u= .
u f

Solution for Exercise 2.64


There are three determinants formed by differentiating W (x); those obtained by differ-
entiating the first and second row are zero, so

f g h f g h
dW
= f g h = f g h .
dx f g h p2 f p1 f p0 f p2 g p1 g p0 g p2 h p1 h p0 h

Multiply the first and second rows by p0 and p1 , respectively, and add to the third row
to obtain
f g h
dW
= f g h = p2 (x)W (x).
dx
p2 f p2 g p2 h

Hence  Z x 
W (x) = W (a) exp dx p2 (x) .
a

Solution for Exercise 2.65


(a) If v = f /g then v = (f g f g )/g 2 and

f f g 2f g 2 2f g
v = 2 + , but f = qf, g = qg,
g g g3 g2
2g 2g
= (f g f g) = v.
g3 g
456 CHAPTER 15. SOLUTIONS TO EXERCISES

Hence
2g g 2
 
g
v = v 2 2 v ,
g g g
 2  2
v g v 3 v
= 6 2q, hence = 2q.
v g v 2 v

(b) If f and g are linearly independent, so are af + bg and cf + dg, so

af + bg av + b
v= = and S(v) = 2q = S(v).
cf + dg cv + d

Solution for Exercise 2.66


Part (c) of this solution contains a simpler geometric proof of the expression for the
radius of curvature, , found in part (b).
(a) The equation of the normal through ( + , + ), where = f () and + =
f ( + ), is

x
y =
m( + )
m ()
 
1 2
= (x ) 1 + O( )
m() m()
m
   
1 2
= x 1 + (x ) + O( ) .
m() m

But = f () + O( 2 ) and using the equation of the normal through the adjacent
point (, ) we obtain an equation for the x-coordinate of the point of intersection of
these two normals,
m
x = (1 + mf ), m() = f ().
m
The y-coordinate is given by
1 1
1 f ()2 .

y = (x ) =
m () m ()

(b) The distance of this point of intersection from (, ) is given by


1  
2 = (x )2 + (y )2 = 1 + f 2
+ m 2
1 + f 2
m ()2
(1 f ()2 )3 1 f ()
= hence = .
f ()2 (1 + f ()2 )3/2

(c) A simpler alternative derivation


The simplest expression for the radius of curvature is = ds/d, where s is the length
along a curve and the angle between the tangent and the x-axis; this leads to an easier
method of deriving the given relation for .
15.2. SOLUTIONS FOR CHAPTER 2 457

The formula = ds/d is a consequence of a simple


geometric construction, shown in figure 15.3. The nor-
y
mals to the curve ABCD at B, a distance s along the P
curve, and at C, a distance s + s along the curve, D
intersect at P . The angles the tangents at B and C
make with the x-axis are and + , respectively,
and since BC approximates a circular arc elementary
considerations show that the angle BP C is . The C
distances BP and CP approximate the radius of cur- s
A B
vature at B, and as s 0, BP CP . Further
the length of the arc BC is and hence + x

s ds
(s) = lim = .
s0 d

Figure 15.3

Treating s as the independent variable gives


1 d d dx d f (x)
= = , but = tan1 f (x) so = .
ds dx ds dx 1 + f (x)2
ds
Also, since s2 = x2 + y 2 , dividing by x and taking the limit x 0 gives =
p dx
1 + f (x)2 , and hence
1 f (x)
= .
(1 + f (x)2 )3/2

Solution for Exercise 2.67


The gradient of the tangent is b/a, hence b = pa. But also p = y/(x a), so
a = x y/p and hence
 
2 y
2 = ab = pa = p x , that is 2p = (px y)2 .
p
The equation defining the curve C is therefore f (x, y, p) = 0, where
f (x, y, p) = 2p + (px y)2 and fp (x, y, p) = 2 + 2x(px y).
The singular solution is obtained by eliminating p from these equations: the second
gives px y = /x, and hence
2 2
 
y
0 = 2 2 + 2 , that is xy = .
x x x 2
Now find the general solution of the equation f = 0: expand to obtain
p2 x2 2p(xy ) + y 2 = 0
which has the solutions
p
dy xy 2 2xy
p= = .
dx x2
458 CHAPTER 15. SOLUTIONS TO EXERCISES
p
Now put v = xy to give xv = 2v 2 2v. One solution of this is v = /2,
which is the singular solution found above.
Now assume > 0, and define z 2 = 2v to give xz = z and hence
1
y = A A2 x, for some constant A, which is the equation of a family of straight
2
lines. The envelope of these lines is the singular solution xy = /2.

Solution for Exercise 2.68


By definition N = N , N (0) = N0 , for some positive constant . This equation
has thegeneral  solution N (t) = N0 exp(t). For C14 , 12 = exp(5600), so N (t) =
t ln 2
N0 exp , where t is measured in years.
5600

Solution for Exercise 2.69


The rate of loss of mass is dm/dt = Ar2 , for some positive constant A. But m = r3 ,
so dm/dt = c for some positive constant c. Hence r = 10 ct. At t = 1, r = 5, so
r = 5(2 t) and the moth ball vanishes after 2 months.

Solution for Exercise 2.70


Let W (t) be the total weight of salt (in kg) in the tank, so the concentration is W/1000,
and
 
W dW W
W (t + t) = W (t) + t 1 that is =1 , W (0) = 0.
1000 dt 1000

The solution of this equation is given by


W  
dW W t
Z
= t, that is = 1 exp .
0 1 W/1000 1000 1000

Thus W = 50 when et/1000 = 1/2, that is t = 1000 ln 2 693.

Solution for Exercise 2.71


For a sphere of radius R the volume of a cap of depth h is V = 13 h2 (3R h); if h = R,
the cap becomes a hemisphere and V = 2R3 /3.
2
The loss
of water in a time t is A v t, where A = a is the area of the hole
and v = 2gh is the speed of the water, of depth h. Thus, if V is the volume of the
water Torricellis law gives
dV p
= a2 2gh.
dt
1 dh p
But V = 13 h2 (3R h), so this gives (6Rh 3h2 = a2 2gh which integrates
3 dt
directly:
0
14R5/2
Z
dh (6Rh1/2 3h3/2 ) = 3a2
p
2gt, that is t = .
R 15 2g
15.2. SOLUTIONS FOR CHAPTER 2 459

Solution for Exercise 2.72


Let the radius of the bowl at height
R z z from the bottom be r(z), so the volume it contains
up to the height z is V (z) = 0 dz r(z)2 , and the rate of change of the volume of water
dV
= a2 2gz. Hence the rate of change of the water level is given by
p
is
dt
dz
r(z)2 = a2 2gz.
p
dt
Thus z = constant if r z 1/4 . On Cartesian coordinates this gives z x4 .

Solution for Exercise 2.73


Since T = T ( + ) T () = T we have dT /d = T , with solution T = T0 e .

Solution for Exercise 2.74


Energy is conserved. If the hanging end of the chain is a distance z below the table
top, the whole chain is moving with speed z, and its kinetic energy is 21 M z 2 , M being
1
the mass of the chain. The potential energy of the hanging portion is gz 2 , where
2
is the linear density, so L = M . Thus energy conservation gives
1 g 2
M z 2 z 2 = l02 , that is z 2 = (z l02 ).
2 g g M
Hence the time taken to fall off the table is
Z L p !
L2 l02
r r
dz g g L+
p = t that is t = ln .
l0
2
z l02 L L l0

Solution for Exercise 2.75


If the intensity at a depth d is I(d), we have I(d + d) I(d) kI(d)d, where k is a
1
positive constant. Hence I = kI and I(d) = I0 ekd . But I0 = I0 ek/10 so
2
 
d ln 2 I0
I(d) = I0 exp and I(d) = when d = 40 ft.
10 16

Solution for Exercise 2.76


If h1 and h2 are the heights of the vertical parts of the liquid on the left and right-hand
sides and Lc the length of the curved parts of the tube, the potential energy, to within
an additive constant, is
1
V = Ag(h21 + h22 ).
2
But L = Lc + h1 + h2 and h = h2 h1 , so h21 + h22 = 12 h2 + constant, and V = 41 Agh2 .
The liquid is incompressible, so it all moves with the same speed, h1 = 12 h, so the
kinetic energy is T = 81 ALh2 . Thus the total energy is

1 1
E= ALh2 + Agh2 .
8 4
460 CHAPTER 15. SOLUTIONS TO EXERCISES

Initially h(0) = h0 and h(0) = 0, so this becomes


2g 2
h2 = (h h2 ).
L 0
The simplest way of solving this equation is to set h = h0 cos t, for some
p angular
frequency > 0, to obtain 2 = 2g/L, hence the period is T = 2/ = 2L/g.
Alternatively, this differential equation is separable, so
h
dh 2g
Z
p = t,
h0 h20 h2 L

which gives the same solution.

Solution for Exercise 2.77

A O x P B
Consider a hole AB as shown in the figure. The force on a
r
particle P in the direction BA is F = kr cos , and if OP = x,
so cos = x/r, then F = kx, k > 0.
Newtons equation of motion is mx = kx and hence for a
particle initially at B,
with x(0) = X and x(0)= 0 the so-
lution is x = X cos( k t) and the period is 2/ k, which is
independent of X, that is the position of the hole.
Figure 15.4
15.3. SOLUTIONS FOR CHAPTER 3 461

15.3 Solutions for chapter 3


Solution for Exercise 3.1
To find the stationary function we need to compute the difference S = S[y+h]S[y] to
O() but, because exercise 3.3 requires the second-order term, we evaluate the difference
to O(2 ). The difference is
Z 1 p p 
S = dx 1 + y (x) + h (x) 1 + y (x) ,
0

where h(0) = h(1) = 0. But


1/2
h (x)

p p
1 + y (x) + h (x) = 1 + y (x) 1 + ,
1 + y (x)
2 !
h (x) 2 h (x)

p
= 1 + y (x) 1 + + ,
2(1 + y (x)) 8 1 + y (x)

where we have used the binomial expansion (1 + z)1/2 = 1 + 21 z 18 z 2 + , which is


equivalent to using the Taylor series for (1 + z)1/2 . Hence
1 1
2
h (x) h (x)2
Z Z
S = dx p dx + O(3 ).
2 0 1 + y (x) 8 0 (1 + y (x))3/2

The functional is stationary if the first-order term is zero for all h(x), otherwise S
would change sign with . Using the result quoted inpthe text (after equation 3.5)
and proved in exercise 4.4 (page 128) this gives 1 + y (x) =constant, that is
y (x) =constant and y(x) = x + . The boundary conditions then give y = Bx for
the stationary path. With this value for y(x), the integrand is real if B > 1 and has
the value S = 1 + B.

Solution for Exercise 3.2


(a) The required expansion is given by first writing the square root as
1/2
2 2

p
2 2 2
p
2
2
1 + + 2 + = 1 + 1 + + .
1 + 2 1 + 2

Now use the binomial expansion (1 + z)1/2 = 1 + 21 z 18 z 2 + to give


r 2
2 2 2 2 2 2
  
2 12 1 2
1+ 2
+ = 1+ + + + ,
1+ 1 + 2 1 + 2
2 1 + 2 8 1 + 2 1 + 2
2 2
= 1+ 2
+ + O(3 ).
1+ 2(1 + 2 )2

Hence
p p 2 2
1 + ( + )2 = 1 + 2 + + + O(3 ).
1 + 2 2(1 + 2 )3/2
462 CHAPTER 15. SOLUTIONS TO EXERCISES

(b) With = y (x) and = h (x) we see, using the argument described in the text,
that the term O() in the expansion of S[y + h] S[y] is zero if y (x) =constant, hence
the straight line defined by equation 3.6 makes the functional stationary. With this
choice of y(x), = m and the second term in the above expansion gives the result
quoted. The second-order term is positive for 6= 0 and all h(x), so the functional has
a minimum along this line.

Solution for Exercise 3.3


The expanson to second-order in is derived in the solution to exercise 3.1. On the
stationary path, y = Bx, the first-order term is, by definition, zero, so we have
Z 1
2
S = dx h (x)2 < 0, B > 1.
8(1 + B)3/2 0
Because this term is always negative, for sufficiently small || we have S[ys +h] < S[ys ],
where ys (x) = Bx is the stationary path, which is therefore a local maximum.

Solution for Exercise 3.4


If a1 = b1 = 1, a2 = z and b2 = z + u the three parts of the Cauchy-Schwarz inequality,
page 41, are
2
X 2
X 2
X
a2k = 1 + z 2 , b2k = 1 + (z + u)2 , ak bk = 1 + z 2 + zu,
k=1 k=1 k=1

and the first resultfollows. There is equality only if a = b, that is u = 0. Divide the
first inequality by 1 + z 2 to derive the second result.

Solution for Exercise 3.5


(a) If F (y ) = (1 + y 2 )1/4 then dF/dy = y /[2(1 + y 2 )3/4 ].
(b) If F (y ) = sin y then dF/dy = cos y .
d z
(c) Since dz (e ) = ez we have dF/dy = F .

Solution for Exercise 3.6


F
In this example equation 3.12 becomes = C = constant, and does not given an
y

equation for y (x). Further, if C and D are constants the the functional becomes
Z b
S[y] = dx (Cy (x) + D) = C [y(b) y(a)] + D(b a).
a

This depends only upon C, D and the boundaries a and b: the value of the functional is
therefore independent of the chosen path, and hence this functional has no stationary
paths.
Alternatively, consider the difference
Z b h i
S = S[y + h] S[y] = dx C(y + h ) + D (Cy + D)
a
Z b h i
= C dx h (x) = C h(b) h(a) .
a
15.3. SOLUTIONS FOR CHAPTER 3 463

Since h(a) = h(b) = 0, S = 0 for any y(x). That is, there is no stationary path.
If C and D depend upon x then
Z b
S = dx C(x)h (x).
a

If C is a constant, because h(a) = h(b) = 0, S = 0 for all y(x), as expected from the
previous analysis. Otherwise, for any C(x) we can chose h(x) so that C(x)h (x) > 0
for a < x < b: since S[y] is stationary only if S = O(2 ), this shows that S[y] has no
stationary path.

Solution for Exercise 3.7


In this example F (x, v) = 1 + x + v 2 and equation 3.16 becomes
p
v = c 1 + x + v 2 where v = y (x).

Squaring and rearranging this equation gives


2
c2

dy
= a2 (1 + x), a2 = .
dx 1 c2

Integrating this gives the solution in the form


Z x
2a  
y(x) A = a dx 1 + x = (1 + x)3/2 1 .
0 3

The value of a is obtained from the boundary condition y(1) = B, that is

2 BA (B A)  3/2

a = 3/2 and hence y(x) = A + (1 + x) 1 .
3 2 1 (23/2 1)

Solution forpExercise 3.8


If F (x, y ) = x2 + y 2 , F is independent of y, we have

F F x F y
= 0, =p and
= p
y x x + y 2
2 y x2 + y 2

giving
dF F F F x + y y
= + y + y = p .
dx x y y x2 + y 2
Since F does not depend explicitly upon y, we have

2 F 2F
 
d F

= 2
y +
dx y y xy

and
2F xy 2F 1 y 2 x2
= , = =
xy (x2 + y 2 )3/2 y 2 (x2 + y 2 )1/2 (x2 + y 2 )3/2 (x2 + y 2 )3/2
464 CHAPTER 15. SOLUTIONS TO EXERCISES

which gives
x2 y xy x(xy y ) x3 (y /x)
 
d F
= = = .
dx y (x2 + y 2 )3/2 (x2 + y 2 )3/2 (x2 + y 2 )3/2 (x2 + y 2 )3/2
Also
y (x + y y )y x(xy y )

dF
= p 2 2 3/2
= 2 ,
y dx x2 + y 2 (x + y ) (x + y 2 )3/2
   
d F dF
so, in this case, = .
dx y y dx

Solution for Exercise 3.9


The chain rule applied to a function G(x, y(x), y (x)) has the form
dG G dy G dy G
=
+ + .
dx y dx y dx x
In this example, where G = F/y , this expression becomes

F dy
       
d F F dy F
= + +
dx y y y dx y y dx x y
2 F 2F 2F
= 2
y + y +
y y y xy
which gives the required expression and is the left-hand side of the inequality.
The right-hand side of the inequality is
   
dF F F F
= + y + y
y dx y x y y
2F F 2 F 2 F
=
+ + y + 2y
xy y yy y
which differs from the left-hand side by the term F/y. Thus, only if F is independent
of y are the derivatives equal.

Solution for Exercise 3.10


Subtract the term F/y to obtain the required result.

Solution for Exercise 3.11


F p F yy
(a) Direct differentiation gives = 1 + y 2,
= p . Differentiating the
y y 1 + y 2
second expression gives
2F y yy 2 y
2
= 2 3/2
= .
(1 + y 2 )3/2
p
y 1+y 2 (1 + y )

Using the expression derived in exercise 3.10, namely


2 2
2F
 
d F F F F F
z= = y + y = 0, since = 0,
dx y y y 2 yy y xy
15.3. SOLUTIONS FOR CHAPTER 3 465

we obtain
yy y 2 1/2
z = 2 3/2
+ 2 1/2
1 + y 2 ,
(1 + y ) (1 + y )
1 
2
 2  
2 2 1

yy y 2 1 ,

= yy + 1 + y y 1 + y =
(1 + y 2 )3/2 (1 + y 2 )3/2
hence the equation z = 0 becomes yy 1 y 2 = 0. But
y 2
 
y
 
d y 2 2 d y
= 2 giving yy y = y , if y 6= 0,
dx y y y dx y
and hence      
d F F 1 2 d y
= y 1 .
dx y y (1 + y 2 )3/2 dx y
(b) If the left-hand side is zero we have
   
d y d y
y2 = 1 or y 2 y = 1.
dx y dy y
Now define z = y /y and consider z to be a function of y, so in the following z = dz/dy
note this is possible because x may be considered a function of y so y /y can be
expressed in terms of y. Now put the second equation in the form y 3 z z (y) = 1, which
can be integrated directly to give z 2Z = C 2 y 2 , for some constant C. Hence, since
dy p dy
z = y /y, = (Cy)2 1 giving p = x + D. Finally, set Cy = cosh
dx (Cy)2 1
to give = C(x + D), that is y = (1/C) cosh(Cx + CD), which is the required solution,
if C = A and CD = B.

Solution for Exercise 3.12


The first result follows directly by replacing F [y], in equation 3.21, by FR from equa-
tion 3.18. Putting x = b cos and y = b sin in the integral we obtain,
Z /2
CD = 8 d sin cos3 = 2.
0

Solution for Exercise 3.13


(a) The expressions for y(x), y (x) and D[y] are
y(x) y (x) D[y]
2
x 2x 3
cos x sin x 1.

(b) If a(x) = x, then


1
11
Z
if y(x) = x , 2
K[y] = dx x(x2 + 2x) = , and
0 12
1
2
Z
if y(x) = cos x, K[y] = dx x(cos x + sin x) = 1 .
0 2
466 CHAPTER 15. SOLUTIONS TO EXERCISES

(c) If a(x) = x and b(x) = 1 then


h i1 Z 1
4
2
dx 3x2 = 3

if y(x) = x , L[y] = 2x + and
0 0
h i1 Z 1
if y(x) = cos x, L[y] = x sin 2x + dx (x sin x + cos x) = 1.
2 0 0

(d) In the first case, y(x) = x2 ,

1 1 1  1
1 4 3 1 3 4
Z Z Z
S[x2 ] = dt s2 + st s2 t2 =

ds ds s t + s t
0 0 0 3 4 t=0
1  
1 4 1 3 31
Z
= ds s + s = .
0 3 4 240

In the second case, y(x) = cos x,


Z 1 Z 1
dt s2 + st cos t

S[cos x] = ds cos s
0 0
1 1
s2
 
t 1
Z
= ds cos s sin t + s sin t + 2 cos t
0 0
Z 1
2 4
= 2 ds s cos s = 4 .
0

Solution for Exercise 3.14 p p


The derivative of f (x) is f (x) = x/ x2 + h21 (d x)/ (d x)2 + h22 . Since

AR x RB dx
sin 1 = = p , and sin 2 = = p ,
SR x2 + h21 RO (d x)2 + h22

where the distances are defined in figure 3.10 (page 113), we see that the distance
travelled by the light is stationary when sin 1 = sin 2 , that is 1 = 2 . Further since

h21 h22
f (x) = 2
+ > 0,
(x2 + h1 ) 3/2 ((d x)2 + h22 )3/2

the stationary point is a minimum.

Solution for Exercise 3.15


(a) We need the difference S = S[y + h] S[y] where h(0) = h(1) = 0, otherwise h(x)
is an arbitrary continuous function. Now, using the Binomial expansion

2 2
 
p 3
1 + + = 1 + 1 + + O( ) ,
2(1 + ) 8(1 + )2
15.3. SOLUTIONS FOR CHAPTER 3 467

and so
2 2
 
p
( + ) 1 + + = 1+ 1+ +
2(1 + ) 8(1 + )2

 

+ 1 + 1 + + ,
2(1 + )
(2 + 3) 2 2 (4 + 3)
= 1++ + + .
2 1+ 8(1 + )3/2

Now substitute = y and = h to obtain


Z 1
2 + 3y 2 1 4 + 3y
Z
S = dx h (x) + dx h (x)2 + O(3 ).
0 2 1 + y 8 0 (1 + y )3/2

If y(x) is a stationary path of S then the term O() is zero. Since h(0) = h(1) = 0 it
follows, as in the text, that y (x) =constant
is a possible solution. Since y(0) = 0 and
y(1) = B this gives y(x) = Bx and S[y] = B 1 + B.

Alternatively, using equation 3.12 (page 100), with F (y ) = y 1 + y , we see that
the stationary path is given by F (y ) = constant and hence y = constant, that is
y = mx + c: since y(0) = 0 and y(1) = B this gives y(x) = Bx.
(b) On substituting Bx for y(x) we see that S takes the value,
1
2 (4 + 3B)
Z
S = dx h (x)2 + O(3 ).
8(1 + B)3/2 0

Then, provided B > 1, S is positive and the functional is a minumum on the sta-
tionary path.

Solution for Exercise 3.16


Observe that Z b
S1 [y] = S2 [y] + dx y (x) = S2 [y] + B A.
a
That is the values of the two functionals differ by a constant, independent of the path.
Hence the stationary paths of the two functionals are the same.
Consider the difference S = S2 [y + h] S2 [y] where h(a) = h(b) = 0:
Z b
S = 2 dx xy (x)h (x) + O(2 )
a

so that S = O(2 ) if xy (x) = c, where c is a constant. Integrating this equation gives


y(x) = d + c ln(x/a), where d is another constant. The boundary condition now give

ln(x/a)
A = d and B = d + c ln(b/a) and hence y(x) = A + (B A) .
ln(b/a)
468 CHAPTER 15. SOLUTIONS TO EXERCISES

Solution for Exercise 3.17


(a) Consider the difference S = S[y + h] S[y] where h(a) = h(b) = 0, so we need
the expansion
dF 1 2 2 d2 F
F (y + h ) = F (y ) + h + h + .
dy 2 dy 2
Hence
b b
dF 1 d2 F 2
Z Z
S = dx h (x) + 2 dx h (x) + O(3 ).
a dy 2 a dy 2
2
(b) If dF/dy =constant then S = O( ) so S[y] is stationary. If dF/dy =constant

then, provided F (z) is not a constant or a linear function of z, y (x) is also a constant.
(c) On the stationary path y (x) is a constant and hence d2 F/dy 2 is constant and
1 d2 F b
Z
S = 2 2 dx h (x)2 + O(3 ).
2 dy a

The integral is positive, so S is positive or negative according as d2 F/dy 2 is pos-


itive or negative. That is S[y] is either a minimum (d2 F/dy 2 > 0) or a maximum
(d2 F/dy 2 < 0). If d2 F/dy 2 = 0 the nature of the stationary path can be determined
only by expanding to higher-order in .

Solution for Exercise 3.18


In this example F (z) = (1 + z 2 )1/4 , where we have used the notation of the previous
exercise. Thus
z 2 z2
F (z) = 2 3/4
, F (z) = ,
2(1 + z ) 4(1 + z 2 )7/4
and hence the stationary path is y = Bx and
Z 1
(2 B 2 )2
S[y + h] S[y] = dx h (x)2 + O(3 ).
8(1 + B 2 )7/4 0

Thus if |B| < 2 the difference is positive for all h(x) and , if sufficiently
small, so
the functional is a minimum along the line f (x) = Bx. For |B| > 2 the difference is
negative and the functional is a maximum. If B = 2 the nature of the stationary
path can be determined only by expanding to higher-order in .

Solution for Exercise 3.19


The potential energy, V , of an element of the rope of length sp
centred on a point x
is given by massheight g, that is V = (s)y(x)g: since s = 1 + y 2 x this gives
Rb p Rb p
the total potential energy as E[y] = g a dx y 1 + y 2 and L[y] = a dx 1 + y 2 is
the length of the chain.

Solution for Exercise 3.20


(a) Since, to first-order, x = sin and y = cos , the distance is
 2 !
z
s2 = x2 + y 2 + z 2 = 2 2 + z 2 = 2 2 + .

15.3. SOLUTIONS FOR CHAPTER 3 469

(b) The length along a curve is just the sum of the small elements which in the limit
R p
0 becomes the integral L[z] = 12 d 2 + z ()2 .

(c) The functional L[z] is the same type as that considered in section 3.3.1 hence its
minimum value is given when z() is a linear function of . The boundary conditions
give the result quoted.

Solution for Exercise 3.21


The Cartesian coordinates of a point (, ) on the cone are
 
(x, y, z) = cos , sin ,
tan
and for the adjacent point at ( + , + ), or (x + x, y + y, z + z) in Cartesian
coordinates, we have, to first-order

x = cos sin , y = sin + cos , z = .
tan
The distance between the two adjacent points is therefore
  2
 2 !
1 1
s2 = 1 + 2 + 2 2 = + 2 2 = 2 + 2 .
tan2 2
sin 2
sin

Hence p the distance between the points 1 and 2 along the curve () is L[] =
R 2
1
d 2 + 2 sin2 .

Solution for Exercise 3.22


Let the velocity of the boat relative to the water be (ux , uy ), where c2 = u2x + u2y , and
we assume that ux is positive. The velocity of the boat relative to land is therefore
(ux , v(x) + uy ). If the path taken is y(x) it follows that
dy uy + v dy
= and hence uy = ux v.
dx ux dx
Also, the time of passage is
a
dx
Z
T [y] = .
0 ux
Now we need an expression for ux . Since c2 = u2x + u2y , we have, on using the above
2
expression for uy , (y (x)ux v) = c2 u2x . This rearranges to the quadratic
1 + y 2 u2x 2vy ux c2 v 2 = 0,
 

having the solutions


p
vy (vy )2 + (c2 v 2 )(1 + y 2 )
ux = .
1 + y 2
Because c > v this quadratic has one positive and one negative root. We need the
positive root:
p
vy + (vy )2 + (c2 v 2 )(1 + y 2 ) c2 v 2
ux = 2
= p .
1+y (vy ) + (c v 2 )(1 + y 2 ) vy
2 2
470 CHAPTER 15. SOLUTIONS TO EXERCISES

Hence
a Z a
p p
(vy )2 + (c2 v 2 )(1 + y 2 ) vy (1 + y 2 )c2 v 2 vy
Z
T [y] = dx = dx .
0 c2 v 2 0 c2 v 2

Solution for Exercise 3.23


The kinetic energy of a particle of mass m and velocity v is 12 m|v|2 and its linear
momentum is mv. For an elastic collision energy and momentum are conserved, so

M V 2 + mv 2 = M V 2 + mv 2 Energy conservation
M V mv = M V + mv Linear momentum in the direction of the block motion

From the second equation v = M (V V )/m v, so conservation of energy gives


2
MV 2 = M V 2 + mv 2 m (v M (V V )/m)
M2
= M V 2 + 2M v(V V ) (V V )2 .
m
But V 2 = (V V )2 2V (V V ) + V 2 and hence
 
M 2
M 1+ (V V ) 2M (V + v)(V V ) = 0,
m

with solutions V = V and


2m m
V =V (V + v) V as 0.
M +m M
The solution V = V gives, from the momentum equation, v = v, which is for the
motion of the particle through the block and we discard this solution. The equation for
v is
2M 2M V + (M m)v m
v = (V + v) v = 2V + v as 0.
M +m M +m M
When m/M is zero the solutions correspond to the elastic collision of a massless particle
from a massive body when the relative velocity before and after the collision is the same.
15.4. SOLUTIONS FOR CHAPTER 4 471

15.4 Solutions for chapter 4


Solution for Exercise 4.1
The first result follows directly from equation 4.3 because F is independent of x and y,
y(a) = y0 = A and y(b) = yN +1 = B. The variable yk for each k = 1, 2, , N appears
in only two terms of the sum, so
    
S yk yk1 yk+1 yk
= F +F
yk yk

and hence, since F depends only upon y and not y, the stationary points are given by
the equations,
   
S yk yk1 yk+1 yk
= F F = 0, k = 1, 2, , N.
yk

Thus F ((yk yk1 )/) = c, k = 1, 2, , N + 1, where c is a constant, independent


of k. This is true for all k so yk yk1 =constant and hence the points (xk , yk ) lie on
a straight line.

Solution for Exercise 4.2


R /2
(a) We have S[y + h] = 0 dx (y + h )2 (y + h)2 . Hence
 

/2 /2
d
Z Z
dx (y + h )h (y + h)h dx (y h yh) .
 
S[y+h] = 2 and S[y, h] = 2
d 0 0

Rb
(b) We have S[y + h] = a dx (y + h )2 x3 . Hence

b b
d (y + h ) y h
Z Z
S[y + h] = 2 dx h and S[y, h] = 2 dx .
d a x3 a x3

Rb
dx (y + h )2 + (y + h)2 + 2ex(y + h) . Hence
 
(c) We have S[y + h] = a

b b
d
Z Z
dx (y + h )h + (y + h)h + ex h and S[y, h] = 2 dx [y h + (y + ex ) h] .
 
S[y+h] = 2
d a a

R1 p p
(d) We have S[y + h] = 0 dx x2 + (y + h)2 1 + (y + h )2 . Hence
" #
1
p
d (y + h)h x2 + (y + h)2 (y + h )h
Z p
S[y+h] = dx p 1 + (y + h )2 + p
d 0 x2 + (y + h)2 1 + (y + h )2

and " p #
1
p
y 1 + y 2 x2 + y 2 y
Z
S[y, h] = dx p h+ p h .
0 x2 + y 2 1 + y 2
472 CHAPTER 15. SOLUTIONS TO EXERCISES

Solution for Exercise 4.3


The functional evaluated at y + h is
Z b Z b
S[y + h] = ds dt K(s, t) (y(s) + h(s)) (y(t) + h(t))
a a

so that
b b
d
Z Z  
S[y + h] = ds dt K(s, t) y(s)h(t) + h(s)y(t) + O() .
d a a

Taking the limit 0 and rearranging this integral gives


Z b Z b Z b Z b
S[y, h] = ds dt K(s, t)y(s)h(t) + dt ds K(t , s )h(t )y(s )
a a a a
Z b Z b
= dt h(t) ds [K(s, t) + K(t, s)] y(s)
a a

where, in the second integral we have put t = s and s = t and then changed the
integration order of the first integral to obtain the final result.

Solution for Exercise 4.4


(a) Clearly g(a) = 0: also
Z b
g(b) = C(b a) dt z(t) = 0 by the definition of C.
a

Then, since g (x) = C z(x):


Z b Z b  
dx z(x)g (x) = dx z(x) C z(x) ,
a a
Z b  2 Z b   Z b  2
= dx C z(x) + C dx C z(x) = dx C z(x) .
a a a

Unless z(x) = C, the integrand is almost everywhere positve and hence the integrand
is zero only if z(x) = C.

Solution for Exercise 4.5


In this case F = y 2 y 2 giving Fy = 2y and Fy = 2y, which leads to the Euler-
Lagrange equation y + y = 0. The general solution of this equation is y = A cos x +
B sin x, where A and B are arbitrary constants determined by the boundary conditions.
The boundary condition at x = 0 gives A = 0 that at x = X gives the solution
y(x) = sin x/ sin X, provided sin X 6= 0. If sin X = 0, that is X = n, n = 1, 2, , the
only solution is the trivial function y(x) = 0.

Solution for Exercise 4.6


Since Fy = 2y and Fy = 1 the Euler-Lagrange equation is 2y + 1 = 0, which has the
general solution y = A + Bx x2 /4, for constants A and B. The boundary conditions
give y(0) = A = 0 and y(1) = A + B 1/4 = 1, giving the solution y = x(5 x)/4.
15.4. SOLUTIONS FOR CHAPTER 4 473

The first-integral is c = y Fy F = 2y 2 y 2 y or y 2 + y = c. Re-arranging




this and separating variables gives

dy
Z
= x or 2 c y = A x.
cy

Putting x = 0 gives 2 c = A and hence y = Ax/2 x2 /4; putting x = 1 gives
y(1) = 1 = A/2 1/4, and hence y = x(5 x)/4.

Solution for Exercise 4.7


Using the result of exercise 3.10 (page 103) we see that if G does not depend explicitly
upon x, G/x = 0, and

2 G 2 G 2 G
   
d G G
y = 2
yy + y y.
dx y y y yy y

But, using the chain rule

2 G 2 G 2 G
     
d G G G
y = y y + y y = y + y + 2y y ,
dx y y y y y yy y y

and so the right-hand side of the previous equation becomes


     
d G G G d G dG d G
y y y = y = y G .
dx y y y dx y dx dx y

Integrate the last equation to give y Gy G = c ( a constant). This is a first-order


differential equation: its general solution will depend upon one other arbitrary constant
d, and to find the solution of the original problem we need to express these constants
(c, d), in terms of the constants (A, B) defined in equation 4.12 (page 130); often this is
difficult, because it involves the solutions of nonlinear equations, and frequently there
are real solutions only for some values of A and B.

Solution for Exercise 4.8


(a) If is a constant and y(x) = equation 4.13 becomes G(, 0) = c.

(b) The second-order Euler-Lagrange equation is

2 F 2F 2F F
2
y +
y + = 0.
y yy xy y

If F (x, y, y ) = G(y, y ) the third term is zero and if y = this equation becomes
Gy (, 0) = 0, assuming that Gy y (, 0) and Gyy (, 0) exist.
Let g(y) = G(y, 0) be a function of y. The equation Gy (, 0) = 0 shows that must
be at a stationary point of g(y) whereas the equation G(, 0) = c, found in part (a),
imposes the weaker restriction that c lies in the domain of g(y).
Thus, in general the constant solution y = of the first-integral, is not a solution of
the Euler-Lagrange equation.
474 CHAPTER 15. SOLUTIONS TO EXERCISES

Solution for Exercise 4.9


In this case F = y 2 + y 2 + 2axy and Fy = 2y and Fy = 2y + 2ax, giving the
Euler-Lagrange equation y y = ax. The general solution of this equation is y =
C cosh x + D sinh x ax, where C and D are arbitrary constants determined by the
boundary conditions. The boundary condition at x = 0 gives C = 0 that at x = 1 gives
the solution
a+B
y(x) = sinh x ax.
sinh 1
Consider the difference S = S[y + h] S[y], where y is the above solution:
Z 1
S = 2 dx h 2 + h2 > 0

0

for all non-zero h(x). Hence the functional has a minimum.

Solution for Exercise 4.10


(a) The Euler-Lagrange equation is
d A
x2 y (x) = 0 which integrates to y (x) =

.
dx x2
Integrating again gives the general solution y(x) = B A/x. The boundary condition
at x = 1 gives A + B = 1 and hence
A
y = 1 A , 1 x < 0.
x
The boundary condition at x = 1 gives B A = 1 and hence
A
y =1+A , 0 < x 1.
x
Because each solution is discontinuous at x = 0, it is not possible to find a single solution
that satisfies both boundary conditions. The du Bois Reymond theorem, quoted on
page 135, gives some idea of the origin of this problem. The integrand of the functional
is F = x2 y 2 hence 2 F/y 2 = 2x2 , which is zero at x = 0; that is condition (d) of the
theorem is not satisfied.
The functions that satisfy the boundary conditions at x = 1 are different and both
are discontinuous at x = 0. Hence there is no continuous function that satisfies both
boundary conditions and the Euler-Lagrange equation.
(b) For the given function


0, 1 x ,

y (x) = 1/, |x| < ,

0, x 1,

so the functional is
1 2
Z
J[y] = dx x2 = .
2 3
The function is continuous provided > 0 and hence on this class of continuous functions
J[y] can be made arbitrarily small, but not zero.
15.4. SOLUTIONS FOR CHAPTER 4 475

(c) The given functions behave similarly to the piecewise continuous function defined
in part (b), as seen in figure 15.5 which depicts graphs for = 0.1 and 0.01.

0.5

-1 -0.75 -0.5 -0.25 0 0.25 0.5 0.75 1

-0.5

-1
Figure 15.5 Graphs of the functions y(x) for = 0.1 (solid
line) and 0.01 (dashed line).

With the given functions


1
y (x) =
x2 + 2
so the integrand is even and the functional becomes

22 1 x2
Z
J[y] = dx
2 0 (x2 + 2 )2
Z 1
2 1
= d sin2 where tan 1 =
2 0
and the second integral is obtained by putting x = tan . Integration gives
 
1
J[y] = (21 sin 21 ) = 2 tan
2 2 2 1 + 2
 
2 1
2 1 tan + 1+2
= 2 .
1 2 tan1

Since tan = + O( ) we see that J[y] = 2/ + O(2 ). Since 0 < < 1, J[y] > 0,
1 3

but can be made arbitrarily small.

Solution for Exercise 4.11


With the definition of (x) given in the exercise,
Z b Z b
F d
dx h(x) = dx h(x)
a y a dx
h ib Z b
= h(x)(x) dx h (x)(x).
a a

The boundary term is zero, because h(a) = h(b) = 0, so equation 4.9 becomes
Z b  
F
S[y, h] = dx (x) h (x).
a y
476 CHAPTER 15. SOLUTIONS TO EXERCISES

On a stationary path S = 0 for all admissible h(x), so the result proved in exercise 4.4
shows that F/y = C for some constant C.

Solution for Exercise 4.12


(a) Since

 1
Z b  
2 2
S[y + h] = G y(b) + h(b) + dx (y + h ) + (y + h)
2 a

diiferentiation with respect to and then setting = 0 gives the Gateaux differential
Z b
S[y, h] = Gy (y(b))h(b) + dx (h y + hy) .
a

Now integrate by parts and use the fact that h(a) = 0 to cast this in the form
  Z b
S[y, h] = y (b) Gy (y(b)) h(b) dx y y h.

a

(b) On the variations with h(b) = 0 the boundary term of S is zero. For S[y] to be
stationary it is necessary that S[y, h] = 0 and it follows from the fundamental lemma
that y y = 0 with y(a) = A.
On the path defined by this equation
 
S[y, h] = y (b) Gy (y(b)) h(b).

Since we require S[y, h] to be zero for all allowed h, which includes those variations
for which h(b) 6= 0, we must have

y (b) = Gy (y(b)).

(c) In this case G(y) = By and the condition at x = b is y (b) = B.

Solution for Exercise 4.13


The Gateaux differential is
b  
F F
Z
S[y, h] = Gy (b, y(b), B)h(b) + dx h + h .
a y y

Integrating by parts and using the fact that h(a) = 0 gives


b    
d F F
  Z

S[y, h] = Fy (b, y(b), y (b)) Gy (b, y(b), B) h(b) dx h.
a dx y y

On the variations with h(b) = 0 the boundary term of S is zero. For S[y] to be
stationary it is necessary that S[y, h] = 0 and it follows from the fundamental lemma
that  
d F F
= 0, y(a) = A.
dx y y
15.4. SOLUTIONS FOR CHAPTER 4 477

On the path defined by this equation


 
S[y, h] = Fy (b, y(b), y (b)) Gy (b, y(b), B) h(b).

Since we require S[y, h] to be zero for all allowed h, which includes those variations
for which h(b) 6= 0, we must have

Fy (b, y(b), y (b)) = Gy (b, y(b), B),

one solution of which is y (b) = B. Thus the solutions of the equation


 
d F F

= 0, y(a) = A, y (b) = B
dx y y

are stationary paths of S[y].

Solution for Exercise 4.14


In this case H = y + x, so Hy = 1 > 0 and |H| = |y + x| |y| + x, for x 0. Thus
with = 0 and we may take (Y ) = x + Y to see that the conditions of Bernsteins
theorem hold and there is a unique solution.
The general solution of this linear equation is y = x + C cosh x + D sinh x for some
constants C and D. The boundary condition at x = 0 gives C = A and that at x = 1
gives B = 1 + A cosh 1 + D sinh 1, so the solution is
sinh(1 x) sinh x
y = x + A + (B + 1) .
sinh 1 sinh 1

Solution for Exercise 4.15


In this case H = x y and Hy = 1, which contradicts the condition Hy > 0.
The general solution of this equation is y = x + C cos x + D sin x. The boundary
condition at x = 0 gives C = 0 and that at x = X gives D = (1 X)/ sin X; thus
for every X 6= n, n = 1, 2, , there is a unique solution. If X = n the equation
and boundary conditions cannot be satisfied and no solution exists. This example does
not contradict Bernsteins theorem because it provides a sufficient, but not a necessary
condition.

Solution
p for Exercise 4.16

If F = 1 + y 2 / y we have
p
F 1 + y 2 F y
= and = p
y 2y 3/2 y
y 1 + y 2

so the Euler-Lagrange equation is


! p
d y 1 + y 2
p + =0
dx y 1 + y 2 2y 3/2

which expands to
p
y y 2 y 2 y 1 + y 2 1 + y 2
p + = 0 that is y = .
y(1 + y 2 )3/2 2y 3/2
p
y 1 + y 2 2y 3/2 1 + y 2 2y

Now define y1 = y and y2 = y1 the above equation, becomes y2 = (1 + y1 2 )/(2y1 ).


478 CHAPTER 15. SOLUTIONS TO EXERCISES

Solution for Exercise 4.17


The first-integral is y 2 (1 y 2 ) = c, where c is a constant. If c = 0 then y = 0 and
y = 1 are solutions. These give the quoted result.

Solution for Exercise 4.18


(a) The triangles ABC, AB1 D and DB2 C are similar because all are isosceles and have
a common angle. Because AD is half AC it follows that AB1 = DB2 = l/2. Thus the
lengths of AB1 DB2 C and ABC are the same and equal to 2l.
Elementary trigonometry gives cos = a/(2l) and tan = 2h/a.

(b) A second division gives 22 similar triangles of height 22 h and a line of length 2l.
After n divisions there are therefore 2n similar triangles of height 2n h and a continuous
line of length 2l. Since this is true for any l, the length of the line is unbounded.

Solution for Exercise 4.19


In this case F = y 2 y 2 2xy so that Fy = 2y , Fy = 2y 2x, and the Euler-
Lagrange equation is y + y + x = 0. The general solution of this equation is y =
A cos x + B sin x x. The boundary condition at x = 0 gives A = 0 and at x = 1 we
have 0 = B sin 1 1 giving the required solution.

Solution for Exercise 4.20


If u = x a and Y (u) = y(x(u)) and c = b a, the functional becomes
Z c
S[Y ] = du F (Y , Y ), Y (0) = A, Y (c) = B,
0

so the Euler-Lagrange equation depends upon c = b a rather that a and b separately,


and hence so does the solution.

Solution for Exercise 4.21


Using the given trial function, the functional becomes
Z 1/2 Z 1
dx 4y12 4y12 x2 4y1 x2 + dx 4y12 4y12 (1 x)2 4y1 x(1 x) ,
   
S(y1 ) =
0 1/2
   
11 2 1 11 2 1 11 2 1
= y y1 + y y1 = y y1 .
6 1 6 6 1 3 3 1 2

This function is stationary at the root of S (y1 ) = 22y1 /3 1/2, that is y1 = 3/44
0.0682.

Solution for Exercise 4.22


(a) In this example F = y 2 + 12xy and F/y = 2y , F/y = 12x. Hence the
Euler-Lagrange equation is y = 6x, y(0) = 0, y(1) = 2, having the general solution
y = x3 + Ax + B, which satisfies the condition at x = 0 if B = 0 and the condition at
x = 1 if A = 1. Hence the stationary path is y = x3 + x.
15.4. SOLUTIONS FOR CHAPTER 4 479

(b) In this example F = 2y 2 y 2 (1 + x)y 2 and Fy = 4y 2 y , Fy = 4yy 2 2(1 + x)y.


The Euler-Lagrange equation is
   2
d dy dy
2 y2 2y + (1 + x)y = 0,
dx dx dx

which simplifies to (yy ) + 21 (1 + x) = 0. Integrating this gives

dy 1d 1 A
y 2 = (1 + x)2 + ,

y =
dx 2 dx 4 2
and integrating again, y(x)2 = B + Ax 16 (1 + x)3 . The boundary conditions then give
y(0)2 = B 61 = 1, so B = 76 , and y(1)2 = 67 + A 86 = 4, so A = 25 6 . Hence the
solution is
1 1
y(x)2 = (1 + x) 25 (1 + x)2 3 = 3 + (1 + x)(6 + x)(4 x).

6 6
The solution is written in this way because it is easier to understand. The cubic
f = (1 + x)(6 + x)(4 x) is zero at x = 6, 1 and 4; f is positive for x < 6 and
negative for x > 4. It follows that y is real only for x < x1 , for some x1 < 6, and
possibly for some x in the interval 1 < x < 4, depending upon the magnitude of f
in this interval. Numerical calculations, which you are not expected to do, show that
x1 6.33 and that y is real in the interval (0.264, 3.59).
(c) The Gateaux differential is
Z 2
1 y h
S[y, h] = Bh(2) + 2 dx 2 , y(1) = A,
2 1 x
  Z 2  
1 1 d y
= B + y (2) h(2) 2 dx h ,
2 2 1 dx x2

the second result being obtained using integration by parts and the fact that h(1) = 0.
Using the subset of variations with h(2) = 0 and using the fundamental lemma shows
that the stationary paths must satisfy the Euler-Lagrange equation,
 
d y dy
2
= 0 that is = x2 with y(1) = A,
dx x dx
for some constant . On the paths that satisfy this equation
1 
S[y, h] = B y (2) h(2),
2
and since h(2) need not be zero, S[y] is stationary only on those paths that satisfy
y (2) = B, because it is necessary that S[y, h] = 0 for all allowed h. The general
solution of y = x2 is y(x) = x3 /3 + and the boundary conditions give
1 1 1
A= + , B = 4 so = B and = A B.
3 4 12
Hence y(x) = B x3 1 /12 + A.

480 CHAPTER 15. SOLUTIONS TO EXERCISES

(d) The Gateaux differential is


Z b
2yh
 
2y(0)h(0) h
S[y, h] = + dx 3 , y(b) = B 2 ,
A3 0 y 2 y
  Z b   
1 1 1 d y
= 2h(0)y(0) + dx + 2 h,
y (0)3 A3 0 y 2 dx y 3

where we have integrated by parts and used the fact that h(b) = 0. Using the subset
of variations with h(0) = 0 and the fundamental lemma shows that S[y] is stationary
only on those paths that satisfy the Euler-Lagrange equation with F = yy 2 and with
the single boundary condition y(b) = B 2 . Since F is independent of x, so we may use
the first-integral, equation 4.13 (page 130), to give y y 2 = c2 , y(b) = B 2 , where c is a
positive constant (since y(b) > 0 the constant must be positive).
On the paths that satisfy this equation
 
1 1
S[y, h] = 2h(0)y(0) 3 ,
y (0)3 A

so S[y] is stationary only if y (0) = A > 0. The general solution is given by (since
y(0) = (Ac)2 ),
Z y
dy y dy x
= that is = .
dx c (Ac)2 y c

Hence y = Ac + x/2c and the boundary condition at x = b gives 2Ac2 2Bc + b = 0,
that is
1  p 
c= B B 2 2Ab
2A
giving the two solutions
 2
x 1  p 
y (x) = Ac + , c = B B 2 2Ab .
2c 2A

Solution for Exercise 4.23


For a function, G, of n variables, (x1 , x2 , . . . , xn ), a stationary point is where
n
X G
k =0 for all k .
xk
k=1

The fact that the sum is zero for all k is the equivalent of the fundamental lemma of
the Calculus of Variations.

Solution for Exercise 4.24


The Euler-Lagrange equation is
!
d w(x)y (x)
p = 0,
dx 1 + y (x)2
15.4. SOLUTIONS FOR CHAPTER 4 481
p
which integrates to w(x)y (x) = A 1 + y (x)2 , where A is a constant. Rearranging
this and integrating again gives the general solution
Z x
1
y(x) = B A du p .
a w(u)2 A2

If w(x) = x this becomes
Z x
1
y(x) = B A du
a u A2

and hence y(x) = C 2A x A2 , where C is a constant.
If w(x) = x the general solution becomes
Z x
1
y(x) = B A du ,
a u A2
2

giving y(x) = C A cosh1 (x/A).

Solution for Exercise 4.25


2
(a) Since F = y 2 1 and F/y = 4y y 2 1 the first-integral of the Euler-


Lagrange equation, equation 4.13 (page 130) is (y 2 1)(3y 2 + 1) =constant. Hence


y 2 = m2 for some constant m, which we assume positive.
(b) The solutions of the equation y (x)2 = m2 that satisfy the boundary condition
y(0) = 0 are y(x) = mx, m > 0. Hence one solution that fits the boundary condition
at x = 1 is y = y1 = Ax and on this path S[y1 ] = (A2 1)2 .
Another solution has the form
(
mx, 0 x 1,
y(x) =
c mx, x 1,

where m, c and are constants. The boundary condition at x = 1 gives c = A + m.


Since the solution needs to be continuous at x = we also have m = c m and hence
= (A + m)/2m.
Because m > 0 and 1 it follows that m A; for m = A we regain the solution
y = Ax, but for m > A we obtain y2 (x).
Another solution is (
mx, 0 x ,
y(x) =
c + mx, x 1.
The boundary condition at x = 1 gives c = A m and the continuity condition gives
m = c + m, and hence = c/2m = (m A)/2m. Since 0 this gives m A,
as before. This gives the solution y3 (x).
Since y2 (x)2 = y3 (x)2 = m2 on both paths S[y2 ] = S[y3 ] = (m2 1)2 .
(c) If A > 1, the minimum value of the functional is (A2 1)2 and this is given by the
solution y = Ax.
If A < 1, we may choose m = 1, for y2 or y3 to give the minimum value of zero.
482 CHAPTER 15. SOLUTIONS TO EXERCISES

Solution for Exercise 4.26


(a) This integral can be evaluated directly, S[y] = y(1) y(0), and its value is indepen-
dent of the path, regardless of the boundary values.
(b) Similarly S[y] = 12 (y(1)2 y(0)2 ).
(c) Since F = xyy , F/y = xy, F/y = xy and the Euler-Lagrange equation is
y = 0, which does not satisfy the boundary conditions.
Alternatively we have
1
1 1 1 1 1 1 1

d 1
Z Z Z
S[y] = dx x y 2 = xy(x)2 dx y 2 = dx y 2 .
2 0 dx 2 0 2 0 2 2 0

The Euler-Lagrange equation for the functional on the right-hand side of this equation
is again y = 0.

Solution for Exercise 4.27


We expect the Euler-Lagrange equations for these two functionals to be identical be-
cause h ix=b
S2 [y] = S1 [y] + G(x, y(x)
x=a
and the boundary term is independent of the path. Now we derive the result directly.
Consider the Euler-Lagrange equation for S2 [y]. First define
dG G G
F (x, y, y ) = F (x, y, y ) + = F (x, y, y ) + + y
dx x y
so that
F F 2G 2G F F G
= + + y and = + .
y y xy y 2 y y y
Hence the Euler-Lagrange equation for F is

2G 2G
     
d F F d F F d G
= + y.
dx y y dx y y dx y xy y 2
But,
2G 2G
 
d G
+ 2
y = ,
xy y dx y
so the Euler-Lagrange equations for F and F are identical, as expected.

Solution for Exercise 4.28


Clearly S[y] 0 and for the given solution the integrand is identically zero, so for this
solution S = 0, its minimum value. The Euler-Lagrange equation is
2
(y 2) y 2 + 2 (y 2x) yy (y 2x) y = 0,

which is satisfied by the functions y(x) = 0 and y(x) = x2 . Thus the given function
satisfies the Euler-Lagrange equation except at x = 0 where y (x) is not defined.
15.4. SOLUTIONS FOR CHAPTER 4 483

Solution for Exercise 4.29


(a) We have
b B
dx
Z Z
S= dx F (x, y, y ) = dy F (x, y, 1/x )
a A dy
so that G(y, x, x ) = x F (x, y, 1/x ).
The Euler-Lagrange equation for G is
 
d G G
= 0, x(A) = a, x(B) = b.
dy x x

Expanding this gives Gx x x + Gx x x + Gy x Gx = 0. Now replace all occurrences


of G by F , using the relations,
G F G 1 F
= x , = F ,
x x x x y
and
2G F 1 2F 2G 1 2F 2G F 1 2F

= , 2
= 3 2, = .
xx x x xy x x y
x y y x yy
Hence the Euler-Lagrange equation for G becomes
   
Fy y 1 1
x + Fx Fxy x + Fy Fyy x Fx = 0
x 3 x x

which reduces to
Fy y 1
3
x Fy y Fx y + Fy = 0. (15.1)
x x
(b) The Euler-Lagrange equation for F is Fy y y + Fy y y + Fx y Fy = 0. But

d2 y x
 
d 1 dy
2
=
= 3,
dx dy x dx x

so this equation becomes


Fy y 1
x + Fy y + Fx y Fy = 0,
x 3 x
which is the same as equation 15.1.

Solution for Exercise 4.30


If y = (y1 , y2 , , yn ) with y0 = A and yn+1 = B and xk+1 = xk + then yk occurs
only in the k and k + 1 terms and
   
S yk yk1 yk+1 yk
= F xk , yk , + F xk+1 , yk+1 ,
yk yk yk
 
yk yk1
= F (zk ) + F (zk ) F (zk+1 ), z = x, yk , .
u v v
484 CHAPTER 15. SOLUTIONS TO EXERCISES

Now we need to express (yk+1 yk )/ in terms of (yk yk1 )/: write


yk+1 yk yk yk1 yk+1 2yk + yk1
= + ,

and use the Taylor expansion
yk+1 2yk + yk1 = y(xk + ) 2y(xk ) + y(xk )
1 1
= y(xk ) + y (xk ) + 2 y (xk ) + 3 y (xk ) + O( 4 ) 2y(xk )
2 6
1 1
+y(xk ) y (xk ) + 2 y (xk ) 3 y (xk ) + O( 4 )
2 6
= 2 y (xk ) + O( 4 ).
Hence
 
yk yk1
zk+1 = xk + , yk + yk + O( 2 ), + yk + O( 3 )

= zk + (1, yk , yk ) + O( 2 ),
which gives
n o
F (zk+1 ) = F (zk ) + Fx (zk ) + yk Fu (zk ) + yk Fv (zk ) + O( 2 ).

It follows that the equation for S/yk becomes


  
S F F F F
= + yk + yk + O( 2 ),
yk u v x u v
  
F G G G F
= + yk + yk + O( 2 ), G = ,
u x u v v
  
S F d F
= + O( 2 ), k = 1, 2, , n.
yk u dx v
Since S/yk = 0 it follows that
 
d F F
= O(), k = 1, 2, , n,
dx v u
and that as 0 we obtain the Euler-Lagrange equation.

Solution for Exercise 4.31


In this more general case we use the approximations
Z 1 n Z 1 n  2
X X z(xk+1 ) z(xk )
dx z(x) h z(xk ) and dx z (x)2 h ,
0 0 h
k=0 k=0

where z(x) is any function and the set of equally spaced points xk = k/(n + 1) defined
in the question. Hence the functional becomes
n n n
1X X X 1
S = (yk+1 yk )2 h yk2 2h xk yk , h = ,
h n+1
k=0 k=0 k=0
n   
1X 2 2 2 2k
= (yk+1 yk ) h yk + yk .
h n+1
k=0
15.4. SOLUTIONS FOR CHAPTER 4 485

(a) If n = 1 there are two terms in the sum; the first is y12 /h, since y0 = 0, and the
second is (1/h h)y12 2hy1 , and since h = 1/2 this gives
7 2 1
S(y1 ) = y y1 .
2 1 2
This function is stationary where S/y1 = 7y1 1/2 = 0, that is y1 = 1/14 = 0.0714,
compared to the exact value of y(1/2) = 0.0697.
The difference between this approximation to S and that obtained in exercise 4.21 is
because the approximations to the functional are different. In both cases we approxi-
mate the solution by the same type of polygon; but in the first case we evaluated the
integrals exactly; in the second case we made an additional approximation to evaluate
the integrals. For the approximation used in exercise 4.21 we have
"Z #
Z 1 Z 1/2 1
dx y (x)2 = 4y12 dx + dx = 4y12 ,
0 0 1/2
"Z #
1 1/2 1
1 2
Z Z
2
dx y(x) = 4y12 dx x +2
dx (1 x)2
= y ,
0 0 1/2 3 1
"Z #
1 1/2 1
1
Z Z
dx 2xy(x) = 4y1 dx x2 + dx x(1 x) = y1 .
0 0 1/2 2

For the approximation used here, these integrals are approximated by


Z 1 1
X
dx y (x)2 = 2 (yk+1 yk )2 = 4y12
0 k=0
1 1 1 1
1X 2 1 k 1
Z Z X
2
dx y(x) = yk = y12 and dx 2xy(x) = yk = y1 .
0 2 2 0 2 2
k=0 k=0

(b) If n = 2, then h = 1/3, y3 = 0 and


     
 2 1 2 1 4
S = 3y1 + 3(y2 y1 )2 y12 + y1 + 3y22 y22 + y2
3 3 3 3
17 2 17 2 2 4
= y + y2 6y1 y2 y1 y2 .
3 1 3 9 9
The stationary points are at the solutions of
S 34 2 S 34 4
= y1 6y2 = 0 and = y2 6y1 = 0
y1 3 9 y2 3 9

which simplify to the given equations. These have the solutions y1 = 35/624 0.0561
and y2 = 43/624 0.0689, which are the approximate values of the solution at x = 1/3
and 2/3 respectively.
486 CHAPTER 15. SOLUTIONS TO EXERCISES

Solution for Exercise 4.32


(a) The Gateaux derivative, equation 4.5 (page 125), of this functional is
Z b
d F
dx h (x) .

S[y, h] = S[y + h] =
d =0 a y
Integrating by parts twice gives
 b Z b  
F d F
S = h (x) dx h (x) ,
y a a dx y
b Z b
d2
   
F d F F
= h (x) h(x) + dx h(x) .
y dx y a a dx2 y
But h(x) and h (x) are both zero at x = a and b, so for the functional to be stationary
we need
d2
 
F F
2
= 0. Integrating this twice gives = c(x a) + d,
dx y y
for some constants c and d.
(b) If F (z) = 12 z 2 the differential equation for y(x) is y (x) = c(x a) + d. Integrating
this twice gives
1
y (x) = c(x a)2 + d(x a) + and
2
1 1
y(x) = c(x a)3 + d(x a)2 + (x a) + .
6 2
The boundary conditions at x = a give y (a) = A2 = and y(a) = A1 = , so
1 1
y(x) = c(x a)3 + d(x a)2 + A2 (x a) + A1 ,
6 2
and the constants (c, d) are determined from the boundary conditions at x = b. Setting
D = b a the two equations y(b) = B1 and y (b) = B2 become, respectively,
1 3 1 2 1 2
cD + dD + A2 D + A1 = B1 and cD + dD + A2 = B2 ,
6 2 2
which simplify to the quoted equations.
Rb
(c) Consider the general functional S[y] = a dx F (y ), so
Z b Z b
F 1 2F
S[y + h] = S[y] + dx h (x) + 2 dx h (x)2 2 +
a y 2 a y
and on the stationary path
b
1 2 2F
Z
S[y + h] S[y] = dx h (x)2 + .
2 a y 2
Since h (x) 0 the sign of this integral depends upon 2 F/y 2 . But, in the present
2

case F (z) = z 2 /2, F (z) = 1 and hence the integral is positive and the stationary path
is a minimum.
15.4. SOLUTIONS FOR CHAPTER 4 487

Solution for Exercise 4.33


First, note that if y(x) and y(x) + h(x) are both admissible functions then h(x) and its
derivative, h (x), are zero at x = a and b. The Gateaux derivative, S[y, h] (page 125),
is Z b
d d

lim S[y + h] = dx F (x, y + h, y + h , y + h ) .
0 d d a =0

Thus
b  
F F F
Z
S[y, h] = dx h + h + h .
a y y y
Integration by parts gives
b  b Z b  
F F d F
Z
dx h = h dx h .
a y y a a dx y

Since h(a) = h(b) = 0 the boundary term vanishes. Similarly,


b  b  b
F F d F
Z Z
dx h = h dx h
a y a a y dx y
b Z b
d2
   
F d F F
= h h + dx h 2 .
y dx y a a dx y

Again the boundary terms vanish because h (a) = h (b) = 0. Hence


b
d2
    
F d F F
Z
S[y, h] = dx h(x) + 2 .
a y dx y dx y

Using the fundamental theorem of the Calculus of Variations we see that a necessary
condition for the functional to be stationary on a function y(x) is that it satisfies the
equation
d2
   
F d F F
+ = 0,
dx2 y dx y y
with the given boundary conditions.

Solution for Exercise 4.34


(a) If F = 1 + y (x)2 the required derivatives are Fy = 2y and Fy = Fy = 0, so
the equation for the stationary function is d4 y/dx4 = 0. The general solution of this
equation is the cubic y(x) = ax3 + bx2 + cx + d, where the constants a, b, c and d
are determined by the boundary condition. Those at x = 0 give y(0) = d = 0 and
y (0) = c = 1; those at x = 1 then give y(1) = a + b + 1 = 1 and y (1) = 3a + 2b + 1 = 1,
so that a = b = d = 0, c = 1 and the solution is y(x) = x.

(b) In this case Fy = 2y , Fy = 0, Fy = 2y, so the equation for the stationary


function is
d4 y  
y = 0, y(0) = 1, y (0) = 0, y = 0, y = 1.
dx4 2 2
488 CHAPTER 15. SOLUTIONS TO EXERCISES

The general solution of this is y(x) = A cos x + B sin x + D cosh x + E sinh x. The
boundary conditions at x = 0 give

y(0) = A + D = 1 and y (0) = B + E = 0

and those at x = /2 give


 
y = B + Dc + Es = 0, y = A + Ds + Ec = 1,
2 2
where c = cosh(/2) and s = sinh(/2). Using the first two equations to substitute for
D and E in the second two gives

(s 1)B + Ac = c and Bc + (s + 1)A = s + 1.

These equations have the solution A = 1 and B = 0, hence E = D = 0, and the


required solution is y(x) = cos x.
15.5. SOLUTIONS FOR CHAPTER 5 489

15.5 Solutions for chapter 5


Solution for Exercise 5.1
The gradient is
dy dy dx a sin 1
= / = = ,
dx d d a(1 cos ) tan(/2)
where we have used the identities sin 2w = 2 sin w cos w and cos 2w = 1 2 sin2 w.
The cycloid is perpendicular to the x-axis when the gradient is infinite, that is when
tan(/2) = 0, or /2 = n, n = 0, 1, .

Solution for Exercise 5.2


The Taylor series for sin and cos are given in the handbook, and the first few terms
are sin = 3 /6 + O(5 ) and cos = 1 2 /2 + O(4 ). Hence,
1 3 1 2
x = a( sin ) = a + O(5 ) and y = a(1 cos ) = a + O(4 ).
6 2
The first equation gives = (6x/a)1/3 , and substituted into the equation for y this
gives y = a(6x/a)2/3 /2.

Solution for Exercise 5.3


For a curve defined parametrically by the functions x(), y(), the area under it and
between = 1 and 2 , is
Z x(2 ) Z 2
dx
A= dx y(x) = d y().
x(1 ) 1 d
For the cycloid, x() = a( sin ), y() = a(1 cos ) and
Z 2 Z 2
A = a2 d (1 cos )2 = a2 d (1 2 cos + cos2 ) = a2 (2 + ) = 3a2 .
0 0

For the length of a curve we use a variant of equation 1.5 (page 15). Suppose that
increases from to + , then to O(), x and y increase by x () and y ()
respectively. Hence the length of the small element of the curve is, using Pythagoras
theorem p
s = x ()2 + y ()2 + O(2 ),
and the length of the curve between 1 and 2 is
Z 2 p
s= d x ()2 + y ()2 .
1

For the cycloid, x () = a(1 cos ), y () = a sin and the length of the arc OP is

Z q Z p
s = a d (1 cos )2 + sin2 = a d 2 2 cos ,
0 0
Z
= 2a d sin(/2) = 4a (1 cos(/2)) = 8a sin2 (/4),
0

where we have used the identity cos z = 1 2 sin2 (z/2) twice.


490 CHAPTER 15. SOLUTIONS TO EXERCISES

Solution for Exercise 5.4


(a) The initial energy is E = mgAX + 12 mv02 and since x decreases during the fall
equation 5.5 becomes
Z 0 X
1 + A2 1 + A2
 q
T = dx p 2 = v02 + 2gA(X x)
X v0 + 2gA(X x) gA 0

1 + A2
 q 
= v02 + 2gAX v0
gA
s
2X p
= 1 + A2 if v0 = 0.
gA

(b) The initial point (X, Y ), where Y = AX, satisfies the equation X 2 +(Y R)2 = R2 ,
which becomes (1 + A2 )X = 2AR. Substituting this into the above equation for T gives
the required, rather surprising, result.

Solution for Exercise 5.5


Since dz/dx = 1/(dx/dz), that is z (x) = 1/x (z) and when x = 0, z = 0 and when
x = b, z = A (because y(b) = 0 and v0 = 0) the funtional becomes
Z A r Z A r
dx 1 + 1/x 2 1 + x 2
T = dz = dz
0 dz z 0 z
where we have ignored the irrelevant external multiplicative factor.
In this representation the integrand, F (z, x ), is independent of x(z), so the Euler-
Lagrange equation is  
d F F 1
= 0 so that = ,
dz x x c

p
where c is a constant. 2
p But Fx = x / z(1 + x ) so the Euler-Lagrange equation

reduces to dx/dz = z/(c2 z).

Solution for Exercise 5.6


If b A then from figure 5.5 we see that u, and hence b must be small. Using the
Taylor series sin x = x x3 /6 + O(x5 ) and cos x = 1 x2 /2 + O(x4 ), we see that the
equation for b becomes
 3
b 2 4 3 3b 9 b
= b + b + that is b + ... .
A 3 45 2A 20 A
In the following only the first term of this expansion is used. Also, since d = 0,
1 2 2 1
x= c (2 sin 2) = c2 3 + O(5 ), y = A c2 (1 cos 2) = A c2 2 + O(4 ).
2 3 2
Putting x = b and = b gives the equation for c,
 3  3
2 2 3b 2 3 2A
b= c hence c = b
3 2A 2 3b
15.5. SOLUTIONS FOR CHAPTER 5 491

so that  3  2
x 2A y 2A  x 2/3
= , =1 =1 .
b 3b A 3b b

Solution for Exercise 5.7


It is convenient to write z() in the form z = c2 sin2 , use the fact that z (x) =
z ()/x () and express the integrand of the functional in terms of :
Z b s Z b s
dx 1 + z ()2 /x ()2 1 x ()2 + z ()2
T = d = d
0 d 2gz() 2g 0 z()

But z = 2c2 sin cos and, since x = 21 c2 (2 sin 2) + d, x = 2c2 sin2 , so that
x 2 + z 2 = 4c4 sin2 and
Z b
2c 2cb
T = d = .
2g 0 2g

But, from the analysis preceeding the exercise, c = A/ sin b and so
s
2A b
T = .
g sin b

Solution for Exercise 5.8


The centre of the circle is on the line y = A and since the y-axis is tangent to the circle,
the coordinates of the centre are (R, A) where R is the (unknown) radius. The equation
of this circle is (x R)2 + (y A)2 = R2 . The point (b, 0) is on this circle and hence

A2 + b2
(b R)2 + A2 = R2 giving R = .
2b
The time of passage is given by equation 5.6, with z = A y. The parametric equations
x = R(1 cos ) and y = A R sin satisfy the equation of the circle and as the particle
moves downwards from (0, A), increases from = 0 to = b where y = 0, that is

A 2Ab
sin b = = 2 ,
R A + b2
so b depends only on the ratio = b/A. Since z = R sin , using the relation dz/dx =
z ()/x (), equation 5.6 becomes
Z b s 2 s Z
1 x () + z ()2 R b 1
T = d = d .
2g 0 z() 2g 0 sin

If b A, sin b is small and we may use a small expansion to obtain approximate


expressions. For small , sin so
s Z s
R b d Rp
T =2 b .
2g 0 2g
492 CHAPTER 15. SOLUTIONS TO EXERCISES

But sin b b = A/R and hence


s r s
R A 2A
T 2 = .
2g R g

Solution for Exercise 5.9


The general equation of a straight line can be written as y = m(x a) + c. The line
passes through (a, A), so c = A, and through (b, B) so B = m(b a) + A and hence
(B A)
y= (x a) + A
ba
is the required equation.
Substituting this into equation 5.11, with u = x a, gives
Z ba  s  2
BA BA
S[y] = 2 du u+A 1+ ,
0 ba ba
(b a)2 + (B A)2 ba
p Z
 
= 2 2
du (B A)u + A(b a)
(b a) 0
p
= (B + A) (b a) + (B A)2 .
2

Pythagoras theorem gives l2 = (b a)2 + (B A)2 , hence S = (B + A)l.

Solution for Exercise 5.10


(a) If y(x) is even and y = + (A )x/a for 0 x a then
Z a  
S() 2p 2 2
A
= a + (A ) dx + x
2 a 0 a
p
= (A + ) a2 + (A )2 .

(b) Differentiating with respect to gives


1 dS 22 2A + a2
= p .
2 d a2 + (A )2
Thus S() is stationary when
1 p 
22 2A + a2 = 0 that is = = A A2 2a2 .
2

There are two, real stationary points only if A > a 2 and none if A is smaller.
(c) The quadratic 22 2A + a2 is negative for < < + , so as increases from
to + , S() decreases and hence S( ) > S(+ ).

(d) If A > a 2 we may write
r
2a2 a2 1 a4
p  
2 2 6
A 2a = A 1 2 = A 1 2 + O((a/A) )
A A 2 A4
15.5. SOLUTIONS FOR CHAPTER 5 493

and hence
a2 a4 a6 a2 a4
   
= + +O , + = A +O .
2A 4A3 A5 2A A3

A
Hence y (x) x and y+ A. With = + , y(x) A and the solution approxi-
a
mates a right circular cylinder. With = , y(x) Ax/a, so the solution increases as
|x| increases. We shall see later that both these solution behave like the exact solutions.

Solution for Exercise 5.11


(a) If f (x) = c cosh(x/c), f (x) = sinh(x/c) and the functional 5.11 (page 155), with
the appropriate change to the limits, becomes,
Z a Z
x a
q
2
S[f ] = 2c dx cosh(x/c) 1 + sinh (x/c) = 4c 2
du cosh2 u, u = , = ,
a 0 c c
= 2c2 ( + sinh cosh ) ,

where we have used the relations 2 cosh2 u = 1 + cosh 2u to evaluate the integral and
sinh 2u = 2 sinh u cosh u to cast the result in this form. Dividing this by a2 we see that
S[f ] 2
the dimensionless area S[f ]/a2 depends only upon , 2 = 2 ( + sinh cosh ) .
a
(b) Since 2 sinh cosh = sinh 2 we define

1 sinh 2 1 sinh 2
F () = + giving F () = (cosh 2 1) .
2 2 2 3

Hence F () = 0 if
(cosh 2 1)
1= = tanh .
sinh 2

Solution for Exercise 5.12


(a) Using the expansion cosh = 1 + 21 2 + O( 4 ), we obtain the small expansion of
g()
1 1 1
g() = cosh = + + O( 3 ),
2
so for small the solution of the equation g() = A/a is a/A. But since = a/c
this gives c A and f (x) = A cosh(x/A) A since |x| a A.
With f (x) = A the area is S = 4Aa. Alternatively, since = a/A 1, so cosh 1
and sinh the result derived in the previous exercise gives S[f1 ]/a2 = 4/, and
hence S1 = 4Aa.

(b) The equation for can be written as

A 1  e
e + e = 1 + e2 .

=
a 2 2
494 CHAPTER 15. SOLUTIONS TO EXERCISES

If 1 the e2 term is negligible by comparison to 1, for instance if = 3, e2 =


0.0025 and if = 5, e2 = 0.0005. Hence, the equation becomes

A 1
= e , ( 1).
a 2

For large ,
1  1 1  1
cosh = e 1 + e2 e and sinh = e 1 e2 e
2 2 2 2
so    2
S[f ] 2 1 2 e 2
2 + e = 2 + .
a2 4 2
Since 1, e2 , that is e2 / 2 1/, so the first term dominates.

Solution for Exercise 5.13


(a) The derivative of g() is g () = 1 sinh 2 cosh which is zero when tanh = 1.
The graphs of y = tanh and y = 1/, for > 0, are shown in the following figure:
tanh increases montonically from zero to unity as increases from 0 to infinity and
1/ decreases monotonically from infinity to zero over the same range of hence there
is one and only one positive real root of tanh = 1.

1.25 y
y=tanh
1
0.75
0.5 y=1/
0.25

0 1 2 3 4
Figure 15.6 Graph of y = tanh and y = 1/.

A numerical calculation shows that g () = 0 at = m = 1.1997 and here g(m ) =


1.5089.

(b) At the stationary point the area is, using the result obtained in exercise 5.11
 
2 1 1
S = 2a + 2 sinh m cosh m
m m
2a2  2a2
= 1 + sinh2 m = cosh2 m since m sinh m = cosh m .
m m

But, by definition,

A 1
= g(m ) = cosh m hence S = 2a2 m g(m )2 = 2A2 m .
a m
15.5. SOLUTIONS FOR CHAPTER 5 495

Solution for Exercise 5.14


Since
 2
S1 2 1 A SG A
2
= 2 ( + sinh cosh ) , where cosh = , and = 2 ,
a a a2 a
the equation S1 = SG gives
2 1
( + sinh cosh ) = 2 2 cosh2 or + sinh cosh = cosh2 .
2
Using the definitions sinh = (e e )/2 and cosh = (e + e )/2 this becomes
1 2  1 2
e e2 = e + 2 + e2 or 1 + e2 = 2.

+
4 4
With increasing from zero the right-hand side increases monotonically from zero and
the left-hand side decreases monotonically from 2 to 1. Hence the equation has one
positive root: this is = g 0.639232, which gives the value A/a = g1 cosh g =
1.8950.

Solution for Exercise 5.15


(a) The functional does not depend explicitly upon x, so we may use the first-integral
p of
2 ).
the Euler-Lagrange equation y F/y F = constant, where F (y, y ) = y(1 + y
p
This gives y = c 1 + y 2 , where c is a positive constant. Re-arranging this equation
then gives the first-order differential equation,
 2
2 dy
c = y c2 , y(1) = y(1) = A.
dx
This equation is separable so can be written in terms of two integrals
dy
Z Z
c p = dx,
y c2
and integration gives
p (x + )2
y c2 = x + or y = c2 +
2c
4c2
for some constant . The boundary conditions at x = 1 give
( + 1)2 ( 1)2
A = c2 + = c 2
+ .
4c2 4c2
Hence = 0 and A = c2 + 1/4c2 . This last equation is a quadratic in c2 so gives
1 p 
c2 = c2 = A A2 1 .
2
Hence, if A > 1 there are two solutions of the Euler-Lagrange equation, but none if
A < 1. The two solutions are,
1
y (x) = 2 4c4 + x2 .

4c
Typical graphs of y (x) are shown in the next two figures: note, that for large values
of A, y+ (x) A.
496 CHAPTER 15. SOLUTIONS TO EXERCISES

A=1.2 A=3
1.25 3
y+(x) y+(x)
1
2
0.75
y(x)
0.5
1 y(x)
0.25
x x
-1 -0.5 0 0.5 -1 1 -0.5 0 0.5 1
Figure 15.7 Graphs of y (x) for A = 1.2, on the left, and A = 3 on the right.

(b) Substituting the general solution (for any c) into the functional gives
r
1 1
Z 1
x2 1
Z p
dx 4c4 + x2 ,

S[y] = 4 2
dx 4c + x 1 + 4 = 3
2c 1 4c 4c 1
1
= 2c + 3 . (15.2)
6c
In order to determine which path gives the largest value of S[y] we consider the difference
 
1 1 1
S[y ] S[y+ ] = 2(c c+ ) + 3 ,
6 c3 c+
 2
c + c+ c + c2

= (c+ c ) + 2 ,
6(c+ c )3
4
= (c+ c )(A 1) > 0 if A > 1,
3
where we have used the relations c+ c = 21 and c2+ + c2 = A, which follow directly
from the original quadratic equation for c2 . This relation shows that S[y ] > S[y+ ] for
A > 1.

If A = 1, c+ = c = 1/ 2 and S = 4 2/3. Further if A 1 we have
r !   
2 A 1 A 1 1
c = 1 1 2 = 1 1 +
2 A 2 2A2 8A4

where we have used the binomial expansion 1 x = 1 21 x 18 x2 + . Hence
 
2 1 1
c+ = A 1 +
4A2 16A4
and on taking the square root
1/2

    
1 1 1
c+ = A 1 1 + + = A 1 + .
4A2 2A2 8A2
Similarly
   
1 1 1 1
c2 = 1+ + giving c = 1+ + .
4A 4A2 2 A 8A2
15.5. SOLUTIONS FOR CHAPTER 5 497

Putting c+ = A and c = 1/2 A we obtain the following approximations
x2 1
y+ A + A and y + Ax2 Ax2 , A 1.
2A 4A
Substituting these approximations for c into the integral 15.2 for S we obtain
4
S[y+ ] 2 A and S[y ] A3/2 .
3

For A close to 1, we find the value of S[y ] by setting A2 = 1 + B 2 , where B is a small


positive constant. This gives
B2 B3
 
1 p  1 B
c2 = 1 + B2 B giving c = 2 + +
2 2 4 16 32
which gives
4 B2 B3
2 + + .
S[y ] =
3 2 3 2
This shows that, as expected, at A = 1, (B = 0), S[y ] = S[y+ ], but also that the two
curves join tangentially at A = 1, as shown in the following figure.

12 S[y]
10
S[y ]
8
6
S[y+]
4
2
A
0
1 1.5 2 2.5 3 3.5 4
Figure 15.8 Graphs of S[y ].

(c) The Goldschmidt curve is defined by


(
0, |x| < 1,
yG (x) =
A, |x| = 1,

so y (x) is not defined at |x| = 1. Hence we define a function that approaches yG (x) as
0 for some parameter . We need only consider positive values of x:

0, 0 x < 1 , 0 < 1,
y (x) = A
A (1 x), 1 x 1.

Then
r
1
r
A A2
Z
S[y ] = 2 dx A (1 x) 1 + 2 ,
1
Z r
2 p v 4p 4
= A(A2 + 2 ) dv 1 = A(A2 + 2 ) A3/2 as 0.
0 3 3
498 CHAPTER 15. SOLUTIONS TO EXERCISES

Solution for Exercise 5.16


The lengths ll and angles k , k = 1, 2, 3 are shown in figure 15.9.

C=(c,d)

3
l3 X 2
3
X
l1
l2 1
1 2
O A
Figure 15.9

The point X has the coordinates (x, y) and we need to find these coordinates so that
the length L = l1 + l2 + l3 is stationary. With the geometry shown
p p p
l1 = x2 + y 2 , l2 = (a x)2 + y 2 , l3 = (c x)2 + (d y)2 ,
y y dy
sin 1 = , sin 2 = , sin 3 = ,
l1 l2 l3
x ax cx
cos 1 = , cos 2 = , cos 3 = .
l1 l2 l3
The derivatives are
L x ax cx
= = cos 1 cos 2 cos 3 = 0,
x l1 l2 l3
L y y dy
= + = sin 1 + sin 2 sin 3 = 0.
y l1 l2 l3

Adding multiples of these gives

ei1 ei2 ei3 = 0 or ei(1 +2 ) ei(2 +3 ) = 1.

Now let k , k = 1, 2, 3, be the angles between the intersecting lines, as shown on the
right of the figure, so 1 + 2 + 3 = 2. Also 1 = 1 2 , 2 = 3 + 2 and
3 = + 1 3 , so that

ei1 + ei2 = 1 giving sin 1 = sin 2 and cos 1 + cos 2 = 1.

The first of these equations has the solutions 1 = 2n+2 and 1 = (2n+1)+2 , but
only the first of these also solves the second equation, and then only if cos 2 = 1/2,
that is 2 = 1 = /3, and hence 1 = 2 = 3 = 120.
In order to classify this stationary point we need the second derivatives: these are

2L x2 (a x)2 (c x)2
     
1 1 1
= 3 + +
x2 l1 l1 l2 l23 l3 l33
y2 y2 (d y)2
= 3 + 3 + > 0.
l1 l2 l33
15.5. SOLUTIONS FOR CHAPTER 5 499

Similarly,
2L x2 (a x)2 (c x)2
= 3 + 3 + >0
y 2 l1 l2 l33
2L xy (a x)y (c x)(d y)
= 3 + + .
xy l1 l23 l33
For a minimum we need Lxx > 0, Lyy > 0 and = Lxx Lyy L2xy > 0. Using the above
expressions we find that
a2 y 2 1  2 1  2
= + yc + xd 2xy + y(c x) (d y)(a x) > 0.
(l1 l2 )3 (l1 l3 )3 (l2 l3 )3
Hence the stationary point is a minimum.

Solution for Exercise 5.17


In case (a), since each diagonal has length L 2 the total length is 2 2L 2.83L.
In case (b) there are three equal length lines, giving a total length 3L. For case (c),
consider the end isosceles triangle; its height is h and base L, with third side d, so

L L L 3 L
= tan = 3 giving 2h = and = sin = giving d = .
2h 3 3 2d 3 2 3

The total length is L = 4d (L 2h) = (1 + 3)L.

Solution for Exercise 5.18


Using the same geometry
as in exercise 5.17 we see that in the first case d1 = 4d +
(aL 2h) = (a + 3)L. In the second case in the calculation of d and h L is replaced
by aL and then d2 = 4d + (L 2h) = (1 + a 3)L.
The functions d1 and d2 linear in a and are equal when a = 1; since d2 has the
larger gradient the inequalities follow.

Solution for Exercise 5.19


The first-integral is
y
p = c, y(0) = 0, y(a) = A > 0.
1 + y 2
At the origin y(0) = 0, so c = 0. Hence there is no differentiable solution.

Solution for Exercise 5.20


The element of length, s, is given by s2 = x2 + y 2 + z 2 , but on the sphere
x = r sin sin , y = r sin cos and z = r cos , where r is the radius. If and are
assumed to depend on a parameter t, we have
 2  2  2  2
ds dx dy dz
= + + .
dt dt dt dt
The chain rule gives
dx d d
= r cos sin + r sin cos ,
dt dt dt
dy d d dz d
= r cos cos r sin sin and = r sin .
dt dt dt dt dt
500 CHAPTER 15. SOLUTIONS TO EXERCISES

On squaring and adding these we see that the cross-terms cancel and that
 2 (  )
2  2
ds 2 d d 2
=r + sin .
dt dt dt

Hence, if the end points of the curve are t = 0 and 1


s 
Z 1 2  2
d d
S=r dt + sin2 .
0 dt dt

If we put t = and t = the two different expressions for S are,


Z b q Z b q
2
S=r 2
d 1 + () sin = r d ()2 + sin2
a a

and the two Euler-Lagrange equations are, respectively,


!
d sin2
q
p = 0 that is sin = c 1 + 2 sin2 ,
2
d 2
1 + sin 2

and !
d sin cos
p p =0
d 2 + sin2 2 + sin2
Expanding this gives the equation quoted.
(a) Using as the dependent variable, the initial condition = 0 gives c = 0 and hence
() = constant, which is the equation of the great circles through the poles.
(b) If a = b = /2, the origin may be chosen to give (a ) = 0. The equation for ()
can be simplified by noting that, for any f ()

d
( f ()) = f () + 2 f (),
d

so by choosing f = 1/ sin2 the above equation can be written in the form


 

 
d cos d cos
2 = , but also = 2 .
d sin sin d sin sin
Hence
cos
= A cos + B sin
sin
for some constants A and B. At = a = /2, since = 0, we see that A = 0; and
since b = /2, we must also have B = 0 and hence () = /2 for all ; that is the
stationary path is along the equator, as might be expected.
15.5. SOLUTIONS FOR CHAPTER 5 501

Solution for Exercise 5.21


(a) We require the general solution of equation 5.13 (page 157) but with the boundary
conditions y(0) = A > 0, y(b) = B A. The general solution of this equation is shown
in the text to be  
x
y = c cosh , c > 0,
c
where c and are constants, to be determined by the boundary conditions. These give,
     p  
b b 2 2
b
A = c cosh and B = c cosh = A cosh A c sinh .
c c c c

Since there are three lengths, b, A and B, we expect the solution to depend upon only
two ratios, which we take to be A = A/b and B = B/b. Defining a third dimensionless
ratio, = c/b, gives
  q  
1 2 1
B = f () where f () = A cosh A 2 sinh .

Since B is real and > 0, we need 0 A.


    
1 1 1
(b) If x is small and positive, cosh(1/x) = sinh(1/x) = exp + O exp
2 x x
and
x2 exp(1/x)
 q 
1 1/x 2
   
f (x) = e A A x + O e1/x = 
2 q  + O e1/x
2 2
2 A + A x2

x2 1/x
= e + O(x4 ), if x A.
4A
Now suppose that x A and set x = A u, where u is small and positive, and the
Taylor expansions are, to first-order in u
1 1 u 1 1 1 u 1
cosh = cosh + 2 sinh , sinh = sinh + 2 cosh
x A A A x A A A
and also
1/2 p
p
q 
2 2
p u
A x = u 2A u = 2Au 1 = 2Au + O(u).
2A
These expansions give
1 p 1
f (x) = A cosh 2Au sinh + O(u), u = A x.
A A
Thus as x 0, f (x) and as x A (from below) f (x) A cosh(1/A) from below.
Also for 0 < x A, f (x) is continuous and positive. It follows that f (x) has at least
one minimum for 0 < x A and that if B > min(f ) the equation for has at least two
real roots; if B < min(f ) there are no real roots.
502 CHAPTER 15. SOLUTIONS TO EXERCISES

It is difficult to prove that there is only one minimum, but numerical results suggest
this to be the case. In the following figure we plot graphs of the scaled function
  p  
f (Ay) 1 1
g= = cosh 1 y 2 sinh , 0 < y 1,
A Ay Ay

for various values of A.

10
A=0.35
8
A=5
A=1 A=0.5
6

4 A=10

0
0 0.2 0.4 0.6 0.8 1
Figure 15.10 Graphs of the function g(y) for A = 10, 5, 1, 0.5
and 0.35.

(c) If A 1, x is necessarily small and for f (x) we may use the approximation derived
in part (b),
q
q 2 2
A x
 
1 1/x 2 1 A x
f (x) e A A x2 so that f (x) e1/x 2+q .
2 2 x2 x 2 2
A x

Thus the minimum is approximately at at the root of


q
2
A x2 x A sin
+q = 2 or =A
x2 2 x (1 + sin ) cos
A x2

where x = A cos . Since A is small so is and to a first approximation x A


1 1
and min(f ) A exp . Thus the Euler-Lagrange equation has two solutions if
 2 A
1 1
B > A exp , approximately. For smaller values of B the equation B = f () has
2 A
no real solutions.

Solution for Exercise 5.22


(a) The general solution of the Euler-Lagrange equation is given by equations 5.6
and 5.8, that is

1 v02
x = d + c2 (2 sin 2) , y =A+ c2 sin2
2 2g
15.5. SOLUTIONS FOR CHAPTER 5 503

where c and d are constants and the path starts at (x, y) = (0, A), where = 0 , and
ends at (b, 0), where = b . We need equations for the four unknowns c, d, 0 and b ,
in terms of A, b and v0 . The initial conditions give
1 v02
d = c2 (20 sin 20 ) and c2 sin2 0 = .
2 2g
The final end point conditions give
1 v02
b = d + c2 (2b sin 2b ) and c2 sin2 b = A + .
2 2g
From these equations we see that 0 and b are related by the equation
sin2 0 v02
2 = k2 = that is sin2 0 = k 2 sin2 b .
sin b 2Ag + v02
Then b is determined by
1 2n o
b = c (2b sin 2b ) (20 sin 20 )
2
v02 n o
= 2 (2b sin 2b ) (20 sin 20 ) , 0 = sin1 (k sin b ).
4gk 2 sin b
This gives b , which then allows 0 to be determined and from these c and d are found.
(b) In the limiting cases v02 2Ag, we expect the solution to be close to the v0 = 0
solution found in the text. In this cases k 2 v02 /(2Ag) 1, so 0 is small and, to a
first approximation is given by 0 = k sin b . Thus the above equation for b becomes
A n
3
o
b= (2b sin 2b ) + O(k ) .
2 sin2 b
The function on the right-hand side of this equation is monotonic increasing for 0
b < : for small b it behaves as 2Ab /3 and it is infinite at b = . Hence, for all
b 0 there is a unique real solution. In the limit v0 = 0 this is the same equation
determined in the text the equation immediately preceding 5.9. With this value of
b we have
A + v02 /2g
c2 = and 0 = k sin b + O(k 3 ).
sin2 b
If v02 2Ag we should expect gravity to have little effect because the initial kinetic
energy (mv02 /2) greatly exceeds the initial potential energy (mgA), so the motion will
be close to the straight line joining (0, A) to (b, 0).
In this case k 1 and we can write
1 2Ag
k2 = , = 1,
1+ v02
so is the ratio of the potential and kinetic energies. Then
sin b
sin 0 = or 0 = sin1 (sin b sin b )
1+
504 CHAPTER 15. SOLUTIONS TO EXERCISES

where
1
=1 = .
1+ 1++ 1+ 2
Now expand the equation for 0 as a Taylor series in ,
1
0 = b tan b + 2 tan3 b + O( 3 ).
2
This equation already shows that the path is approximately a straight line, because
b 0 is O(), and this short segment of the ellipse is approximated, to this order, by
a straight line. However, we shall continue with the analysis.
The equation relating b to b is obtained using the following expansion, correct to O(),
20 sin 20 = 2b 2 tan b sin (2b 2 tan b ) + O(2 )
= 2b sin 2b 4 tan b sin2 b + O(2 )
so that the equation for b becomes
v02 p
b= 1 + tan b = A tan b .
g
Thus b is the angle between the downward vertical and the straight line between the
end points.
Now put = b tan b , where is a parameter such that = 0 and b when
= 1 and 0, respectively. The x-coordinate is
1 2
x= c {(2 sin 2) (20 sin 20 )}
2
and since, to first-order,
2 sin 2 = 2b 2 tan b sin (2b 2 tan b )
= 2b sin 2b 2 tan b sin2 b
we find that x = 2c2 (1 ) tan b sin2 b . But c2 sin2 b = A(1 + )/, tan b = b/A
and = /2 so x = (1 )b. For the y-coordinate, since sin = (1 ) sin b
A
y = (1 + ) c2 (1 )2 sin2 b

A A
= (1 + ) (1 + )(1 )2 A.

As expected this gives the parametric equation of a straight line between the initial and
final points.

Solution for Exercise 5.23


Use the result given in exercise 3.2 (page 97) and the fact that the term O() is,
by definition, zero on the stationary path to cast the difference in the first required
form. Now change the integration variable from z to , and use the result x (z) =
x ()/z () = tan to obtain the given integral. This integral exists and is positive;
hence the stationary path is a local minimum.
15.5. SOLUTIONS FOR CHAPTER 5 505

Solution for Exercise 5.24


Use the formula derived in exercise 5.4 with X replaced by x and Y by f (x) to derived
the required expression. The function T (x) is independent of x, so the differential of
gT 2 /2 is zero:
2x x2 f
 
d 1 2
gT = 2 + f = 0
dx 2 f f
and hence
df 2xf
= 2 .
dx x f2
This homogeneous equation is solved by introducing a new function v(x) defined by
f = xv, so the equation becomes separable,

v(1 + v 2 ) 1 v2
  Z
dv 1 2v dx
Z Z
x = or dv = dv = .
dx 1 v2 v(1 + v 2 ) v 1 + v2 x
This integrates to
v f
= Ax and since v = this gives x2 + (f )2 = 2 .
1 + v2 x
This equation represents a circle of radius with centre at (0, ).

Solution for Exercise 5.25


(a) Consider the functional
Z a p
S[y] = dx y 3 1 + y 2, y(a) = A.
a

y3
The integrand is independent of x, so the first-integral is p = c3 . Symmetry
1 + y 2
about x = 0 suggests that y(x) is even, so y (0) = 0 and then y(0) = c, where c is
positive. Rearranging this gives
Z y Z y
 y 6 c3 du 1
y 2 = 1 or x = = c3 du p .
c c u 6 c6
c (u 2 c2 )(c4 + c2 u2 + u4 )

Now put y = c cosh (x) where (x) is defined implicitly by


Z
x 1
= dv p .
c 0 1 + cosh v + cosh4 v
2

If (a) = a then A, c and a are related by A = c cosh a and, from the above integral
a
a 1 a
Z
cosh a = dv p . that is = f (a )
A 0 1 + cosh v + cosh4 v
2 A

where z
1 1
Z
f (z) = dv p .
cosh z 0 1 + cosh v + cosh4 v
2
506 CHAPTER 15. SOLUTIONS TO EXERCISES

(b) Since, for z > 0


Z z Z
1 1
dv p <= dv p
2 4
0 1 + cosh v + cosh v 0 1 + cosh v + cosh4 v
2

we have f (z) / cosh z < . Thus the equation a/A = f (a ) has real solutions only
if a < A: for large separations of the ends, a > A, there are no solutions of the
Euler-Lagrange equation. Numerical evaluation of the integral gives = 0.701
Now we show, by approximating f (z), that for small z, f (z) is increasing and for large
z it is decreasing, so f (z) has at least one maximum and the equation a/A = f (a ) has
at least two real roots for small a/A.
For small v

3 1 1 1
p =q = 1 v2 + v4 + ,
2 4
1 + cosh v + cosh v 1 + sinh2 v + 1 4 2 24
3 sinh v

since sinh v = v + v 3 /6 + , where the expansion is valid if sinh2 v + 1


3 sinh4 v < 1,
that is |v| < 0.801. Hence for small z

z 2
f (z) = z 3 + .
3 3 3

For large z we write, using the above definition of


 1/2
1 1 1 1
Z
f (z) = ( g(z)) with g(z) = dv 1+ + .
cosh z z cosh2 v cosh v cosh4 v
2

Provided cosh2 v + cosh4 v > 1, that is v > 0.722, we may expand the square root to
give Z  
dv 1 1
g(z) = 1 + .
z cosh2 v 2 cosh2 v 8 cosh4
But

dv 22n 2nz
Z Z
2n
= 22n dv e2nv 1 + e2v 1 + O(e2z ) .

= e
z cosh2n v z 2n

Hence g(z) = 2e2z + O(e4z ) and f (z) 2ez .

Solution for Exercise 5.26


Consider a segment of width x having volume V = y 2 x and surface area S =
2ys, s being the arc length, determined in exercise 5.3 (page 149). The para-
metric equations of the cycloid are x = a( sin ), y = a(1 cos ) and then
s() = 8a sin2 (/4). Thus the integrals for the area and volume are
2 2
ds dx
Z Z
S = 2 d y() and V = d y()2 .
0 d 0 d
15.5. SOLUTIONS FOR CHAPTER 5 507

Using the expressions for y and s we find that the surface area is
Z 2 Z 2
S = 4a 2
d (1 cos ) sin(/2) = 8a 2
d sin3 (/2)
0 0
/2
64 2
Z
= 32a2 d sin3 = a .
0 3
Similarly the volume is
Z 2 Z 2
3
V = a3 d (1 cos ) = a3 d 1 + 3 cos2 = 5 2 a3

0 0

where we have used the fact that the mean of odd powers of the cosine function is zero,
Z 2+a
dx cos2n+1 x = 0 for any real a.
a

Solution for Exercise 5.27


(a) Considerpan element of the container of width y forming a ring of radius x and
width s = y 2 + x2 . The surface area and volume of this segment are S = 2xs
and V = x2 y, so that
Z Z
ds dy
S() = 2 d x() and V () = d x()2 .
0 d 0 d
Since x = a( sin ), y = a(1 cos ) and s() = 8a sin2 (/4) these become the
expressions quoted.
(b) If x is small, is small and sin = 3 /6 + O(5 ) giving
Z  3
a2 5


S() = 4a2 d + O(6 ) = + O(7 ).
0 6 2 15
 
But x = a 3 /6 5 /120 + O(7 ) so that
 1/3  2/3 !
6x 1 6x
= 1+ +
a 60 a

and hence
2 2/3 1/3 5/3
S(x) = 6 a x + O(x7/3 ).
5
Similarly the volume for small is
Z  3 2 !
3 9 a3 8
V () = a d + O( ) = 2
+O(10 ) = 62/3 a1/3 x8/3 +O(x10/3 ).
0 6 8.6 8

(c) For S() we have


Z
S() = 4a2 d ( sin(/2) sin sin(/2))
0
Z  
2 1 1
= 4a d sin(/2) cos(/2) + cos(3/2) .
0 2 2
508 CHAPTER 15. SOLUTIONS TO EXERCISES

Using integration by parts


Z
d sin(/2) = 4 sin(/2) 2 cos(/2)
0

and hence
 
1 32 2
S() = 4a2 3 sin(/2) 2 cos(/2) + sin(3/2) with S() = a .
3 3

For the volume,


Z
V () = a3 d 2 sin 2 sin2 + sin3 .

0

But

1

1 1
Z Z
3
d sin = d (3 sin sin 3) = 3 cos + cos 3
0 4 0 4 3 0
2 3 1
= cos + cos 3
3 4 12
and

1
Z Z
d sin2 = d (1 cos 2)
0 2 0
(  )
1 2 1 1 1
Z
= sin 2 d sin 2
4 2 2 0 2 0
 i 
1 2 1 1 1h
= sin 2 cos 2
4 2 2 4 0
1 2 1 1
= sin 2 + (1 cos 2).
4 4 8
and finally
Z h i Z
d 2 sin = 2 cos + 2 d cos
0 0 0
( )
h i Z
2
= cos + 2 sin d sin
0 0
2
= cos + 2 sin 2(1 cos ).

Combining these integrals we obtain,


 
19 5 1 1 1 1
V () = a3 + cos + cos 2 + cos 3 + (4 sin + sin 2) 2 (1 + 2 cos ) .
12 4 4 12 2 2

If = , V = a3 ( 2 /2 8/3).
15.5. SOLUTIONS FOR CHAPTER 5 509

Solution for Exercise 5.28


(a) The gradient of the tangent at Q is given by

dy sin 1
tan = = = ,
dx 1 cos tan(/2)

where we use the identities sin 2x = 2 sin x cos x, cos 2x = 12 sin2 x. Hence tan tan(/2) = 1,
so cos( + /2) = 0 which means that + /2 is an odd integer multiple of /2. But
when = 0, = /2 and when = , = 0, so + /2 = /2.
(b) If s() is the length OQ the straight line QR is of length l s() and the horizontal
and vertical distances from Q to R are (l s()) cos and (l s()) sin , respectively.
Since = /2 /2 we see that the coordinates of R are

xR = xQ + (l s()) sin(/2) and yR = yQ + (l s()) cos(/2).

(c) Since s() = 8a sin2 (/4), see exercise 5.3 (page 149), the length OQC is given by
putting = , LOCD = 4a. Then if l = LOCD

xR = a( sin ) + 4a 1 2 sin2 (/4) sin(/2)




= a( sin ) + 4a cos(/2) sin(/2) = a( + sin ),

and

= a(1 cos ) + 4a 1 2 sin2 (/4) cos(/2)



yR
= a(1 cos ) + 4a cos2 (/2) = a(1 cos ) + 2a(1 + cos ) = a(3 + cos ).
510 CHAPTER 15. SOLUTIONS TO EXERCISES

15.6 Solutions for chapter 6


Solution for Exercise 6.1
dy dy dz dy 1 1 dY
(a) With x = z 2 the chain rule gives = = = , where
dx dz dx dz dx/dz 2z dz
Y (z) = y(z 2 ), so the functional becomes
2 !
Z 
1 dY
Z
2 2
S[Y ] = 2 dz z Y , where Z = X 1/2 ,
0 4z 2 dz
Z  
1 1 2
Z
= dz Y 4z 2Y 2 .
2 0 z

The Euler-Lagrange equation is now


 
d Y
+ 4z 2Y = 0,
dz z

which expands to zY Y + 4z 3 2 Y = 0. With = 1 this gives equation 6.1, with


a = 2.

(b) The second derivative is obtained using the chain rule again,

d2 y
   
d 1 dY 1 1 d 1 dY
= =
dx2 dz 2z dz dx/dz 2z dz 2z dz
2
   2 
1 1 d Y 1 dY 1 d Y dY
= = z .
2z 2z dz 2 2z 2 dz 4z 3 dz 2 dz
 2 
1 d Y dY
2
Hence y + y = 0 becomes z 2 + 2 Y = 0, giving the same Euler-
4z 3 dz dz
Lagrange equation for Y (z), as before.

Solution for Exercise 6.2


(a) Here the integrand is F = y 2 so Fy = 2y and the Euler-Lagrange equation is
y = 0; alternatively apply the first-integral, equation 4.13 (page 130), to give y 2 = c.

(b) If y = G(z) the chain rule gives y (x) = G (z)z (x) and the functional becomes
Z b
S[z] = dx G (z)2 z 2 .
a

Now the integrand is F = G (z)2 z 2 , giving Fz = 2G (z)2 z and Fz = 2G (z)G (z)z 2 ,


so the Euler-Lagrange equation becomes

d  2 
G (z) z G (z)G (z)z 2 = 0.
dx
15.6. SOLUTIONS FOR CHAPTER 6 511

d  2 
But G (z) z = G (z)2 z + 2G (z)G (z)z 2 , and hence the Euler-Lagrange equa-
dx
tion becomes z G (z)2 + G (z)G (z)z 2 = 0. But G(z) is monotonic, so G (z) 6= 0, and
the Euler-Lagrange equation is

G (z)z + G (z)z 2 = 0.

Now make the same change of variables in the original Euler-Lagrange equation y = 0.
The chain rule gives
2
d2 y d2 z

dy dz dz
= G (z) and = G
(z) + G (z) ,
dx dx dx2 dx2 dx

which leads to the same equation as derived above.

Solution for Exercise 6.3


The chain rule gives y (x) = G (z)z (x), so the functional becomes
Z b
S[z] = dx F (x, G(z), G (z)z ).
a

Since
F F y F F y F y

= = Fy G (z) and = + = Fy G (z) + Fy G (z)z
z y z z y z y z
the Euler-Lagrange equation is
d
(Fy G (z)) Fy G (z) Fy G (z)z = 0,
dx
where F and its derivatives, Fy and Fy , are evaluated at y = G(z) and y = G (z)z .

Solution for Exercise 6.4


If r cos = r sin , y (x) is infinite, equation 6.13; since in the original Cartesian formu-
lation we implicitly assume that y (x) is continuous, and hence bounded in any finite
region, we may also assume that r cos r sin 6= 0

Solution for Exercise 6.5


Since
dy dy dx sin + r cos
= / =
dx dr dr cos r sin
the functional 6.10 becomes
Z rb
sin + r cos
 
dx
S[] = dr F r cos , r sin ,
ra dr cos r sin

and since dx/dr = cos r sin , this gives the required result.
512 CHAPTER 15. SOLUTIONS TO EXERCISES

Solution for Exercise 6.6



(a) Since Fr = r / r2 + r 2 and Fr = r/ r2 + r 2 the Euler-Lagrange equation is
r
 
d r
=0
d r2 + r 2 r2 + r 2
which expands to
2
r r (rr + r r ) d2 r

r dr
2 =0 = r 2 r2 = 0.
2
r +r 2 (r + r ) 2 3/2
r + r 2
2 d2 d

But (ra r ) = ra r + ara1 r 2 , so with a = 2 the above equation becomes
 
d 1 dr
r = 1.
d r2 d

(b) Put w = 1/r, and this equation becomes w + w = 0, with the general solution
w = A cos + B sin . Hence the quoted result.
(c) Differentiate the definitions x = r cos , y = r sin with respect to the new indepen-
dent variable x to obtain
1 = r cos r sin and y = r sin + r cos ,
where the prime denotes differentiation with respect to x. Eliminate r by multiplying
the first equation by sin , the second by cos and subtracting, to give r2 = xy y.
Now eliminate by multiplying the first equation by cos , the second by sin and
adding, to give rr = x + yy , and hence
dr dr d (x + yy )r
= / = .
d dx dx xy y
Hence the functional becomes
Z b s Z b p
d (x + yy )2 r2 1 + y 2
S= dx r2 + 2
= dx (xy y) .
a dx (xy y) a |xy y|

Assuming that xy y > 0, that is > 0 this gives the result quoted.
(d) The stationary path in Cartesian coordinates is the straight line y = mx + c. The
boundary conditions give m = 1 + /(b a) and c = a/(b a), so
 
a
y = 1+ x .
ba ba

In polar coordinates the point x = y = a is r = a 2, = /4. At the other end
cos = b/r and sin = (b + )/r, so B = (b a)/a and A = (b a + )/a giving
a
r= .
(b a + ) cos (b a) sin
When = 0 this gives r = 0 unless cos = sin when r = a/ cos , that is x = a, which
is clearly incorrect. In this limit the straight line passes through the origin and the
problem is that polar coordinates are not defined here (r = 0, but is undefined).
15.6. SOLUTIONS FOR CHAPTER 6 513

Solution for Exercise 6.7


With the polar coordinates x = r cos , y = r sin the derivative y (x) is given by
equation 6.13 (page 178) and hence

r 2 + r2
1 + y (x)2 = 2.
(r cos r sin )

Since we are assuming that y (x) is bounded, from the definition of admissible functions,
r cos r sin 6= 0 on the curve. Here, for simplicity, we assume r cos r sin > 0:
in the opposite case the analysis changes slightly, but the final result is the same. The
functional becomes Z b
dx r r 2 + r2
S[r] = d ,
a d r cos r sin
where tan a = y(a)/a and tan b = y(b)/b. Using equation 6.12 this gives
Z b p
S[r] = d r r2 + r 2 .
a

F r 2 + 2r2 F rr
The derivatives of F = r r2 + r 2 are = and
= , giving
r r2 + r 2 r r2 + r 2
the Euler-Lagrange equation,

rr r 2 + 2r2
 
d
= 0.
d r2 + r 2 r2 + r 2
Expanding this gives

rr + r 2 rr (rr + r r ) 2r2 + r 2
= 0,
r2 + r 2 (r2 + r 2 )3/2 r2 + r 2
and this reduces to r3 r 3r2 r 2 2r4 = 0; division by r3 gives the quoted equation.
In order to simplify this, consider the first two derivatives of 1/r ,

r () d2 r () r ()2
   
d 1 1

= +1
and 2
= +1 + ( + 1) +2
d r r d r r r
and hence
r ()2 r+1 d2
 
1
r () = ( + 1) .
r d2 r
Thus if we set = 2 and substitute for r (), our equation becomes

r3 d2 d2 z
 
1 1
2r = 0 or + 4z = 0 where z = 2 .
2 d2 r2 d2 r

The general solution of the equation for z is z = r2 = A cos 2 + B sin 2, but since

2xy x2 y 2
sin 2 = 2 sin cos = and cos 2 = cos2 sin2 = ,
r2 r2
this becomes A(x2 y 2 ) + 2Bxy = 1.
514 CHAPTER 15. SOLUTIONS TO EXERCISES

Solution for Exercise 6.8


The functional is
Z B   Z B  
dx 1 1
S[x] = dy F = dy x (y)F ,
A dy dx/dy A x (y)

which is the required result.

Solution for Exercise 6.9


(a) The variable change gives
 
1 A(x) 2 dx
Z
S[y] = du y (u) + B .
2 dx/du du

dx dx
Z
By choosing = A(x), that is u(x) = , this functional becomes that quoted.
du A(x)
(b) The Euler-Lagrange equation for the original functional can be written in the form
y +A y /ABy /A = 0. Comparison with the given equation shows that A /A = 1/x,
hence A = 1/x, and also By /A = 4x2 y +8x2 , so that B(x, y) = 2xy 2 +8xy +g(x), where
g(x) is an arbitrary function of x; since g(x) does not contributeto the Euler-Lagrange
equation we set g(x) = 0. Then, from part (a) u = x2 /2, x = 2u and the functional
becomes
d2 y
 
1 2
Z
2
S[y] = du y (u) + 2y + 8y with Euler-Lagrange equation 4y = 8.
2 du2

Solution for Exercise 6.10


(a) The function is
B B
dx 1 1 1
Z Z
S[x] = dy = dy , x(A) = 1, x(B) = 2.
A dy x2 x (y)2 A x2 x (y)

(b) Putting F = 1/(x2 x ) we have

F 1 F 2
= and = 3 ,
x (xx )2 x x x

so the Euler-Lagrange equation is

2x
   
d 1 2 d 1 2
2
3 = 0, but = 3 ,
dy (xx ) x x dy (xx )2 2
x x 3 x x

which gives the required equation.


(c) Integrating once gives x2 x = c for some constant c. Integrating again gives
x(y)3 /3 = cy + d. The boundary conditions give 1/3 = cA + d and 8/3 = cB + d
7y + B 8A
and hence x3 = .
BA
15.6. SOLUTIONS FOR CHAPTER 6 515

Solution for Exercise 6.11


Applying the boundary conditions gives, for y1 (x), 0 = A+C and 1 = B+C cosh(/2)+
D sinh(/2), and, for y2 (x), 0 = A + C and 1 = B + C cosh(/2) + D sinh(/2).
Since A = C and A + C = 0, A = C = 0. Then the equations for B and D become
B + D sinh(/2) = 1 and B + D sinh(/2) = 1. Adding these two solutions gives
2D sinh(/2) = 0, so D = 0 and hence B = 1. Thus the required solution is y1 (x) =
sin x and y2 (x) = sin x.

Solution for Exercise 6.12


In this case F = y1 2 + y2 2 + y1 y2 so Fy1 = Fy2 = 0, Fy1 = 2y1 + y2 and Fy2 = 2y2 + y1 ,
d d
Hence the two Euler-Lagrange equations are (2y1 + y2 ) = 0 and (2y2 + y1 ) = 0.
dx dx
These may be integrated directly to give 2y1 + y2 = a1 and 2y2 + y1 = a2 , where
a1 and a2 are constants. Now integrate again to obtain 2y1 + y2 = a1 x + b1 and
2y2 + y1 = a2 x + b2 . But the boundary conditions at x = 0 give b1 = 1 and b2 = 2; at
x = 1 we have 4 = a1 + b1 and 5 = a2 + b2 . Hence a1 = 3 and a2 = 3 and the solutions
are 2y1 + y2 = 3x + 1 and 2y2 + y1 = 3x + 2. Multiplying the first equation by 2 and
subtracting from the second now gives y1 = x and y2 = x + 1.

Solution for Exercise 6.13 " 2 #


1
1 3 2
Z

We can write the functional in the form S[y1 , y2 ] = dx y1 + y2 + y2 . On
0 2 4
defining z1 = y1 + y2 /2 this becomes
1  
3 2 1
Z
2
S[z1 , y2 ] = dx z1 + y2 , z1 (0) = , z1 (1) = 2, y2 (0) = 1, y2 (1) = 1.
0 4 2

The associated Euler-Lagrange equations are simply z1 = 0 and y2 = 0 which can be


integrated directly to yield z1 = A1 x+B1 and y2 = A2 x+B2 . The boundary conditions
for z1 (x) give B1 = 1/2 and A1 + B1 = 2 so A1 = 3/2. For y2 (x) we find B2 = 1 and
A2 + B2 = 2, so A2 = 1. Hence the solution is
3 1 1
z1 = x+ and y2 = x + 1 and y1 (x) = z1 (x) y2 (x) = x.
2 2 2

Solution for Exercise 6.14


Pn Pn
(a) In this example F (y, y ) = yi Aij yj yi Bij yj giving

i=1 j=1

n n
F X dyj F X
=2 Akj and = 2 Bkj yj , 1 k n,
yk j=1
dx yk j=1

which gives the quoted Euler-Lagrange equations.

(b) Using the standard rules for matrix multiplication we see that
n X
X n n X
X n
y Ay = yi Aij yj , y By = yi Bij yj ,
i=1 j=1 i=1 j=1
516 CHAPTER 15. SOLUTIONS TO EXERCISES

which gives the required functional.


Similarly
n  2  n
X d y X d2 yj
(By)k = Bkj yj and A 2 = Akj 2 , 1 k n,
j=1
dx k j=1 dx

so the Euler-Lagrange equation is Ay + By = 0 and assuming that A is non-singular,


so that A1 exists, we obtain the given equation by multiplying by A1 .
Pn
(c) If y = k=1 ak (x)zk , equation 6.34 becomes
n  2 
X d ak
zk + ak A1 Bzk = 0.
dx2
k=1

But A1 Bzk = k2 zk , so on multiplying by z j for each j and using the fact that
2
zj zk = jk we obtain aj + j aj = 0, for j = 1, 2, , n.

Solution for Exercise 6.15


(a) If
b  
1 2
Z q
2
y12 + y22 ,

S[y1 , y2 ] = dt y + y2 V (r) , r=
a 2 1
F dV r r
we have = and r = yk , k = 1, 2. Hence the Euler-Lagrange equations
yk dr yk yk
are
d2 yk dV yk
+ = 0, k = 1, 2.
dt2 dr r
Z b  
1 2 1 2 2
(b) For the functional S[r, ] = dt r + r V (r) we have F = r2 and
a 2 2
F = 0 so the Euler-Lagrange equation for is
 
d 2 d L
r = 0 and hence = 2 ,
dt dt r

for some constant L. Also Fr = r and Fr = r 2 V (r) so the Euler-Lagrange


d2 r
equation for r is 2 + V (r) r 2 = 0. Substituting for = Lr2 gives the required
dt
equation.

Solution for Exercise 6.16


Differentiating the given expressions for y1 , y2 and y3 with respect to t gives

y1 = cos sin , y2 = sin + cos and y3 = z

so that y1 2 + y2 2 + y3 2 = 2 + 2 2 + z 2 . Hence the functional becomes


Z b  
1 2 2 2 2

S[, , z] = dt + +z V () .
a 2
15.6. SOLUTIONS FOR CHAPTER 6 517

The three Euler-Lagrange equations for , and z are, resectively


 2
d2 d
+ V () = 0,
dt2 dt
d2 z
 
d 2 d
= 0 and = 0.
dt dt dt2

The second of these equations gives = L2 , where L is a constant, and hence the
first becomes L2 3 + V () = 0. In this coordinate system the Euler-Lagrange
equations for and z are uncoupled.

Solution for Exercise 6.17


(a) Equation 6.43 can be written in the form y +(p /p)y +(q/p)y = b/a2 . Dividing 6.42

by a2 gives, by definition, the same equation; comparing these term by term Zgives p /p =
a1 /a2 and q/p = a0 /a2 . Integrating the first of these gives p(x) = exp dx a1 /a2
and q = a0 p/a2 .
(b) Here F (x, y, y ) = py 2 qy 2 + 2pb/a2, with derivatives Fy = 2py and Fy =
2qy + 2pb/a2 , so the Euler-Lagrange equation is
 
d dy bp
p + qy = .
dx dx a2

Solution for Exercise 6.18


(a) The derivatives required for the Euler-Lagrange equations are
F1 F1
= y, = f (x, y)
y y
x Z x
F2 F2
Z
= y du f (u, y), = y du fy (u, y)
y c2 y c2

Hence the Euler-Lagrange equation for F1 is y = f (x, y). For F2 the equation is
 Z x  Z x
d dy
du f (u, y) + y du fy (u, y) = 0
dx dx c2 c2

which simplifies to
Z x Z x
d2 y
 

f (x, y) + du y (x)fy (u, y) + y (x) du fy (u, y) = 0,
dx2 c2 c2

which gives the required equation.


(b) The difference is
Z y Z x
F1 F2 = dv f (x, v) + y du f (u, y).
c1 c2
518 CHAPTER 15. SOLUTIONS TO EXERCISES

Careful inspection of the right-hand side suggests considering the function


Z x Z y
g(x, y) = du dv f (u, v)
c2 c1

having the partial derivatives


Z y x
g g
Z
= dv f (x, v), = du f (u, y)
x c1 y c2

so that
y x
dg g g dg
Z Z
= + y = dv f (x, v) + y du f (u, y) and hence F1 F2 = .
dx x y c1 c2 dx

Solution for Exercise 6.19


In this example y = f (y, y ) = y y, so equation 6.47 for z is
z z z
(y + y) y + z = 0.
y y x
The equation u u = 0 has the solution u = ex which suggests transforming to
v(x, y, y ) where z = vex . The equation then becomes
v v v
(y + y) y = 0.
y y x
One solution of this is v = c = constant. Then the equation Fy y = z integrates to
1 2 x
F = cy e + y A(x, y) + B(x, y),
2
where A and B are functions of x and y only. The derivatives of F are

Fy = y Ay + By , Fyy = Ay , Fxy = cy ex + Ax ,

so the Euler-Lagrange equation 6.46 becomes

cy + cy + (Ax By ) ex = 0.

Setting c = 1 and (Ax By ) = yex yields the required equation. As in the case
treated in the text, there are several possible solutions of this equation. We choose
A = 0, B = 21 y 2 ex to give the functional

1
Z
dx y 2 y 2 ex .

S[y] =
2

Solution for Exercise 6.20


If x = z c ,
dy dy dz 1 dy
= = c1
dx dz dx cz dz
15.6. SOLUTIONS FOR CHAPTER 6 519

so the functional becomes


b  
1
Z
c1 2 2 2
S[y] = c dz z y y
a c2 z 2c2
b
y 2
 
1
Z
= dz c2 2 z c1 y 2
c a z c1
1/c 1/c
where a = a and b = b . The Euler-Lagrange equation for this functional is derived
from the relations
F 2y F
= , = 2c2 2 z c1 y
y z c1 y
and is
2y
 
d
+ 2c2 2 z c1 y = 0,
dz z c1
which expands to
1 d2 y c 1 dy
c + c2 2 z c1 y = 0.
z c1 dz 2 z dz
Multiply by z c to obtain the required equation.

Solution for Exercise 6.21


The functional depends only upon y1 and y2 so the Euler-Lagrange equations are
   
d F d F
= 0 and = 0.
dx y1 dx y2
Differentating these equations gives
2 F 2F 2 F 2 F
y1 + y2 = 0 and y + y = 0.
y1 2 y1 y2 y1 y2 1 y2 2 2
If d 6= 0 the only solution of these equations is y1 = 0 and y2 = 0 and integration gives
the two straight lines, y1 (x) = Ax + B and y2 (x) = Cx + D.
F F
If d = 0 we observe that since d is just the Jacobian determinant of y and y the
1 2
derivatives are functionally related, that is G(F/y1 , F/y2 ) = 0 for some function
G(u, v): since Fy1 and Fy2 are both functions only of y1 and y2 , this means that y1
and y2 are functionally related. Thus the Euler-Lagrange equation for y1 gives y1 = A,
for some constant A, and that for y2 gives y2 = B, where A and B are related. Thus
the solutions of the Euler-Lagrange equation are related straight lines, and are not
independent as in the case d 6= 0.
If the functional
 depends only upon one dependent variable, y(x), the Euler-Lagrange
2F

d F
equation is = 0 and the equivalent of the determinant is d = . If
dx y y 2
F F
=constant, that is F y , there is no stationary path and d = 0. If 6=constant,
y y
the stationary path is a straight line and d 6= 0.

Solution for Exercise 6.22


Observe that
d
= + y + y
dx x y1 1 y2 2
520 CHAPTER 15. SOLUTIONS TO EXERCISES

so that
Z b    d 
S2 [y1 , y2 ] = dx F x, y1 , y2 , y1 , y2 + ,
a dx
h ib
= S1 [y1 , y2 ] + (x, y1 , y2 ) .
x=a

The boundary term is independent of the path so the stationary paths of the two
functions, S1 and S2 are identical.

Solution for Exercise 6.23


Replace each of the terms gk (x)yk , k = 1, 2, in the original functional by

d  
gk (x)yk = gk (x)yk gk yk , k = 1, 2,
dx
to obtain S2 .

Solution for Exercise 6.24


(a) The required differentials of the integrand are

F ex F 1
= , = .
y 2 y ex y y 2 y ex y

The Euler-Lagrange equation is therefore

ex
 
d 1
+ = 0.
dx 2 y ex y 2 y ex y

On expanding this expression we obtain

ex y + ex y y ex
+ = 0.
4(y ex y )3/2 2 y ex y

Rearranging this gives y (3ex 1) y + 2e2x y = 0.

(b) If u = ex , so x = ln u, and Y (u) = y(x(u)) we have, using the chain rule

dy dY du dY dy dY
= = ex hence ex = = Y ,
dx du dx du dx du
and hence Z 1 p
S= du Y (u) + Y (u).
0

In this case F = Y + Y and F /Y = F /Y = (Y + Y )1/2 /2, and the Euler-
Lagrange equation is
 
d 1 1
= 0 or Y + 3Y + 2Y = 0.
du 2 Y + Y 2 Y +Y
15.6. SOLUTIONS FOR CHAPTER 6 521

(c) If x = ln u then
dy dY dx dY d2 y dY d2 Y
= = ex and 2
= ex + e2x 2 .
dx du du du dx du du
Substituting these relations into the equation y (3ex 1)y + 2ex y = 0 gives
e2x Y (u) + ex Y (u) + 3ex 1 ex Y (u) + 2e2x Y = 0,


which reduces to Y + 3Y + 2Y = 0.

Solution for Exercise 6.25


If F = y 2 + z 2 + 2yz then Fy = 2y , Fy = 2z and Fz = 2z , Fz = 2y, so the
d2 y d2 z
Euler-Lagrange equations are 2
z = 0 and y = 0. Putting z = y into
dx dx2
the second equation gives d4 y/dx4 y = 0. Now put y = exp(x), where and
are constants, to obtain 4 = 1, that is = 1 and i. Thus the general solution is
y = A cos x + B sin x + C cosh x + D sinh x. The boundary conditions at x = 0 give
A + C = 0 and C A = 0 and hence A = C = 0: the boundary conditions at x = /2
then give 3/2 = B + D sinh(/2) and 1/2 = B + D sinh(/2). Subracting these
equations gives B = 1/2 and adding gives 2D sinh(/2) = 2. Hence
sinh x 1 sinh x 1
y= + sin x and z = sin x.
sinh(/2) 2 sinh(/2) 2

Solution for Exercise 6.26


If y(x) is a stationary path and y(x) + g(x) a varied path the standard analysis gives
Z X
n2
   

F [y, h] = 2 dx xy (x)h (x) x y(x)h(x)
0 x
and integration by parts gives
X
n2
   
d
Z
X
F = 2 xy (x)h(x) 0 2

dx h(x) (xy ) + x y(x) .
0 dx x
The boundary term vanishes at x = 0 because y (x) is finite here, and at x = X because
h(X) = 0. Hence the Euler-Lagrange equation is
n2 d2 y
 
d dy

y(x) or x2 2 + x + x2 n2 y = 0.

(xy ) + x
dx x dx dx
(a) If x = f (u) then the chain rule gives
dy dy du w (u)
= =
dx du dx f (u)
and the variational principle becomes
Z u1
n2
   
f (u) 2 2
F [w] = du f (u) w (u) f (u) w(u)
u f (u)2 f (u)
Z u0 1
f (u) 2
   
f (u) 2 2
= du w (u) f (u)f (u) n w(u) ,
u0 f (u) f (u)
where f (u0 ) = 0 and f (u1 ) = X.
522 CHAPTER 15. SOLUTIONS TO EXERCISES

(b) If f = eu this functional becomes


Z u1
du w (u)2 e2u n2 w(u)2 .
  
F [w] =
1

The Euler-Lagrange equation for this functional is w (u) + e2u n2 w = 0, with the


solution w(u) = Jn (eu ).


15.7. SOLUTIONS FOR CHAPTER 7 523

15.7 Solutions for chapter 7


Solution for Exercise 7.1
All the given transformations depend continuously upon only one parameter, . Trans-
formations (a) and (c) reduce to the identity when = 0, but (b) gives (x, y) = (x, y).
Hence (a) and (c) define a one-parameter family of transformations.

Solution for Exercise 7.2


(a) Separating variables puts the equation in the form
y t
1
Z Z
dy = dt, z = y(0),
z y(1 y) 0

then integrating gives


y
zet
 
y
ln =t and rearranging this gives y = (z, t) = .
1y z 1 + (et 1) z

(b) There is one parameter, t, and at t = 0, y = z, because this is the initial condition.

Solution for Exercise 7.3


In this case "
d 2  2 #
dy 1 dy 2
Z
G= dx
c dx dx
But
   
dy 1 dy1 dy2 dx dy 2 dy1 dy2 dx
= cosh + sinh and = sinh + cosh
dx dx dx dx dx dx dx dx

so that  2  2  2 " 2  2 #
dy 1 dy 2 dx dy1 dy2
= .
dx dx dx dx dx
Hence
d  2 " 2  2 #
dx dx dy1 dy2
Z
G = dx
c dx dx dx dx
" 2 2 #
d 
1 dy1 dy2
Z
= dx .
c 1 + g (x) dx dx

Hence G = G only if g (x) = 0, that is if g(x) is a constant.

Solution for Exercise 7.4


The Taylor expansions of the trigonometric functions give cos = 1+O(2 ) and sin =
+ O(3 ). Hence, to O() we have y 1 = y1 y2 , y2 = y1 + y2 which can be written
in the form y 1 = y1 + 1 , y 2 = y2 + 2 , where 1 = y2 and 2 = y1 .
524 CHAPTER 15. SOLUTIONS TO EXERCISES

Solution for Exercise 7.5


In this case = 0, 1 = y2 and 2 = y1 so equation 7.15 becomes

F F
y2 + y1 = 2y1 y2 2y2 y1 = constant.
y1 y2

Solution for Exercise 7.6


Consider the general transformation

y 1 = y1 + g1 (x), y 2 = y2 + g2 (x), x = x + g3 (x)

so that    
dy 1 dy1 dx dy 2 dy2 dx
= + g1 (x) , = + g2 (x) ,
dx dx dx dx dx dx
and the functional 2 2 !
d  
dy 1 dy 2
Z
G= dx +
c dx dx
becomes, to first-order in ,
" 2 2 #
d  
1 dy1 dy2 dy1 dy2
Z

G= dx + + 2 g1 (x) + g2 (x) .
c 1 + g3 (x) dx dx dx dx

Now consider the three specific cases:


Rd
(i) If g1 = g and g2 = g3 = 0, then G = G + 2 c dx g (x)y1 (x).
Rd
(ii) If g2 = g and g1 = g3 = 0, then G = G + 2 c
dx g (x)y2 (x).

(iii) If g3 = g and g1 = g2 = 0, then


" 2 2 #
d 
1 dy1 dy2
Z
G= dx + .
c 1 + g (x) dx dx

In all cases G = G, for all admissible y1 and y2 , only if g (x) = 0, that is if g(x) is a
constant, which we set to unity in the following. The resulting first-integrals are:
F
(i) 1 = 1, 2 = = 0, hence =c giving y1 = c,
y1
F
(ii) 2 = 1, 1 = = 0, hence =c giving y2 = c,
y2
F F
(iii) = 1, 1 = 2 = 0, hence F y1 y2 = c giving y1 2 + y2 2 = c.
y1 y2
where c is a constant.
In this example the first-integral arising from the invariance with respect to trans-
lations in x can also be derived from the two first-integrals due to the invariance under
the translations in y1 and y2 . Thus not all symmetries lead to new first-integrals.
15.7. SOLUTIONS FOR CHAPTER 7 525

Solution for Exercise 7.7


In this case, since x = x we have
Z d "  2  2 ! #
1 dy 1 dy 2
G= dx + + V (y 1 y 2 ) .
c 2 dx dx

Replacing yk by yk and expanding to first-order in gives


Z d "  2  2 !   #
1 dy1 dy2 dy1 dy2
G = dx + + g (x) + + V (y1 y2 ) ,
c 2 dx dx dx dx
Z d  
dy1 dy2
= G+ dx + g (x).
c dx dx

Thus G = G only if g =constant. In this case equation 7.15 becomes, on setting g = 1


and since 1 = 2 = 1, Fy1 + Fy2 = constant, that is y1 + y2 = constant.

Solution for Exercise 7.8


(a) The Euler-Lagrange equation is

d2 y
 
d 2 dy 2 dy
x + x2 y 5 = 0 which expands to 2
+ + y 5 = 0.
dx dx dx x dx

(b) When expressed in terms of x and y the functional is


2 !
b 
dy 1
Z
2
S= dx x y6 .
a dx 3

Now change variables, x = x, y = y,


b
2 2 6 6
Z  
3 2
S = dx x y (x) y
a 2 3
Z b
2 4 6
 
2 2 2
= dx x y (x) y = S.
a 3

If 2 = 1 the functional is invariant, see the definition 7.1 (page 201).


Now set = 1 + , and work to first-order in , so = 1 /2 and, from equation 7.13,
= x, = y/2. The first-integral, equation 7.15 (page 203) becomes
 
2 2 2 1 2 6 2 2
yx y + x x y x y 2x y = c
3

where c is a constant. This becomes


 
1
x2 yy + x3 y 2 + y 6 = c.
3
526 CHAPTER 15. SOLUTIONS TO EXERCISES

(c) The substitution y = Ax gives


1
A2 x2+1 + A2 2 x2+1 + A6 x3(2+1) = c,
3
hence setting = 1/2 and c = A2 (4A4 3)/12, gives the solution y = Ax1/2 .
The same substitution into the Euler-Lagrange equation gives
A( + 1)x2 + A5 x5 = 0,

so with 2 = 5 and A( + 1) + A5 = 0, that is = 1/2 and A = 1/ 2, this
gives two particular, real solutions of the Euler-Lagrange equation.
(d) The first-integral is
1
x3 y 2 + x2 yy + x3 y 6 = c.
3
Differentating with respect to x gives, after a division by x,
y 2x2 y + xy + 4xy 2 + xy 6 + 2yy 1 + x2 y 4 = 0.
 

This can be written in the form


 2y
y 2x2 y + xy + 2x2 y + xy 2yy

x
+y 5 2x2 y + xy 2x2 y 5 y


+2yy 1 + x2 y 4

= 0.
That is  
2 5

2x2 y + xy = 0.

y + y +y
x

The solutions of this equation are the solutions of 2x2 y + xy = 0, that is y = A/ x, A
being a constant, and any solution of the original Euler-Lagrange equation.

Solution for Exercise 7.9


Expand the expression immediately before equation 7.19 to first-order in ; since G is
independent of we have
Z d
d
0 = dx F (x, y, y )
c dx
Z d n    !
F X F dk d F
+ dx + k + yk
c x yk dx dx yk
k=1
n n 
Z d ! 
X F d F X F F k
= dx F y + + k + (15.3)
c yk k dx x yk yk x
k=1 k=1

which is just equation 7.19. But integration by parts gives


Z d n
! " n
! #d
X F d X F
dx F y = F y
c yk k dx yk k
k=1 k=1 c
n
Z d !
d X F
dx F y
c dx yk k
k=1
15.7. SOLUTIONS FOR CHAPTER 7 527

and
n
" n
#d n
d d  
F dk F d F
Z X X Z X
dx = k dx k
c yk dx yk c dx yk
k=1 k=1 c k=1
" n
#d n
d
F F
X Z X
= k dx k
yk c yk
k=1 c k=1

where we have used the Euler-Lagrange equation to derive the last line.
Hence, equation 15.3, becomes
" n
! n
#d
X F X F
0 = F y + k
yk k yk
k=1 k=1 c
n
!!
d
F d F
Z X
+ dx F y .
c x dx yk k
k=1

The term in brackets inside the integral is, on expanding the total derivative
n  
F F X F F F F
yk + yk yk yk = 0,
x x yk yk yk yk
k=1

where we have used the Euler-Lagrange equations again.

Solution for Exercise 7.10


In this example the integrand is invariant under translations in y, and in equation 7.13
n = 1, = 0 and = 1 = 1, so the invariant, equation 7.15 becomes Fy = constant.
The Euler-Lagrange equation is, since Fy = 0,
 
d F F

= 0 giving = constant.
dx y y

Solution for Exercise 7.11


(a) Since F = xyy 2 , Fy = xy 2 and Fy = 2xyy and the Euler-Lagrange equation is
2
d2 y

d dy dy
(2xyy ) xy 2 = 0 or 2xy 2 + x + 2y = 0.
dx dx dx dx

(b) The integrand is homogeneous of degree zero in x, so the transformation to the new
independent variable x, where x = x, with a constant leaves the integral invariant.
To use Noethers theorem we set = 1 + , so (x) = x and = 0, see equation 7.13.
Then, equation 7.15 becomes
 
F
F y x = c where F = xyy 2 ,
y
which gives the required result.
528 CHAPTER 15. SOLUTIONS TO EXERCISES

Solution for Exercise 7.12


(a) On putting x = x and y = y we obtain
 2  2
dy dy
Z Z
S[y] = dx x3 y 2 = 2 4 dx x3 y 2 = 2 4 S[y].
dx dx

The functional is invariant if 2 = 1.


Put x = (1 + )x, so to first-order y = (1 /2)y and by comparison with equation 7.13
we see that = x and = y/2, and the first-integral, equation 7.15, becomes

1
yFy + x (F y Fy ) = constant where F = x3 y 2 y 2 .
2
Substituting for F gives

x3 y 3 y + x4 y 2 y 2 = c = constant.

(b) Putting y = Ax , so y = Ax1 , into the first-integral gives

A4 x4+2 + A4 2 x4+2 = c,

which is satisfied only if 4 + 2 = 0, that is = 21 , and c = A4 ( + 1) = 14 A4 .


The Euler-Lagrange equation is

d
x3 y 2 y 2x3 yy 2 = 0.

2
dx
Differentiation gives
1 3
x3 y 2 y + x3 yy 2 + 3x2 y 2 y = 0 or y + y 2 + y = 0.
y x

Substituting y = Ax into this equation yields

A( 1)x2 + A 2 x2 + 3Ax2 = 0 or 2A( + 1)x2 = 0.

This equation is satisfied only if = 1.

(c) The Euler-Lagrange equation can be written in the form

d 3
(yy ) + (yy ) = 0,
dx x
which integrates directly to yy = Ax3 , where A is a constant. A further integration
gives
1 2 A B A
y = 2 + that is y 2 + 2 = B,
2 2x 2 x
where B is a constant. This gives the general solution of the Euler-Lagrange equation.
Further, if yy = Ax3 the first-integral reduces to Ay 2 + A2 x2 = c.
15.7. SOLUTIONS FOR CHAPTER 7 529

dy dy 1
(d) If x = ua then = and the functional becomes
dx du aua1
 2
1 dy
Z
S[y] = du u2a+1 y 2 .
a du
Z
2
With a = 1/2 this simplifies to the equivalent functional S[y] = du (yy ) . The
integrand, F = (yy )2 , is independent of u, so the first-integral is y Fy F = (yy )2 = c2 ,
where c is a constant. Hence yy = c, which integrates to 12 y 2 = cu+b for some constant
b: because u = x2 this is the same solution as derived in part (c). On the other hand,
the function y = Ax1/2 from part (b) becomes Au1/4 which is not a solution of the
equation yy (u) = c.

Solution for Exercise 7.13


In the first case if F/y = 0 the differential equation derived in exercise 4.33 (page 143)
can be written as
     
d d F F d F F
= 0 and hence = constant.
dx dx y y dx y y
In the second case, F/x = 0 it is easiest to use a version of the analysis described
in section 7.2.1. Equation 7.3 becomes
Z d
G = du F (y(u), y (u), y (u))
c
Z d Z c Z d
= du F (y, y , y ) + du F (y, y , y ) du F (y, y , y )
c c d

On putting u = v + and expanding to first-order in we obtain


Z d  
F F F d

F (y, y , y ) c + O( 2 ).

G= dv F (y, y , y ) + y + y + y
c y y y
(15.4)
Because G is invariant under translations this gives
Z d  
F F F d
F (y, y , y ) c + O(),

0= dv y + y + y
c y y y
which is the equivalent of equation 7.4. Now integrate by parts
Z d  d Z d  
F F d F
dv y = y dv y
c y y c c dv y
Z d   d Z d 2
 
F F d F d F
dv y = y y + dv y 2 .
c y y dx y c c dv y
Thus equation 15.4 becomes
d Z d
d2
       
F d F F F d F F
y y + y F + dv y
+ = O().
y dx y y c c y dv y dv 2 y
530 CHAPTER 15. SOLUTIONS TO EXERCISES

But the integrand is zero and the end points c and d are arbitrary and hence
   
F d F F
y y F = constant.
y dx y y
15.8. SOLUTIONS FOR CHAPTER 8 531

15.8 Solutions for chapter 8


Solution for Exercise 8.1
(a) In this case f = 3x2 and f = 6x, so f (0) = f (0) = f (0) = 0. The function is
negative for x < 0 and positive for x > 0, so is stationary but has no extreme value at
x = 0. If f (x) = x3 + x, then f = 3x2 + , which has no real zeros if > 0, but two
if < 0. At these zeros f 6= 0: the negative zero is a local maximum and the positive
zero is a local minimum.
(b) Since F (a) = g (a) 6= 0, F (x) is not stationary at x = a. In the neighbourhood of
x = a put x = a + z and put G(z) = F (a + z) and expand in a Taylor series,
1 1  
G(z) = f (a) + g (a)z + g (a)z 2 + f (a) + g (a) z 3 + O(z 4 )
2 6
and
1  
G (z) = g (a) + g (a)z + f (a) + g (a) z 2 + O(z 3 ).
2
The stationary points near x = a are given by the solutions of G (z) = 0. If we ignore
the terms O(z 2 ) this gives z = g (a)/g (a), which is O(1), and here we cannot expect
the second-order Taylor series to be accurate. But the equation
1  
g (a) + g (a)z + f (a) + g (a) z 2 = 0
2
has the solution
2g (a)
z2 = + O(2 ),
f (a)

that is z = O( ), and if is sufficiently small the term of O(z 3 ) in the expansion
of G(z) is negligible. Hence if g (a)/f (a) < 0 there are two real stationary points,
otherwise there are none.

Solution for Exercise 8.2



2 0
(a) If f = x21 x22 then det(H) = = 4.
0 2

2 0
(b) If f = x21 + x22 then det(H) = = 4.
0 2

2 2
2 0
(c) If f = x1 x2 then det(H) = = 4.
0 2

Solution for Exercise 8.3


Since fx = 3x2 3y 2 and fy = 6xy
there is a stationary point at the origin. The
6x 6y
Hessian is det(H) = which is zero at the origin. This stationary point
6y 6x
is therefore degenerate.
532 CHAPTER 15. SOLUTIONS TO EXERCISES

Solution for Exercise 8.4


(a) Write f = (y 4x)2 14x2 , so the stationary point at the origin is a saddle.
(b) For f = 2x2 + 4y 2 + z 2 + 2(xy + yz + xz) we have
fx = 4x + 2y + 2z, fxx = 4, fxy = 2, fxz = 2,
fy = 8y + 2x + 2z, fyy = 8, fyz = 2,
fz = 2z + 2y + 2x, fzz = 2,
and the Hessian determinant at the origin and its descending minors are

4 2 2
4 2
det(H) = D3 = 2 8 2 = 24 giving D2 =
= 28 and D1 = 4.
2 2 2 2 8

Hence 2 is positive definite and the stationary point is a minimum. The eigenvalues
of H are (0.689, 3.58, 9.73), which are all positive.

Solution for Exercise 8.5


Consider the function f (x, y) = x2 + y n , n 3, so fx = fy = 0 at the origin and
fxy = 0 everywhere. Since fxx = 2 and fyy = n(n 1)y n2 the Hessian determinant is
zero at the origin and 2 = 2x2 , so is independent of y. Hence the stationary point is
degenerate.
In these circumstances the nature of the stationary point is determined by the
higher-order terms in the expansion. In this case the function is simple because it is
separable. On the x-axis (y = 0), f (x, 0) = x2 and it has a minimum; on the y-axis,
f (0, y) = y n , which is a minimum if n is even, but for odd n it is a saddle. Hence
x2 + y 4 has a minimum at the origin and x2 + y 3 has a saddle at the origin.

Solution for Exercise 8.6


The Hessian determinant is

f fxy
det(H) = D2 = xx 2
= fxx fyy fxy ,
fxy fyy

and D1 = fxx . If D2 > 0 then fxx fyy > 0 and fxx and fyy must have the same sign.
For a minimum the quadratic form must be positive definite, which gives the first
set of conditions. For a maximum the quadratic form must be negative definite, which
gives the second set of conditions.
If D2 < 0 we proceed as follows. Suppose the stationary point is at (a, b), define
u = x a, v = y b, so the quadratic form is 2 = u2 fxx + 2uvfxy + v 2 fyy and 2 = 0
when
u fxy D2
=
v fxx
Hence when D2 < 0 there are two lines through the stationary point, given by u =
fxy D2 v on which the quadratic form is zero and it has different signs either
side of these lines. Near the stationary point the cubic terms in the expansion of (x, y)
can be neglected, so it behaves like 2 and the stationary point is a saddle.
15.8. SOLUTIONS FOR CHAPTER 8 533

Solution for Exercise 8.7


(a) For the function f = (x3 + y 3 ) 3(x2 + y 2 + 2xy) we have fx = 3x2 6(x + y) and
fy = 3y 2 6(x + y). Clearly there is a stationary point at the origin. But also fx = 0
if x + y = x2 /2 which on substituting into fy = 0 gives y 2 = x2 , so y = x, giving
another stationary point at (4, 4).
Near the origin the second-order terms are f = 3(x+y)2 < 0 except if x+y = 0; along
this line the function is zero. The Hessian is zero at the origin so this is a degenerate
stationary point.
The second derivatives are fxx = 6(x 1), fxy = 6 and fyy = 6(y 1) and putting
x = 4 + u, y = 4 + v gives f + 64 = 9u2 6uv + 9v 2 = (3u v)2 + 8v 2 , which is positive
unless u = v = 0. This stationary point is therefore a minimum.
(b) For f (x, y) = x4 + 64y 4 2(x + 8y)2 the first derivatives are
fx = 4x(x2 1) 32y, fy = 32 8y(y 2 1) x .
 

An obvious solution of the equations fx = fy = 0 is x = y = 0. The equation fy = 0


also gives x = 8y(y 2 1) which, when substituted into the other equation, gives
5
fx = 32y(y 2 1) 64y 2 (y 2 1)2 1 32y = 0 64(y 2 1)3 = 1 y2 =
 
= = .
4

Hence (x, y) = ( 5, 5/2). The second derivatives are
fxx = 4(3x2 1), fyy = 256(3y 2 1) and fxy = 32.
Consider each stationary point in turn.

The origin. Near here f 2(x + 8y)2 so f (0, 0) > f (x, y) except along the line
x + 8y = 0, on which f (8y, y) = 64 65y 4 , so this stationary point is a saddle. Also,
H(0, 0) = 0, and the stationary point is degenerate.

The point ( 5, 5/2). The second derivatives have the value fxx = 56, fxy = 32
and fyy = 704 and the Hessian determinant is
 
56 32
H= giving det(H) = D2 = 38400, D1 = 56,
32 704
so this stationary is a minimum.

The point ( 5, 5/2). The second derivatives and the Hessian have the same values
as at the point ( 5, 5/2) so this stationary is also a minimum.

Solution for Exercise 8.8


Differentiating with respect to a and b gives
N N N
!
X X X
= 2 (a + bxi yi ) = 2 N a + b xi yi ,
a i=1 i=1 i=1
N N N N
!
X X X X
= 2 xi (a + bxi yi ) = 2 a xi + b x2i xi yi .
b i=1 i=1 i=1 i=1
534 CHAPTER 15. SOLUTIONS TO EXERCISES

Setting these expressions to zero gives the quoted equations.


We now need to show that this is a minimum. The second derivatives are
N
X N
X
aa = 2N, bb = 2 x2k , and ab = 2 xk ,
k=1 k=1

and the Hessian determinant is


N N
!2
X X
det(H) = D2 = 4N x2k 4 xk .
k=1 k=1

Now use the Schwarz inequality, page 41, with bi = 1 for all i, to see that det(H) 0
(with equality only if all the xi values are the same) and since aa = 2N > 0 we see
that the stationary point is a minimum.

Solution for Exercise 8.9 Z


b Z 1
Put x = a + (b a)y so dx f (x) = (b a) dy f (a + (b a)y) and, since
a 0
g(a + (b a)y) = (b a)2 y(1 y)
Z b 1
1
Z
2 5
dx g(x) = (b a) dy y 2 (1 y)2 = (b a)5 ,
a 0 30
b 1
1
Z Z
dx g (x)2 = (b a)3 dy (1 2y)2 = (b a)3 .
a 0 3
2
Hence I < I1 if (b a) < 10.

Solution for Exercise 8.10


Without a bound on |h (x)| admissible functions for which h (x) = O(1/) exist, as
shown in section 4.6, so 2 2 = O(1).

Solution for Exercise 8.11


(a) Jacobis equation is u = 0 (Q = 0) or u u = 0 (Q = 1). The solutions are,
respectively, u = x a and u = sinh(x a), neither of which is zero for any x > a.
(b) Jacobis equation is u + u = 0 with solution u = sin(x a), which is zero at
x = a + , so the conjugate point is a = a + .

Solution for Exercise 8.12


The Euler-Lagrange equation is y + y = 0, y(0) = 0, y(X) = 1, with solution
y = sin x/ sin X. Also, P (x) = 1, Q(x) = 1, so the associated Jacobi equation is
u + u = 0, which is the equation dealt with in exercise 8.11 (b), from which we see
that the conjugate point is x = . Thus if X < the solution yields a local min-
imum; otherwise it does not. Further, at the boundary X = the solution to the
Euler-Lagrange equation does not exist.

Solution for Exercise 8.13


For this functional Q = 0 and P = Fy y and the second variation is
Z b
2 [y, h] = dx P (x)h 2 .
a
15.8. SOLUTIONS FOR CHAPTER 8 535

If P (x) 6= 0 for a x b, 2 has the same sign as P for all h, and the stationary
path is either a minimum (P (x) > 0)or a maximum (P (x) < 0), that is there are no
conjugate points.
Alternatively, Jacobis equation is (P u ) = 0, with solution
Z x
dv
u(x) = P (a) 6= 0 for x > a,
a P (v)

provided P (x) does not change sign, at which point the integral may cease to exist.

Solution for Exercise 8.14


Multiply Jacobis equation 8.13 by u and integrate,
Z b     h ib Z b
d du  
dx u P Qu = uu P dx P u 2 + Qu2 .
a dx dx a a

But, by definition the left-hand side is zero and since u(a) = u(b) = 0, 2 [y, u] = 0.
Hence, from equation 8.7, we see that for h = u, S = O(3 ) and the nature of the
stationary point cannot be determined by this order of expansion.

Solution for Exercise 8.15


dw w2
Equation 8.15 is, provided P (x) 6= 0, = Q(x) + and from equation 8.18
dx P (x)

dw u u 2 u
= P + 2 P P ,
dx u u u
which gives

u u
 
d du
P + P Q = 0 and hence P Qu = 0.
u u dx dx

Solution for Exercise 8.16


In this example, P = 1 and Q = 1, so equation 8.21 becomes
Z X n o Z X
h 2 h2 + (1 )h 2 = dx h 2 h2 ,
 
J [h] = dx
0 0

and the associated Euler-Lagrange


equation
is h + h = 0. The general solution of
this is
h(x) = A cos(x ) + B sin(x ) and the initial conditions give A = 0 and
B = 1/ , so
1
h(x, ) = sin(x ) and lim h(x, ) = x.
0

Thus the lines defined by h(x, ) = 0 are, for 6= 0, given by x = k for integer k.

Solution for Exercise 8.17


The equation derived in the text is
d
(Fy y h + Fyy h) Fyy h Fyy h = O()
dx
536 CHAPTER 15. SOLUTIONS TO EXERCISES

which expands to
   
d dh d
Fy y Fyy Fyy h = O().
dx dx dx
Take the limit 0 to obtain the required result.

Solution for Exercise 8.18


(a) Consider two neighbouring values of the constant, c and c + so that

y(x, c + ) = y(x, c) + y(x, c) + O( 2 )
c
and since, by definition, y(a, c) = y(a, c + ) = A it follows that yc (a, c) = 0. By
comparing the above equation with the relation z(x) = y(x) + h(x) used in the text
to derive equation 8.25, and by associating yc (x, c) with h(x), we see that yc (x, c)
satisfies the equation
 
d dyc
P (x) Q(x)yc = 0, yc (a, c) = 0.
dx dx

(b) The associated Euler-Lagrange equation is y + y = 0 and the general solution


satisfying the boundary condition at x = 0 is y = c sin x, so that yc = sin x.
For this problem equation 8.25 is, since P = 1 and Q = 1, h + h = 0, h(0) = 0,
which is clearly satisfied by yc = sin x.

Solution for Exercise 8.19


If b a is sufficiently small then, on ignoring terms O(b a),

P (x) = P (a) + O(x a) > 0 and Q(x) = Q(a) + O(x a)

and Jacobis equation becomes u + u = 0 with = Q(a)/P (a). The Taylor expan-
sion of the solution is
1
u(x) = u(a) + (x a)u (a) + (x a)2 u (a) + O((x a)3 ).
2
But by definition, u(a) = 0 and u (a) = 1, so from the Jacobis equation u (a) = 0
and the approximate solution is u(x) = (x a) + O((x a)3 ), which is positive for
sufficiently small b a.

Solution for Exercise 8.20


On the stationary path
1 2 2 sin cos 1
x= c (2 sin 2) and z = c2 sin2 so z (x) = = .
2 1 cos 2 tan
Also
z 1 z 2 1
Fz = p , Fz z = p 2 3/2
=
z(1 + z 2 )
z(1 + z )2 z(1 + z ) z(1 + z 2 )3/2
15.8. SOLUTIONS FOR CHAPTER 8 537

hence P = Fz z = (1/c) sin2 . In addition



1 + z 2 3 p 3
Fz = , Fzz = 1 + z 2 = 5 6 ,
2z 3/2 4z 5/2 4c sin

and
z cos
Fzz = = 3 3 .
2z 3/2 1+z 2 2c sin
These expressions can be used to find Q,

z
 
3 p 2
1d
Q = 1+z +
4z 5/2 2 dx z 3/2 1 + z 2
 
3 1 d cos 1
= +
4c5 sin6 2c3 d sin3 2c2 sin2
3 cos2
 
3 1 1 1
= 5 6 + 5 2 2 4 = 5 4 .
4c sin 4c sin sin sin 2c sin

The Jacobi equation is


 
d du df df d 1 df
P Qu = 0 and since = = 2 2
dx dx dx d dx 2c sin d

this becomes
d2 u
sin2 2u = 0.
d2
Substituting the given function into this equation shows that the general solution is
 
A cos
u() = +B 1 ,
tan sin

where A and B are constants. But u(0) = 0, so A = 0, to give


   
cos cos
u() = B 1 and u () = B .
sin sin2 sin

cos
But, for > 0, 2 > sin 2 giving > sin cos and > , showing that for
sin2 sin
0 < < , u() is an increasing function and there are no conjugate points.

Solution for Exercise 8.21


Using the result of exercise 3.4 (page 97) we have
p p y h
1 + (y + h )2 1 + y 2 p
1 + y 2

so that
b
y f (x)
Z
S[y + h] S[y] dx p h.
a 1 + y 2
538 CHAPTER 15. SOLUTIONS TO EXERCISES

But the Euler-Lagrange equation is


!
d y f (x) y f (x)
p = 0 that is p = d = constant
dx 1 + y 2 1 + y 2

and hence Z b h i
S[y + h] S[y] d dx h (x) = d h(b) h(a) = 0.
a

Solution for Exercise 8.22


(a) P = 6y , Q = 0 and, since the solution of the Euler-Lagrange equation is y (x) = m,
Z 1
m a constant, 2 [y, h] = m dx h 2 .
0
2
(b) P = 4(3y 1), Q = 0 and, since the solution of the Euler-Lagrange equation is
Z 1
2

y (x) = m, m a constant, 2 [y, h] = 4(3m 1) dx h 2 .
0
2 2 2 2 2
(c) P = 4y (3y 1), Q = 2(y 1) 8(yy (y 1)) and

Z 1
dx P (x)h 2 Q(x)h2 .

2 [y, h] =
0

Z 1 
(d) Since S[y + h] = exp dx F (x, y + h ) on differentiation with respect to
0
we obtain
1
d
Z
S[y + h] = S[y + h] dx h Fy (x, y + h ).
d 0
Putting = 0 gives the Gateaux differential,
Z 1
S[y, h] = S[y] dx h Fy (x, y ).
0

Now differentiate again,


1 2
d2
Z

S[y + h] = S[y + h] dx h Fy (x, y + h )
d2 0
Z 1
+S[y + h] dx h 2 Fy y (x, y + h ).
0

Putting = 0 gives
(Z
1 2 Z 1
)
2
2 S[y, h] = S[y] dx h Fy (x, y ) + dx h Fy y (x, y )
0 0

On a stationary path S[y, h] = 0 and since S[y] 6= 0 we must have


Z 1
dx h Fy (x, y ) = 0 so that Fy (x, y ) = constant.
0
15.8. SOLUTIONS FOR CHAPTER 8 539

Hence Z 1
2 S[y, h] = S[y] dx h 2 Fy y (x, y ).
0

Solution for Exercise 8.23


(a) Expanding the integrand gives
1
F (x, y + h) = F (x, y) + Fy + 2 h2 Fyy + O(3 )
2
The functional is stationary if Fy (x, y) = 0 and then
Z b
2 [y, h] = dx Fyy (x, y)h2 .
a

(b) The integrand of the second variation is given by the second-order term in the
Taylor expansion of F (x, y + h, y + h , y + h ). This is
1
Fyy h2 + Fy y h 2 + Fy y h 2 + Fyy hh + Fy y h h + Fyy hh .

F2 =
2
Hence
Z b
dx Fyy h2 + Fy y h 2 + Fy y h 2

2 [y, h] =
a
Z b
+2 dx (Fyy hh + Fy y h h + Fyy hh ) .
a

But, from section 8.3, assuming that h(a) = h(b) = 0,


Z b
1 b d
Z

dx Fyy hh =
dx h2 (Fyy )
a 2 a dx
and similarly if h (a) = h (b) = 0, that is, if y (x) is given at x = a and x = b,
Z b
1 b d
Z
dx Fy y h h = dx h 2 (Fy y )
a 2 a dx
Finally, we have
Z b b
d
Z
b
dx Fyy hh = [Fyy hh ]a (hFyy ) dx h
a a dx
Z b Z b
d
= dx hh (Fyy ) dx h 2 Fyy
a dx a
Z b  
1 2 d 2
= dx h Fyy h Fyy .
a 2 dx2
Hence Z b
dx h2 P0 (x) + h 2 P1 (x) + h 2 P2 (x)

2 =
a
540 CHAPTER 15. SOLUTIONS TO EXERCISES

where
dFyy d2 Fyy
P0 (x) = Fyy +
dx dx2
dFy y
P1 (x) = Fy y 2Fyy and P2 (x) = Fy y .
dx

Solution for Exercise 8.24


In this example
Z b
S[y + h] = S[y] + dx B(x)h (x)
a

and there is no term O(2 ).

Solution for Exercise 8.25


(a) First note that S[y] 0 for all y and its smallest possible value is 0.
Since F = y 2 (y 2 1)2 the integrand is independent of x, so the first-integral exists and
is
y Fy F = y 2 y 2 1 3y 2 + 1 = c
 

where c is a constant. If c = 0 then either y(x) = 0 (which does not satisfy the boundary
condition at x = 1) or y = x.
Thus the stationary path is y = x and on this path S[y] = 0, so it gives a global
minimum.
Further
d
P (x) = Fy y = 4y 2 (3y 2 1) = 8x2 and Q(x) = 2(y 2 1)2 8 yy (y 2 1) = 0

dx
Z 1
giving 2 [y, h] = 8 dx x2 h 2 > 0.
0

(b) If y(1) = A 6= 1 this becomes a much more difficult problem, since y = x is not a
solution and near x = 0 the stationary path is quite different from this, no matter how
small A 1.
The first-integral, y 2 (y 2 1)(3y 2 + 1) = c, shows that if y(0) = 0 then either,
(a) c = 0, with y(x) = 0 or y(x) = x, or
(b) c 6= 0 and limx0 y 2 y 4 = c/3.
The first possibility gives solutions that do not fit the boundary conditions.
Consider the second case. Near x = 0 the equation is approximated by y 2 y 4 = c1 , and
this has the solution y = c2 x2/3 , with c1 = c62 (2/3)4 .
Now write the first-integral as the quadratic in y 2 ,
 
c 1  p 
4 2
3y 2y 1 + 2 = 0 with solution y 2 = y 4y 2 + 3c .
y 3y

If c = 0 this reduces to y 2 = (y 2y)/3y, and the upper sign gives the previous
equation, y 2 = 1.
15.8. SOLUTIONS FOR CHAPTER 8 541

If c 6= 0, then since y(0) = 0, we must have c > 0 and only the upper sign gives a real
solution; hence the required equation for y is
Z y s
2 1  p
2
 3y
y = y + 4y + 3c giving x(y) = dy p . (15.5)
3y 0 y + 4y 2 + 3c

Note that if y is small this becomes


Z y s r
3y 4 3/2
x(y) dy = y giving y x2/3 ,
0 3c 3c

in accordance with the previous analysis. On defining a new variable z by y = z 3c/2
the dependence upon c is removed to an external factor
Z z s
3c 3z 2y
x(z) = dz , z= .
2 0 2
z+2 z +1 3c
The boundary condition at x = 1 gives a relation between c and A which allows us to
deduce that solutions exist only if A 1. Equation 15.5, with x = 1 and y = A gives
Z A s
3y
1= dy f (y) where f (y) = p . (15.6)
0 y + 3c + 4y 2

For c = 0, f (y) = 1, so A = 1, as expected: for c 6= 0 we rely on graphical methods.


Graphs of the integrand for 0 < y < 2 and c = 0.1 , 0.3 and 1.0 are shown in figure 15.11,
and here we see that for small c, f (y) 1 for most values of y, and
that f (y) increases
rapidly from zero at the origin to close to unity over a distance 3c/2.

1
c=0

0.8 c=1
c=0.3
0.6
c=0.1
0.4 c=0.01
0.2
00 0.5 1 1.5 2
Figure 15.11 Typical graphs of f (y), defined in equa-
tion 15.6, for c = 0.1, 0.3 and 1.

Equation 15.6 shows that the value of A needs to be such that the area under f (y) for
0 < y < A is unity. It follows from the figure that if c > 0 we must have A > 1 and
that no solutions exists if A < 1.

Solution for Exercise 8.26


The Jacobi equation for this functional is
d2 u 1
+ 2u = 0 with solution u = sin x.
dx2
542 CHAPTER 15. SOLUTIONS TO EXERCISES

Then from theorem 8.6 (page 223), with 2 S[y, h] = 2S[h], we see that S[h] > 0 if the
interval (0, a) does not contain a conjugate point, that is a < . In this case, put
= k/a to give
a a
k2
Z Z
dx f (x)2 > dx f (x)2 for any k < .
0 a2 0

Equation 8.11 (page 217) gives


Z a a
2
Z
2

dx f (x) > 2 dx f (x)2
0 a 0

which is a weaker inequality.

Solution for Exercise 8.27


In this example F = 4y 2 8y y 2 , so Fy = 2y and Fy = 8y 8 giving the
Euler-Lagrange equation

y + 4y = 4 with the general solution y = 1 + cos 2x + sin 2x

The boundary condition at x = 0 gives y(0) = 1 + = 1, so = 0: at at x = /4,


y(/4) = 1 + = 0. Hence the stationary path is y(x) = 1 sin 2x.
Now evaluate the functional along y(x) + h(x), where y is the stationary path. The
functional is
Z /4
S[y + h] = S[y] 2 dx h 2 4h2 , h(0) = h(/4) = 0.

0

But, from exercise 8.26 with a = /4 and we see that


/4 /4
16k 2
Z Z
dx h 2 > dx h2 , k < .
0 2 0

It follows that S[y + h] < S[y] for all h, proving the result.

Solution for Exercise 8.28


The integrand depends only upon y , so the stationary path is the straight line y = Ax/a,
joining the end points. But if b = 1, S[y] = y(a) = A, which is independent of the path,
and if b = 0, S[y] = a, also independent of the path; thus if b = 0 or 1 there are no
stationary paths.
Since F = y b we have
 (b2)
(b2) A
P (x) = F y y = b(b 1)y = b(b 1) , Q(x) = 0.
a

Hence the second variation is positive if b > 1 or b < 0 and the stationary path is a
minimum of S[y]. If 0 < b < 1, P < 0 and the stationary path is a maximum of S[y].
Consider the variation h(x) = sin(nx/a), so h (x) = O(n) and choose n > 1, to
see that this not a strong minimum.
15.8. SOLUTIONS FOR CHAPTER 8 543

Solution for Exercise 8.29


Consider the solution with the parameters (, ) and ( + h1 , + h2 ), where is suffi-
ciently small so that second-order terms may be neglected and h1 and h2 are constants,
then

y(x, + h1 , + h2 ) = y(, ) + Y (x, , ) + O( 2 ) where Y = h1 y + h2 y .

We may choose h1 and h2 such that Y (a, , ) = 0 and Yx (a, , ) = 1. Since Y


is a solution of the variational equation, that is Jacobis equation, then if R(x) =
y (x)/y (x) has the same value at the points x = a and x = a it follows that Y (a) = 0
so the two points are conjugate.

Solution for Exercise 8.30


(a) The first-integral of the functional can be written in the form
3y 3
= 2,
y 2 c

where c is a positive constant. The general solution of this equation is 2 y = cx + d:
since y(0) = 1, d = 2 and then 2A = ca + 2 giving the solution
1
y = (1 bx)2 where b= (1 A) > 0.
a

(b) In this example


 
6y 3 d 1 3 1
P = 4
= 4 >0 and Q = 2 = .
y 8b y dx y 3 4b (1 bx)4
2

Hence Jacobis equation is

u 2b2 u d2 u
 
d du
+ =0 which expands to (1bx)2 +2b(1bx) +2b2 u = 0.
dx (1 bx)2 (1 bx)4 dx 2 dx

(c) If u = C(1 bx) then u = bC(1 bx)1 and u = b2 C( 1)(1 bx)2


and substituting these expressions into the equations gives 2 3 + 2 = 0, and = 1
and 2. Thus
u(x) = (1 bx) + (1 bx)2
and the initial conditions, u(0) = 0, u (0) = 1, give u(x) = x(1 bx). This solution has
one zero at x = 1/b = a/(1 A) > a, so the stationary path is a minimum of S[y].
544 CHAPTER 15. SOLUTIONS TO EXERCISES

15.9 Solutions for chapter 9


Solution for Exercise 9.1
(a) Define f (x) by d(x)2 = f (x) = x2 a2 x + a2 and differentiate,

f (x) = 2x a2 = 0 when x = a2 /2.

The function f is quadratic with a single minimum (since


p the coefficient of x2 is pos-
2
itive), at xp
= a /2. If a < 2, the distance d(x) = f (x) therefore has a minimum

at (a2 /2, a 1 a2 /2). For a > 2, the distance d(x) has no stationary points for
x 1, and is monotonically increasing as x decreases from unity. Hence for a > 2
the smallest distance is at (1, 0), but this is not a stationary point.

(b) In the parametric representation d( )2 = g( ) = 4 + (a2 2) 2 + 1, and

g ( ) = 2 2 2 2 + a2 = 0 when = 0 and 2 = 1 a2 /2.




Also g ( ) = 12 2 + 2(a2 2): at = 0, g = 2(a2 2) and at 2 = 1 a2 /2,


g = 4(2 a2 ), so we have the following classification.
a =0 2 = 1 a2 /2

a> 2 minimum

a< 2 maximum minimum

Solution for Exercise 9.2

Consider curve defined parametrically by (x(t), y(t)) and


an element of arc, AB, with A at t and B at t + t. To B
first-order in t the lengths AC and CB of the right angled
triangle ACB are xt and yt, respectively. Pythagoras s yt
theorem gives the length of the hypotenuse AB to be s2 =
(x2 + y 2 )t2 + p
O(t3 ). Thus on taking the limit t 0 A
we obtain s = x2 + y 2 ; the length between ta and tb is C
x t
therefore
Z tb Z tb p
s= dt s = dt x2 + y 2 .
ta ta

Solution for Exercise 9.3


The area, A, under the graph of y(x) is
Z xb Z tb
A= dx y = dt xy.
xa ta

Now use the identity xy = d(xy)/dt yx to obtain the alternative expression for the
area Z tb h itb
A= dt xy + xy .
ta ta
15.9. SOLUTIONS FOR CHAPTER 9 545
R tb  
Adding these two expressions gives 2A = ta
dt xy xy + bB aA.
y
The areas of the triangles OA A and OB B are, respectively,
1 1
2 aA and 2 bB. Since the area of OBB is the sum of the

areas of OAA , OAB and A ABB , we obtain B

1 tB
Z xB
1 1
Z
dt (xy xy) + aA + dx y(x) = bB, A
2 tA 2 xA 2
x
which is the relation derived in the question. O A B

Solution for Exercise 9.4


We change variables in the conventional manner:

b
1 b
 
1 dt dy d dx d
Z Z  
S= d x y = d x()y () x ()y() .
2 a d d dt d dt 2 a

Solution for Exercise 9.5


The area of the first
Z three curves, all traversed anticlockwise with increasing , are given
1 2
by the integral d (xy x y), so we have the following areas.
2 0
ab 2
Z
d cos2 + sin2 = ab.

Ellipse: Ae =
2 0Z
2
3
Aa = a2 d cos4 sin2 + sin4 cos2

Astroid:
2 Z0
2
3 3 2
= a2 d cos2 sin2 = a .
2 0 8
Z 2 h
Cardioid: Ac = a2 d (2 cos cos 2)(cos cos 2)
0 i
+ (sin sin 2)(2 sin sin 2)
Z 2
2
= 3a d (1 cos ) = 6a2 .
0
The area of the cycloid is given by

2 2
1 a2
Z Z  
Acy = d (x y y x) = d (1 cos )2 ( sin ) sin
2 0 2 0
2
a2
Z
= d (2 2 cos sin ) = 3a2 .
2 0

The lengths of the curves are given by the formula


Z 2 p
L= d x2 + y 2 .
0
546 CHAPTER 15. SOLUTIONS TO EXERCISES

For these four cases the lengths of the curves are


Z 2 p p 
Ellipse : Le = d a2 sin2 + b2 cos2 = 4aE 1 b2 /a2 , b a,
0
Z 2
Astroid : La = 3a d | sin cos | = 6a,
0
Z 2 Z
Cardioid : Lc = 2 2a d 1 cos = 8a d sin(/2) = 16a,
0 0
Z 2 q Z 2
2 2
Cycloid : Lcy = a d (1 cos ) + sin = 2a d sin(/2) = 8a.
0 0

Solution for Exercise 9.6


The functional is
Z 1 1  2 Z 1
y y 2
Z
S= dx y 2 = dt x = dt .
0 0 x 0 x

Note that we are free to choose a convenient interval for t. In this case = y 2 /x and
the equations for x and y are, respectively
d y 2
   
d y
= 0, 2 = 0.
dt x2 dt x
Integrating these gives z = y/x =constant, in both cases.

Solution for Exercise 9.7


The functional is Z 1 Z 1

S= dx F (y ) = dt xF (y/x)
0 0
so = xF (z), where z = y/x = y (x), giving
z z
= F (z) + xF (z) = F (z) zF (z) and = xF (z) = F (z).
x x y y
The Euler-Lagrange equation for x is
d h i
F (z) zF (z) = 0 or z zF (z) = 0.
dt
dF (z)
The Euler-Lagrange equation for y is = 0 or zF (z) = 0. These are the same
dt
provided z 6= 0: if z = 0 then z = 0 so the equations always have the same solution. If
F (z) = 0 then neither of the Euler-Lagrange equations can be solved for z.

Solution for Exercise 9.8


The Euler-Lagrange equations for x and y are, respectively,
! !
d x d y
p = 0 and p = 0. (15.7)
dt x2 + y 2 dt x2 + y 2
15.9. SOLUTIONS FOR CHAPTER 9 547

Hence x = cx and y = cy for some constants cx and cy .


The general solution of these Euler-Lagrange equations is therefore x = cx t + dx ,
y = cy t + dy , with dx and dy constants. Eliminating t gives xcx ycy = dx cy dy cx ,
which agrees with the quoted expression if = cy , = cx and = dx cy dy cx .
For the boundary conditions given, dx = dy = 0 and x = at, y = At, so y = Ax/a.
If a = 0 then x = 0 for all t and y = At, but this solution cannot be represented by an
equation of the form y = mx.

Solution for Exercise 9.9


The integrand does not depend upon the independent variable, t, so, in equation 7.13
(page 202), we may put = 1 and 1 = 2 = 0. Then equation 7.15 becomes, on
replacing (y1 , y2 ) by (x, y) and F by ,

= xx + yy + constant.

Solution for Exercise 9.10


Differentiation with respect to x gives
 
y
= F xFy = F y Fy .
x x x
Differentiation with respect to y gives
 
y
= xFy = Fy .
y y x

Solution for Exercise 9.11


(a) In this case

x = f (r, ) = r cos , y = g(r, ) = r sin , z = h(z) = z

so
s2 = r2 sin2 2 + r2 cos2 2 + z 2 = r2 2 + z 2
giving E = r2 , F = 0, G = 1 and 2 = r2 2 + z 2 .
(b) Equations 9.18 and 9.19, with u = and v = z, are respectively

2
d r = 0 and d q z = 0.
q
dt dt
r2 2 + z 2 r2 2 + z 2

One solution of these equations is = c1 and z = c2 , where c1 and c2 are constants.


Hence = c1 t + d1 and z = c2 t + d2 . Both equations are linear in t, hence z is
a linear function of ; thus z = A( 1 ) + z1 and the boundary conditions give
A = (z2 z1 )/(2 1 ).
Alternatively, introduce the distance s along the curve, defined by
Z t q
s(t) = dt z 2 + r2 2
0
548 CHAPTER 15. SOLUTIONS TO EXERCISES

so the equations become z (s) = a and (s) = b with solutions

s s
z(s) = z1 + (z2 z1 ) , (s) = 1 + (2 1 ) , 0 s S,
S S

where S is the length of the curve. Setting t = s/S gives the previous solution, but any
other parameter will suffice, for instance s = S sin(t/2) with 0 t 1.

Solution for Exercise 9.12


On expanding the Euler-Lagrange equation for x we obtain,

dx y x + y x y x(xx + y y)
= p 2 .
dt x 2
x + y 2 (x + y 2 )3/2

The numerator of the right-hand side simplifies to xy y 2 yy yx+ xy x2 + y 2 . Similarly




the equation for y expands to

dy y 2 + y y y y(xx + y y) p 2
= p 2 x + y 2 ,
dt y x2 + y 2 (x + y 2 )3/2

and the numerator of the right-hand side simplifies to yy x2 xxy y x2 x2 + y 2 .



Hence, equation 9.13 becomes
h i h i
xxy y 2 yy yx2 + x2 y x2 + y 2 + yy yx2 xxy y 2 x2 y x2 + y 2 = 0.

Solution for Exercise 9.13


If y = c cosh(t + ) the second of equations 9.27 for x becomes x = c||, since x > 0,
giving x = c||t + , for some constant . The boundary conditions for x then give
= 0 and c|| = a, so x = at.
The boundary conditions for y give A/c = cosh( ) and hence = 0 and
A/c = cosh(a/c).

Solution for Exercise 9.14


The Euler-Lagrange equations for x and y are respectively
 2  
d y d y y
=0 and = 0 giving = m = constant.
dt x dt x x

Hence y = mx and a further integration gives y = mx + c. Since x = y = 0 when t = 0,


c = 0: and since x = y = 1 when t = 1, m = 1 giving y = x.

Solution for Exercise 9.15


The corners at all the points equivalent to that at B lie on the line y = x a, for
some a > 0. For the first triangle, the point A is at the origin, so the coordinates of
the other points are B = (a, 0) and C = (a/2, a/2), as shown in figure 15.12.
15.9. SOLUTIONS FOR CHAPTER 9 549

y y=x y=x a

o
45 x
A
O B
Figure 15.12 Diagram showing the first right angled triangle, ABC.
Since the angle CAB is /4 and the length of the diagonal is a the
triangle is isosceles and the length of the shorter sides is a/ 2.

It follows that the apex of the N th triangle has coordinates x = y = N a/2; if this is at
(1, 1) then N a = 2.
On the segments parallel to the x-axis the functional has value 0 (because y = 0),
but on each of the other segments its value is S1 = a/2. So the whole integral is
S = N S1 = 1.
On making a arbitrarily small, and increasing N so that N a = 2, we see that all
points on the zig-zag path are at most a distance a/2 from the straight line y = x.

Solution for Exercise 9.16


The parametric equations give y 2 = 4a2 t2 = 4ax, which is the equation of a parabola.
The required length is
Z 1 p Zp 1 Z 1 p
S = dt x2 + y 2 = (2at)2 + (2a)2 = 4a
dt dt 1 + t2
1 1 0
 
= 2a (1 + sinh 1 cosh 1 ) = 2a sinh1 1 + 2 where sinh 1 = 1.

h i
But sinh1 z = ln(z + 1 + z 2 ), so S = 2a 2 + ln(1 + 2) .

Solution for Exercise 9.17


The equation of an epicycloid is most easily obtained using complex variables to repre-
sent the distances shown in figure 15.13.

y
O

O P x

Figure 15.13

The complex number representing the vector OO is z1 = (R + r)ei . The vector O P


is represented by z2 = rei , so that OP = z1 + z2 . But, since the smaller circle rolls
550 CHAPTER 15. SOLUTIONS TO EXERCISES

on the larger circle r = R; also + + = and hence


z = (R + r) exp(i) + r exp(i( )
 
R+r
= (R + r) exp(i) r exp i .
r
The real and imaginary parts give the quoted equations.
Put a = R/r so that
x y
= (1 + a) cos cos(1 + a), = (1 + a) sin sin(1 + a)
r r
and
x h i y h i
= (1 + a) sin sin(1 + a) , = (1 + a) cos cos(1 + a) .
r r
With increasing the curve is traced out anticlockwise, so the area is given by
1 2
Z
S = d (xy x y)
2 0
Z 2
1 2 n  
= r (1 + a) d (1 + a) cos cos(1 + a) cos cos(1 + a)
2 0
  o
+ sin sin(1 + a) (1 + a) sin sin(1 + a)
Z 2
1 2  
= r (1 + a)(2 + a) d 1 cos cos(1 + a) sin sin(1 + a)
2 0
Z 2
1 2
= r (1 + a)(2 + a) d (1 cos a).
2 0

If a = n is an integer this gives S = (R + r)(R + 2r).


For the length we use the above relations for x and y to give, with a = n,
x2 + y 2 = 2r2 (1 + a)2 (1 cos n) = 4r2 (1 + a)2 sin2 (n/2)
so that the length is
2  
1
Z

S = 2r(1 + n) d sin
n = 8r(1 + n) = 8(R + r).
0 2
In the limit r R the epicycloid comprises a set of n approximate cycloids made
by a circle or radius r. The length of each approximate cycloid is approximately 8r, so
the total length is 8nr = 8R.

Solution for Exercise 9.18


R 2
(a) If k 1 the area is most conveniently given by A = 0 dx y(x), that is
Z 2 Z 2
A = a2 d x ()y() = a2 d y()2
0 0
Z 2
2
d 1 2k cos + k 2 cos2 = a2 2 + k 2 .
 
= a
0
15.9. SOLUTIONS FOR CHAPTER 9 551

(b) Consider the loop surrounding the origin. The values of when x = 0 are given by
= k sin and if (0, ) is the real root of this equation the required values of are
0, : but the points corresponding to are identical. Since y() = a(1 k cos ) >
y(0) = a(1 k), = 0 corresponds to the bottom of the loop and = the top. Hence
the area of the loop is given by
Z Z
2

d 1 + k 2 2k cos k sin
 
A = d (yx y x) = a
0 0
= a2 (1 + k 2 ) 2k sin + k( cos sin ) .
 

This equation simplifies if the equation for , = k sin , is used,

A = a2 k 2 2 + k cos .
 

(c) The loop surrounding the origin intersects the positive x-axis when y = 0, that is
cos = 1/k: the root required is the negative root in the interval (, 0) which we
denote by , so 0 < < . At this point x () = 0 (since x () = y()) so the curve
is perpendicular to the x-axis. It follows, because of symmetry, that the maximum
width of the loop is 2x0 where x0 = a(k sin ). Hence adjacent loops intersect when
2x0 = 2a, that is when k satisfies the equation
p
= k 2 1 cos1 (1/k).

Solution for Exercise 9.19


In this example x = a(1 1/ cosh2 u) = a tanh2 u and y = a sinh u/ cosh2 u, so the arc
length is
s
v
sinh2 u v iv
Z Z h
4
S = a du tanh u + =a du tanh u = a ln(cosh u) ,
0 cosh4 u 0 0

= a ln(cosh v).

The shape of the curve defined by these equations is shown in figure 15.14.

y/a

x/a
Figure 15.14 Graph showing the curve defined by the parametric equations
x = a(utanh u), y = a/ cosh u. The cusp on the y-axis, at y = a, corresponds
to u = 0 and near here y/a = (1 (3x/a)2/3 .
552 CHAPTER 15. SOLUTIONS TO EXERCISES

Solution for Exercise 9.20


Consider the two close points with parameters t and t + t: if A = (x(t), y(t)),
B = (x(t + t), y(t + t)) and C is at the origin, O then the area enclosed by OAB,
S is approximately the same as the area of the triangle formed by joining A to B by
a straight line. Thus since x3 = y3 = 0
1  1
S = x(t)y(t + t) x(t + t)y(t) + O(t2 ) = (xy xy) t + O(t2 ).
2 2
1 t2
Z
Hence the area of the region OP Q is S = dt (xy xy) .
2 t1

Solution for Exercise 9.21


Since (x, y, x, y) = (x, y, x, y), by putting = 1/x we find that (x, y, 1, y (x)) =
(x, y, x, y)/x which is the first relation. Differentiate the first expression with re-
spect to x to give x (x, y, x, y) = x (x, y, x, y) and with = 1/x the second result
is obtained.

Solution for Exercise 9.22


Since (x, y, x, y) = xF (x, y, y/x) differentiation with respect to x gives
 
y
x = F + xFy = F y Fy (x, y, y ).
x x
Now observe that the choice of the parameter t is irrelevant, so let t = x, and then the
first Euler-Lagrange equation for x becomes, since x = 1
d
(F y Fy ) Fx = 0
dx
and expanding this gives
 
dFy dFy
Fx + y Fy + y F y F y
y y Fx = 0 that is Fy y = 0.
dx dx
Further, since y = Fy and y = xFy the second Euler-Lagrange equation for y
becomes
dFy
Fy = 0.
dx
Hence both parametric Euler-Lagrange equations are equivalent to the original Euler-
Lagrange equation.

Solution for Exercise 9.23


(a) The Euler-Lagrange equation for x is
!
d 2x + y p
p = 0 which integrates to 2x + y = A x2 + xy + y 2 , (15.8)
dt x2 + xy + y 2

where A is a constant. Dividing by x gives the equation


p
2 + y (x) = A 1 + y (x) + y (x)2 .
15.9. SOLUTIONS FOR CHAPTER 9 553

(b) This equation can be rearranged to give y =constant, with general solution y = mx + c.
The boundary conditions give c = 0 and m = Y /X.
(c) Equation 15.8, involving x and y, can be rearranged to given an equation involving
only one derivative, y (x) because it is a homogeneous function of degree 1 in x and y,
because x is homogeneous with degree zero, see exercise 1.25(c) (page 28). This is in
contrast to the example treated in the following exercise.

Solution for Exercise 9.24


(a) The Euler-Lagrange equations for x and y are, respectively

d d
(2x + y) = 0 and (x + 2y) = 0,
dt dt
which can both be integrated once to give 2x + y = A and x + 2y = C. Notice that
neither of these can, alone, be rearranged to given an equation for y (x), as was possible
in the previous exercise. Integrate again to obtain 2x + y = At + B, x + 2y = Ct + D.
(b) The boundary condition at t = 0 gives B = D = 0 and that at t = 1 gives x = Xt,
y = Y t.
(c) In this case both equations are needed to find a solution because the equations are
not homogeneous functions of x and y.

Solution for Exercise 9.25


The Euler-Lagrange equations for x and y are
! !
d y x 1 d x y 1
p y = 0 and p + x = 0
dt 2 2
x + y 2 2 dt 2 x2 + y 2 2

respectively. Integrating these gives, respectively,


x y
y+ p = A2 and x p = A1 .
x2 + y 2 x2 + y 2

These may be rearranged in the form


x y
p = (y A2 ) and p = x A1 .
x2 + y 2 x2 + y 2

Squaring and adding these gives (x A1 )2 + (y A2 )2 = 2 which is the equation of a


circle of radius with centre at (A1 , A2 ).
554 CHAPTER 15. SOLUTIONS TO EXERCISES

15.10 Solutions for chapter 10


Solution for Exercise 10.1
Suppose the stationary solution, y(x) exists then T [y] is stationary. One set of allowed
variations about this curve are those curves that pass through the same end points
as y(x). The stationary path for these boundaries is a cycloid because the Euler-
Lagrange equation must be the same as the conventional brachistochrone, only the
boundary conditions are different. The present boundary conditions merely picks out
a different cycloid.

Solution for Exercise 10.2


The functional is
Z X p
S[y] = dx 1 + y 2, y(0) = A,
0

and the Euler-Lagrange equation has the solution y = 0. The solution passing through
(0, A) is therefore y = mx + A, for some m to be determined. The natural boundary
condition, equation 10.7, is
y
Fy = p = 0,
1 + y 2
that is, y = m = 0, so that y = A, which defines a straight line parallel to the x-axis.

Solution for Exercise 10.3


The Euler-Lagrange equation is y + y = 0 and the solution satisfying the boundary
condition at x = 0 is y = A cos x + sin x, for some . The natural boundary condition
is Fy = 2y = 0 at x = /4,
that is cos /4 A sin /4 = 0, that is = A. Thus the
required solution is y = A 2 sin(x + /4).

Solution for Exercise 10.4


The analysis follows that of the text, the only difference being that the varied path,
y + h, is fixed at x = b, so h(b) = 0. Then equation 10.5 becomes
Z b    
F d F F
S[y, h] = h(a) dx h(x).
y x=a a dx y y

The same reasoning as used in the text gives the required result.

Solution for Exercise 10.5


The Euler-Lagrange equation is y y = 0 and the solution satisfying the boundary
condition at x = 1 is y = B cosh(1 x) + sinh(1 x), for some . The natural
boundary condition is Fy = 2y = 0 at x = 0, that is B sinh 1 + cosh 1 = 0. Thus the
required solution is y = B cosh x/ cosh 1.

Solution for Exercise 10.6


Using the path defined in equation 10.10 we have

dz dz . dx 1 dx 4b
z = = = and = sin2 ,
dx d d tan d
15.10. SOLUTIONS FOR CHAPTER 10 555

and the time is given by the functional, see equation 5.6 (page 150),
Z /2 r s Z /2 s
1 dx 1 + z 2 b b
T = d =2 d = .
2g 0 d z g 0 g

Solution for Exercise 10.7


The Euler-Lagrange equation for this problem is dFy /dx = 0 and the natural boundary
condition at x = b is Fy |x=b = 0. Hence the Euler-Lagrange equation becomes Fy = 0,
that is,
( )
F 1 c2 y 2 v2
= v giving y (x) = .
y c2 v 2 c2
p
c2 (1 + y 2 ) v 2

Integrating this and assuming that v(x) 0 and that y (x) 0 gives
1 x
Z
y(x) = du v(u).
c 0

Solution for Exercise 10.8


(a) In this case F = y 2 y 2 so F/y = 2y , F/y = 2y giving the Euler-Lagrange
equation y + y = 0.
(b) First note that no boundary conditions are given. Suppose that y(x) and y(x) +
h(x) are two admissible functions. Then the Gateaux differential is
h i Z b
S[y, h] = 2 gb h(b)y(b) ga h(a)y(a) + 2 dx (h y hy) .
a

Integrate by parts to put this in the form


    Z b
S[y, h] = 2h(b) y (b) + gb y(b) 2h(a) y (a) + ga y(a) 2 dx (y + y) h.
a

Using the subset of variations with h(a) = h(b) = 0 and the fundamental lemma of the
Calculus of Variations we see that S[y] is stationary only on those paths satisfying the
equations y + y = 0. On these paths the Gateaux differential is
   
S[y, h] = 2h(b) y (b) + gb y(b) 2h(a) y (a) + ga y(a)

and this is zero for all variations only if ga y(a) + y (a) = 0 and gb y(b) + y (b) = 0.

Solution for Exercise 10.9 Z b


On the varied path S[y + h] = dx F (x, y + h, y + h , y + h ) so differentiating
a
with respect to , using the chain rule, gives
Z b  
dS F F F
= dx h + h + h
d a y y y
where the derivatives of F are evaluated at y + h. Now put = 0 to obtain the result.
556 CHAPTER 15. SOLUTIONS TO EXERCISES

Solution for Exercise 10.10


In this example F = 12 y 2 gy, so the Euler-Lagrange equation is y (4) = g with
y(0) = y (0) = 0. The general solution of this equation is
g 4
y(x) = x + Ax3 + Bx2 + Cx + D
24
for some constants A, B, C and D. The conditions at x = 0 give C = D = 0. The
conditions 10.17 becomes y (L) = 0 and y (3) (L) = 0 so that

g 2 g gL gL2
L + 6AL + 2B = 0 and L + 6A = 0 = A= , B= ,
2 6 4
g 2 2
x x 4Lx + 6L2 .

giving the solution y(x) =
24
Solution for Exercise 10.11
(a) The Gateaux differential is given in equation 10.14 and since h(a) = h(b) = 0 this
reduces to
 b Z b  2     
F d F d F F
S[y, h] = h + dx + h.
y a a dx2 y dx y y

Using the subset of varied paths for which h (a) = h (b) = 0, we see that y(x) satisfies
the Euler-Lagrange equation

d2
   
F d F F
2

+ = 0, y(a) = A, y(b) = B.
dx y dx y y

The other boundary conditions are obtained by considering those paths for which
h (a) = 0 and those for which h (b) = 0, which gives Fy |a = Fy |b = 0.

(b) In this problem F = 21 y 2 gy and the appropriate Euler-Lagrange equation is


y (4) = g, with the boundary conditions are y(0) = y(L) = 0. The general solution of
this equation that satisfies the conditions y(0) = 0 is
g 4
y(x) = x + Ax3 + Bx2 + Cx.
24
But y(L) = 0 and since Fy = y (x) we also have y (0) = y (L) = 0. Since
g 2
y (x) = x + 6Ax + 2B the condition at x = 0 gives B = 0. Then the conditions at
2
x = L give

gL gL4 gL4 gL3


A= and 0 = + CL giving C=
12 24 12 24
so that
g g
x4 2Lx3 + L3 x = x(L x) L2 + xL x2 .
 
y(x) =
24 24
15.10. SOLUTIONS FOR CHAPTER 10 557

Solution for Exercise 10.12


The integrand of the functional is independent of x, so the first integral is
!
1 p 2
y 2 1 1
F y Fy = 1+y p = p =
y 1+y 2 y 1+y 2 c

were c is a constant. Rearranging gives


y
c2 u
Z
y 2 = 1 or, assuming y > 0, x= du .
y2 0 c u2
2

p
Integration gives x = c c2 y 2 , or y 2 + (x c)2 = c2 . This is the equation of a
circle, radius c with centre at (c, 0), and is also the solution for y < 0.
The transversality conditions shows that the stationary path must be perpendicular
to the line y = x a. But the only lines that are perpendicular to a circle are its
diameters, and hence this line must pass through the centre of the circle, that is c = a,
giving the stationary path y 2 + (x a)2 = a2 .
The same result follows algebraically, but this derivation is more difficult. With
= y x + a, the boundary condition 10.26 becomes

y 1
Fy + (y Fy F ) = 0 that is p =0
y 1 + y 2 c
p
but y 1 + y 2 = c therefore y = 1.

If the intersection is at x = v, the equation
for y , with y = 1 gives y = c/ 2and
from the equation for the circle v = c(1 1/ 2); the required root is v = c(1 + 1/ 2),
the other root corresponding to y = 1. Now substitute these coordinates into the
straight line equation, y = x a, to see that c = a.

Solution for Exercise 10.13


The solution of the Euler-Lagrange equation is given in equations 5.8 (page 151) and
the boundary conditions at x = 0 give d = 0, so

1 2
x= c (2 sin 2) , y = A c2 sin2 , 0 b .
2
p
The integrand of the functional may be taken to be F = 1 + y 2 / A y, so the
transversality condition 10.26 gives, since = x/a + y/b 1,

y
 
1 1 dy a
p =0 that is = .
A y 1 + y 2 a b dx b

dy 1 a
Hence the equation for b is = = . Finally, at = b the end of the
dx tan b b
cycloid is on the line x/a + y/b = 1 that is

c2   1 
2b sin 2b + A c2 sin2 b = 1.
2a b
558 CHAPTER 15. SOLUTIONS TO EXERCISES

Since sin 2b = 2 sin b cos b this becomes


 
2
 a 2
 A
c b sin b cos b sin b = a 1
b b
but b/a = tan b so this simplifies to c2 b = a(1 A/b), which gives c once a, b and
A are known. Notice that the solution exists only if A < b, as would be expected.

Solution for Exercise 10.14


(a) Points on the ellipse can be parameterised by x = a cos and y = b sin . Substi-
tuting these expressions into the equation of the straight line gives
r
a b a2 b2
cos + sin = 1 which gives 2
+ 2 cos( ) = 1,
A B A B
where
a/A b/B
cos = q and sin = q .
a2 b2 a2 b2
A2 + B2 A2 + B2

The equation for can be written in the form cos( ) = AB/, and this has real
roots only if |AB| . If |AB| > the equation has only complex roots and the
ellipse and the line do not intersect.
Z p
(b) The functional is S[y] = dx 1 + y 2 where the pairs of coordinates (u, v), on
u
the ellipse, and (, ), on the line, satisfy the equations
u2 v2
e = + 1 and l = + 1
a2 b2 A B
and also v = y(u) and = y().
The general solution of the Euler-Lagrange equation is y = mx + c, where m and c are
constants, which are chosen to satisfy the boundary conditions.
For the boundary conditions, equation 10.29, we require
y m 1
Fy = p = and y Fy F = .
1+y 2 1 + m2 1 + m2
The boundary conditions on the ellipse give
u m v 1 mu v
2 = 0 and hence = 2. (15.9)
a 2 1 + m2 b 1 + m2 a2 b
The boundary conditions on the straight line give
1 m 1 1 A
=0 and hence m= , (15.10)
A 1 + m2 B 1 + m2 B
which is the condition for the stationary path to be perpendicular to the straight line.
Also these points lie on the boundary curves,
u2 v2
2
+ 2 =1 and v = mu + c (15.11)
a b
15.10. SOLUTIONS FOR CHAPTER 10 559

and

+ = 1 and = m + c. (15.12)
A B
Thus we have six equations for the six parameters (u, v), (, ) and (m, c) that we need
to find.
The distance along the stationary path is
Z p p
S[y] = dx 1 + m2 = ( u) 1 + m2 . (15.13)
u

Now m = A/B is given directly, equation 15.10, and subtracting equations 15.11
from 15.12 gives v = ( u)m: rearranging this and substituting for v from 15.9
gives  2 
b
= m + mu 1 .
a2
Substitute this into the first of equation 15.12 gives
 2
 2 u

2 2 2 b 2 2 2
+ 2 = AB 2 .

B +A +A u 1 = AB or ( u) B + A
a2 a

Using
equations 15.11 and 15.9 we obtain (since u > 0) u = aB/ and since 1 + m2 =
A2 + B 2 /B we have

A2 + B 2 AB
S[y] = ( u) = .
B A2 + B 2

There are easier ways of deriving this result.

Solution for Exercise 10.15


As in the text the appropriate solution of the Euler-Lagrange equations are x = ct and
y = dt, which is equivalent to y = mx with m = d/c. The boundary bondition 10.37
then gives ( )c + ( )d = 0. If the boundary curve can be represented by a function
f (x), then f (x) = / so / = c/d and mf (x) = 1.
If two lines intersect at a point where they have gradients m1 and m2 the angle
between them, is given by
 
1 1 1 m2 m1
= tan m2 tan m1 = tan .
1 + m1 m2

If m1 m2 = 1 the lines intersect at right angles. Hence the condition mf (x) = 1


means that the stationary path and the boundary line intersect at right angles.

Solution for Exercise 10.16


The functional defining the distance between the origin and a point on the parabola is
Z v p
S[y] = dx 1 + y 2 , y(0) = 0,
0

with the right-hand end of the path on the curve (x, y) = y 2 + a2 (x 1) = 0.


560 CHAPTER 15. SOLUTIONS TO EXERCISES

The solution of the associated Euler-Lagrange equation satisfing the boundary con-
dition at x = 0 is y = mx for some constant m. The boundary condition on the
parabola gives, on using equation 10.26, a2 y = 2y and hence v = a2 /2. At this point
y = ma2 /2, and since this point lies on the parabolawe obtain m2 = 4/a2 2. Thus
there are stationary paths if a < 2 and none if a > 2. Note that the stationary path
through (1, 0) is not given by this method.

Solution for Exercise 10.17


First consider the case A = 0. Suppose that z(x) is a solution, so that S[z, h] = 0;
then for any constant c, S[cz] = c2 S[z] and since cz(x) also satisfies the boundary
conditions if A = 0, S[cz, h] = 0, cz is also a stationary path and there is no unique,
nontrivial solution.
The Gateaux differential is, since h(0) = 0,
Z Z L

S[y, h] = 2Cy()h() + dx y1 h + dx y2 h
0
  Z Z L
= h() 2Cy() + y1 () y2 () + y2 (L)h(L) dx hy1 dx hy2 .
0

Using the same arguments as in the text we see that y1 and y2 satisfy the Euler-Lagrange
equations

y1 = 0, y1 (0) = A, 0 x , and y2 = 0, x L,

together with the following conditions at x =

y1 () y2 () + 2Cy() = 0 and y1 () = y2 (),

the first to make the path stationary and the second to ensure that the path is continous.
In addition, the natural boundary condition at x = L, see equation 10.7, gives Fy =
y (L) = 0, that is = 0.
The Euler-Lagrange equations have the following solutions

y1 (x) = x + A and y2 (x) = x + ,

where , and are constants, and the natural boundary condition at x = L, y (L) = 0,
gives = 0.
Continuity at x = gives = + A and the other condition at x = gives
+ 2C = 0, and these two equations give
2CA A
= , and = .
1 + 2C 1 + 2C
Hence the solution is
1 + 2C( x)

A
, 0 x ,
y(x) = 1 + 2C
A

, x L.
1 + 2C
Note that if A = 0, y(x) = 0.
15.10. SOLUTIONS FOR CHAPTER 10 561

Solution for Exercise 10.18


Differentiate equation 10.49 with respect to and then set = 0,
h i
S = F (c, y1 (c), y1 (c)) F (c, y2 (c), y2 (c))
Z c   Z b  

+ dx h1 Fy + h1 Fy + dx h2 Fy + h2 Fy .
a c

Integrate by parts to give


   
S = lim F + h1 Fy lim F + h2 Fy
xc xc+
Z c   Z b  
dFy dFy
+ dx h1 Fy + dx h2 Fy .
a dx c dx

Using the subset of variations for which = 0 gives equations 10.50 and 10.51, for y1 (x)
and y2 (x). On these paths S reduces to equation 10.49.

Solution for Exercise 10.19


(a) The general solution of the Euler-Lagrange is y = mx + c and since y(0) = 0, c = 0;
also y(2) = 1, so m = 1/2 giving y = x/2.
(b) Substituting y + h into the functional and expanding in powers of gives
Z 2
1 2 2
Z
2
S[y + h] = S[y] +
dx Fy h + dx Fy y h 2 + O(3 ), F (y ) = y 2 (1 y ) .
0 2 0

The terms O() is, by definition, zero on a stationary path and since

Fy y = 2 1 6y + 6y 2


on the path y = x/2 we have


Z 2
S[y + h] = S[y] 2 dx h 2 + O(3 ).
0

Hence for all allowed h and 0 < || 1, S[y + h] S[y] < 0, so this stationary path is
a local maximum of S[y].

Solution for Exercise 10.20


We require the simultaneous solutions of the equations f (x) = f (y) and g(x) = g(y),
where
g(z) = z(z 1)(2z 1) and f (z) = z 2 (z 1)(3z 1).
Clearly x = y (that is m1 = m2 ) are solutions but since c = (1 m2 )/(m1 m2 ), these
solutions are excluded. The solutions given are those for which f (z) = g(z) = 0, so now
we require solutions for which f and g are nonzero.
Given a value of y, with f (y) 6= 0 and g(y) 6= 0, the equation g(x) = g(y) is a cubic
for x and has either one or three real solutions. If there is one real solution this must be
x = y. If there are three real solutions, then one is x = y leaving two other potentially
562 CHAPTER 15. SOLUTIONS TO EXERCISES

interesting solutions; denote these by x1 (y) and x2 (y). These must also be solutions of
the quartic, f (x) = f (y). But the root structure of a quartic and a cubic is different; for
instance roots of the quartic will coalesce are different values of y than for the cubic, so
it is unlikely that there is a range of y for which x1 (y) and x2 (y) satisfy both equations.
There may, however, be accidental coincidences; we now show that there are none.
Consider the differences,
f (x) f (y) = (x y)F (x, y) and g(x) g(y) = (x y)G(x, y)
where F (x, y) and G(x, y) are respectively symmetric cubic and quadratic functions of
x and y. The solution x = y is of no interest, so we require the solutions of the equations
F (x, y) = 3(x3 + y 3 ) + 3x2 y + 3xy 2 4(x2 + y 2 + xy) + x + y = 0,
G(x, y) = 2(x2 + y 2 ) + 2xy 3(x + y) + 1 = 0.
These equations are more conveniently expressed in terms of the variables u = x + y
and v = xy, (so when x = y, u2 = 4v)
F (u, v) = 3u3 4u2 6uv + u + 4v = 0,
G(u, v) = 1 3u + 2u2 2v = 0,
which gives
1
1 3u + 2u2

v=
2
and
F = (1 u)(3u2 6u + 2) = 0.
If u = 1 then v = 0 and (x, y) = (0, 1) and (1, 0), which are the solutions found in the
text.
If 3u2 6u + 2 = 0, u = 1 3/3 and v = 1/3 3/6, so u2 = 4v giving x = y.
Hence there are no real solutions other than those found in the text.

Solution for Exercise 10.21


Suppose the corners are at c1 and c2 , with 0 < c1 < c2 < 2, and that the gradients are
m1 (for 0 x < c1 ), m2 (for c1 < x < c2 ) and m3 (for c2 x 2). The Weierstrass-
Erdmann conditions can be applied at each corner so the following solutions are possible,
(m1 , m2 , m3 ) = (1, 0, 1) and (0, 1, 0).
Consider each in turn.
For (m1 , m2 , m3 ) = (1, 0, 1) the continuous stationary path is constructed by draw-
ing any horizontal line joining the lines y = x and y = x 1, that is

x, 0 x c,
y(x) = c, c x c + 1,
x 1, c + 1 x 2, 0 < c < 1.

For (m1 , m2 , m3 ) = (0, 1, 0) the stationary path is



0, 0 x c,
y(x) = x c, c x c + 1,
1, c + 1 x 2, 0 < c < 1.

15.10. SOLUTIONS FOR CHAPTER 10 563

Solution for Exercise 10.22


The general solution of the Euler-Lagrange equation is y = mx + c (because the inte-
grand depends only upon y ). Hence the continuous solution, with one corner at x = a,
has the form 
m1 x, 0 x a < 4,
y(x) =
m2 (x 4) + 2, a x 4,
2 4m2
with continuity at x = a giving m1 a = m2 (a 4) + 2, that is a = .
m1 m2
The Weierstrass-Erdmann condition 10.54 gives m1 (m21 1) = m2 (m22 1) and three
obvious solutions are (m1 , m2 ) = (0, 1), (1, 0), (1, 1) and m1 = m2 , which we
ignore, because it does not give a corner.
The condition 10.53 gives (m21 1)(3m21 + 1) = (m22 1)(3m22 + 1). Only the solution
(m1 , m2 ) = (1, 1), also satisfies this equation.
Thus there are two stationary paths
 
x, 0 x 3, x, 0 x 1,
y(x) = and y(x) =
6 x, 3 x 4, x 2, 1 x 4.

The value of S is the same on each path, S = 0, which is a global minimum.

Solution for Exercise 10.23


Using the Taylor series for sin 2 and cos 2 the left-hand side of equation 10.71 becomes
 3  
1 1  
23 1 22 + O(24 ) 1 22 + O(24 ) = 23 1 22 + O(24 ) ,
6 2

so the equation can be written in the form


1/3
2 = d 1 22 + O(24 ) .

If |2 | 1 the approximate solution of this equation is 2 = d, so we may put 2 = d


in the right-hand side to obtain
 
 1/3 1
2 = d 1 d2 + O(d4 ) = d 1 + d2 + O(d4 ) .
3

For small |2 |
1
tan 2 = 2 + 23 + O(25 )
 3   
1 1 2
= d 1 + d2 + d3 + O(d5 ) = d 1 + d2 + O(d5 ),
3 3 3

and hence
 
1 1 1 2 2 4
p2 = = = 1 d + O(d ) .
tan 2 d(1 + 32 d2 + O(d4 )) d 3
564 CHAPTER 15. SOLUTIONS TO EXERCISES

Solution for Exercise 10.24


If a b we see from figure 10.14 that p2 p1 > 1/ 3, so we may use the approximation
(p2 ) p32 to write equation 10.70 in the form
b
p32 = (p1 ),
a
and then using the approximation (p2 ) (p1 ), see figure 10.11 , equation 10.73
becomes
 4/3
A (p2 ) 3 4 A 3 b
, but (p2 ) p2 giving (p1 ) ,
a (p1 ) 4 a 4(p1 ) a
that is  4/3
A b3 1
(p1 )1/3 , p1 > .
a a4 3

Since (p) is monotonic increasing
for p > 1/ 3 the smallest value of the right-hand
side is given by setting p1 = 1/ 3. Then we see that if a b the equation for p1 has
a solution only if
 4/3  1/3  4/3
A 3 b 1 b
= 1.09 ,
a 4 a 3 a
and there are no solutions for smaller values of A.
Numerical calculations with a = 1, b = 10, 20 and 30 give this lower boundary
to be 19.5, 53.3 and 94.2 respectively. The above formula gives 23.5, 59.2 and 102
respectively.

Solution for Exercise 10.25


Clearly G(1) = 0 and for large p, on using the binomial expansion
 
3p 4 4 ln p 7
G(p) = 1 +
4(1 + p2 )2 3p2 3p4 3p4
  
3p 2 3 6 4 4 ln p 7
= 1 2 + 4 + O(p ) 1+ 2 4
4 p p 3p 3p4 3p
 
3p 2 4 ln p
= 1 2 + O(p4 ) .
4 3p 3p4
Hence
3p 1 ln p
G(p) = 3 + O(p3 ),
4 2p p
which gives the result quoted.
The derivative of G(p) can be obtained by defining
3 4 7 1
f (p) = p + p2 ln p so that f (p) = (3p2 1)(p2 + 1)
4 4 p
and
(1 3p2 )f (p) 3p2 1 3p2 1
 
11 1 4 2
G (p) = + = + p + p + ln p .
(1 + p2 )3 p2 + 1 (p2 + 1)3 4 4
15.10. SOLUTIONS FOR CHAPTER 10 565

If h(p) = 11/4 + p4 /4 + p2 + ln p then clearly h(p) > 0 for p > 1. At p = 1/ 3,
h(p) = 111 12 ln 3 > 0 and h (p) = p3 + 2p2 + 1/p > 0 for p > 0; hence h(p) > 0 for
36
p > 1/ 3 and therefore G (p) > 0 for p > 1/ 3.

Solution for Exercise 10.26


Since
b a+k
1
Z Z
S2 [y + h] = (a + k)2 +
dx F (x, y + h ) dx F (x, y + h )
2 a a

differentiation with respect to gives


b
d
Z
S2 [y + h] = (a + k)k + dx h Fy (x, y + h )
d a
Z z

dx h Fy (x, y + h ), (15.14)

kF z, y (z) + h (z)
a

where z = a + k. Now set = 0 to obtain the Gateaux differential


Z b
S2 [y, h] = ak kF a, y (a) + dx h Fy (x, y ).

a

Integrating by parts and using the fact that h(b) = 0 then gives the required result,
b
dFy
Z
S2 [y, h] = ak kF a, y (a) h(a)Fy a, y (a)

 
dx h .
a dx

Solution for Exercise 10.27


The first derivative of S2 [y+h] with respect to is given in the solution of exercise 10.26,
equation 15.14. Differentiating this expression again gives
Z b
d2 S2
= k2 + dx h 2 Fy y (x, y + h )
d2 a
 


d

k kFx z, y (z) + h (z) + Fy z, y (z) + h (z) y (z) + h (z)
d
Z z
kh (z)Fy z, y (z) + h (z) dx h 2 Fy y (x, y + h ),

a

where z = a + k. Putting = 0 this becomes


b
d2 S2
Z
2
= k + dx h 2 Fy y (x, y )
d2 a
k 2 Fx (a, y (a)) k ky (a) + h (a) Fy a, y (a) kh (a)Fy (a, y (a)).
 

But equation 10.75 can be expanded in powers of to give


 
2 1 2

0 = (ky (a) + h(a)) + k y (a) + kh (a) + O(3 ).

2
566 CHAPTER 15. SOLUTIONS TO EXERCISES

Since this equation is true for all in a neighbourhood of the origin it follows that
ky (a) + h(a) = 0, as in equation 10.76, and k 2 y (a) + 2kh (a) = 0. Thus the second
derivative becomes the simple expression
b
d2 S2 h Z
2
i
= k 1 Fx a, y
(a) + dx h 2 Fy y (x, y ).
d2 a

But,
x 1 2x(3y 2 1)
F = so that Fx = and Fy y = ,
1 + y 2 1 + y 2 (1 + y 2 )3
and since y (a) = 1, see equation 10.81 the second derivative becomes
b
d2 S2 1 x(3y 2 1) 2
Z
= k2 + 2 dx h .
d2 2 a (1 + y 2 )3

It follows that provided 3y 2 > 1 the second variation is positive for all nonzero k and
h(x), and that the stationary path is a weak local minimum.

Solution for Exercise 10.28


(a) The functional can be written
1  
u A 1 x
Z
S1 [z] = b2 du , B= n+ , u=
0 1 + B 2 sin2 (n + 1/2)u b 2 b
n Z p Z 1
2
X (n+1/2) u u
= b du 2 + b2 du 2 .
p=1
p1
(n+1/2)
1+ B2 sin (n + 1/2)u n
(n+1/2)
1+ B2 sin (n + 1/2)u

In each integral of the sum put (n+1/2)u = p1+w, 0 w 1, and (n+1/2)u = n+w
in the last integral to write this in the form
n Z 1 Z 1/2
b2 X p1+w b2 n+w
S1 [z] = dw 2 + dw .
(n + 1/2)2 p=1 0 2
1 + B sin w (n + 1/2)2
0 1 + B 2 sin2 w

But
1 1
p1+w 1 p
Z Z
dw p dw = , p = 1, 2, , n
0 1 + B 2 sin2 w 0 1+ B2 2
sin w 1 + B2
and
1/2 1
n+w 1 n+1
Z Z
dw (n + 1) dw =
0 1 + B 2 sin2 w 0 1+ B2 2
sin w 1 + B2
so that
n+1
b2 X b2
S1 [z] p= (1 + O(1/n)) .
(n + 1/2)2 1 + B 2 p=1 2 1 + B2

But B = O(n) so S1 [z] = O(1/n) for large n. Hence, given any number > 0, an n can
be found such that S1 [z] < .

(b) Since max(z (x) = O(n) the derivative is not bounded and z(x) satisfies the D0
norm.
15.10. SOLUTIONS FOR CHAPTER 10 567

Solution for Exercise 10.29


The natural boundary condition is, see equation 10.7, Fy = 0 at x = b, that is

f (x, y)y
p = 0 at x = b.
1 + y 2

Since f (x, y) 6= 0, this means that y (b) = 0 and that the stationary path is perpendic-
ular to the line x = b.

Solution for Exercise 10.30


The general solution of the Euler-Lagrange equation is y(x) = c cosh(x/c + d), for some
constants c and d. The boundary condition at x = 0 gives A = c cosh d, and at x = a,
y (a) = 0, as shown in exercise 10.29, gives
a  a
y (a) = sinh + d = 0 that is d = .
c c
 
ax a
Hence y(x) = c cosh with A = c cosh . With = a/c the equation for c be-
c c
1
comes A/a = cosh , which was considered in section 5.3 (equation 5.16, page 158).
This has two real solutions if A > 1.509a and none for smaller A.

Solution for Exercise 10.31


(a) On the path y + h, with h(a) = 0, the functional is
  Z v
S[y + h] = G y(v) + h(v) + dx F (x, y + h, y + h )
a

so that differentiating with respect to and then setting = 0 gives


Z v

S[y, h] = h(v)G (y(v)) + dx (hFy + h Fy )
a
Z v  
dFy
= h (G (y) + Fy ) + dx Fy h.

x=v a dx

Using the subset of variations for which h(v) = 0, we see that y(x) satisfies the Euler-
Lagrange equation. Then the boundary terms shows that the boundary condition at
x = v is
Fy (v, y(v), y (v)) + G (y(v)) = 0.

(b) If the right end of the path satisfies (v, y(v)) = 0 and the varied path is y + h,
and ends at v + for some , the same analysis that leads to equation 10.19 gives
 
x + y (v)y + h(v)y = 0.

On the varied path


  Z v+
S[y + h] = G y(v + ) + h(v + ) + dx F (x, y + h, y + h ).
a
568 CHAPTER 15. SOLUTIONS TO EXERCISES

It is convenient to write
y(v + ) + h(v + ) = y(v) + (y (v) + h(v)) + O(2 )
then differentiate with respect to , and then set = 0 to obtain the Gateaux differential
  Z v
S = G (y(v)) y (v) + h(v) + F (v, y(v), v (v)) + dx (hFy + h Fy ) .
a

Integrating the second terms of the integral by parts gives


    Z v 
dFy

S = G (y)y + F + h G + Fy + dx Fy h.
a dx
Because is proportional to h, using the subset of variations for which h(v) = 0, we
deduce that y(x) satisfies the Euler-Lagrange equation. Then it follows that at x = v
   
G (y) + Fy x + y Fy F y = 0.

Solution for Exercise 10.32


The Euler-Lagrange equation has a first integral,
1 1
F y Fy = p =
y 1 + y 2 c

so that, as in the solution to exercise 10.12, (x c)2 + y 2 = c2 . The boundary conditions


show that this circle must intersect the circle x2 + (y r)2 = r2 at right angles.
The two circles intersect perpendicularly at the origin, see figure 15.15. At other
points the angles of intersection are the same, so the circle (x c)2 + y 2 = c2 satisfies
the boundary condition for all c and there are infinitely many stationary paths.

y Boundary curve

Stationary path
r
r
c
x
c
Figure 15.15

Solution for Exercise 10.33 p


Since F (x, y, y ) = f (x, y) 1 + y 2 exp( tan1 y ) we have
( )
y

1
p
Fy = f (x, y) exp( tan y ) p + 1 + y 2
1 + y 2 1 + y 2
f (x, y)
= p ( + y ) exp( tan1 y )
1 + y 2
15.10. SOLUTIONS FOR CHAPTER 10 569

so that
f (x, y)
y Fy F = p exp( tan1 y )(y 1).
1 + y 2
The boundary condition 10.26 then gives
f (x, y)
p {( + y )x + (y 1)y } = 0 at x = v.
1 + y 2

If the gradient of and the stationary path at x = v are respectively tan C and tan ,
so x = y tan C and y = tan , this boundary condition becomes

( + tan ) tan c + ( tan 1) = 0

which rearranges to (tan tan c ) = 1 + tan tan c , giving the required result.

Solution for Exercise 10.34


On the varied path y + h, with h(0) = h (0) = h(1) = 0, the functional has the value
Z 1 Z 1
dx x(y + h) + (y + h )2 dx (xh + 2h y ) .

S[y + h] = giving S =
0 0

Integrating by parts twice gives


h i1 Z 1  

S = h y hy
+ dx x + 2y (4) h.
0 0

Since h(0) = h (0) = h(1) = 0 this gives


Z 1  

S = h (1)y (1) + dx x + 2y (4) h,
0

and since on an admissible path h (1) can take any value we must have y (1) = 0. The
Euler-Lagrange equation is therefore

2y (4) (x) = x, y(0) = 0, y (0) = 0, y(1) = 0, y (1) = 0.

The general solution of this equation is y(x) = x5 /240 + Ax3 + Bx2 + Cx + D. The
boundary conditions y(0) = y (0) = 0 give D = C = 0, and the other two conditions
give the equations
1 1
0= +A+B and 0 = + 6A + 2B
240 12
with solution A = 3/160 and B = 7/480 giving y(x) = x2 (1 x)(2x2 + 2x 7)/480.

Solution for Exercise 10.35


First compute the Gateaux differential by evaluating the functional on the path y + h,
where h(0) = h (0) = 0,
  Z L 
1

2
E[y + h] = M g y(L) + h(L) + dx (y + h ) (x)g(y + h)
0 2
570 CHAPTER 15. SOLUTIONS TO EXERCISES

so that
L
dE
Z  
= M gh(L) + dx h (y + h ) gh
d 0

and putting = 0 gives the Gateaux differential


Z L  
[E, h] = M gh(L) + dx h y gh .
0

Integrating by parts twice gives


  Z L  
[E, h] = M g + y (L) h(L) + y (L)h (L) + dx y (4) g h(x).
0

Hence the stationary path satisfies the equation

d4 y Mg
= (x)g, y(0) = y (0) = y (L) = 0, y (3) (L) = .
dx4
If is independent of x the general solution of this equation that satisfies the boundary
conditions at x = 0 is
g 4
y(x) = x + Ax3 + Bx2
24
and the constants A and B are determined by the two conditions at x = L; since
g 2 g
y (2) (x) = x + 6Ax + 2B and y (3) (x) = x + 6A
2
these give the equations
 
g Lg L
A= (M + L) and B = M+ .
6 2 2

Hence
g 4 x3 Lx2
 
L
y(x) = x (M + L)g + M+ g,
24 6 2 2
g 2 2 Mg 2
= x (x 4xL + 6L2 ) + x (3L x).
24 6

Solution for Exercise 10.36


This problem is essentially the same as that considered in section 10.5.1, but with the
addition of natural boundary conditions at both ends, as in exercise 10.11. Let the
stationary path be (
y1 (x), 0 x ,
y(x) =
y2 (x), x L,
with y1 () = y2 (). Using the same analysis as used to derive equation 10.43 we obtain
Z   Z L  
E = M gh() + dx y1 h1 gh1 + dx y2 h2 gh2 .
0
15.10. SOLUTIONS FOR CHAPTER 10 571

Since Z h i Z
dx y h = y h y h + dx y (4) h

the Gateaux differential becomes


h i h iL
E = M gh() + y1 h1 y1 h1 + y2 h2 y2 h2
0
Z   Z L  
(4) (4)
+ dx y1 g h1 + dx y2 g h2 .
0

On collecting relevant terms together and using the fact that h(x) is continuous at x =
and that h1 (0) = h2 (L) = 0 this becomes

y2 y1 M g h(x) + (y1 h1 y2 h2 )
 
E =

x= x=
h i h i

y1 h1 + y 2 h2
x=0 x=L
Z   Z L  
(4) (4)
+ dx y1 g h1 + dx y2 g h2 .
0

Now choose the subset of variations that make all the boundary terms zero to see that
y1 and y2 satisfy the Euler-Lagrange equations
(4)
y1 = g, y1 (0) = 0, 0 x ,
(4)
y2 = g, y2 (L) = 0, x L.

The coefficient of h() in E gives


 
M g = y2 () y1 () .

Also, since h1 (0) 6= 0 and h2 (L) 6= 0 the natural boundary conditions at x = 0 and L
are
y1 (0) = y2 (L) = 0.
Finally choose the subset of variations for which h (x) is continuous to see that y1 () =
y2 (), that is y (x) is continuous at x = .
For the sake of completeness we now show how these conditions can be used to
find the solution when is independent of x. This analysis was not requested in the
question.
The two solutions of the Euler-Lagrange equation that fit the boundary conditions
at x = 0 and L are
g 4
y1 (x) = x + a1 x3 + b1 x,
24
g
y2 (x) = (L x)4 + a2 (L x)3 + b2 (L x),
24
so there are four further constants to be determined by the conditions just derived. The
conditions of the second and third derivatives at x = give
gL
M g = gL 6(a1 + a2 ) and 6a2 L = 6(a1 + a2 ) + (2 L).
2
572 CHAPTER 15. SOLUTIONS TO EXERCISES

Hence
Mg Lg M g Lg
a1 = ( L) and a2 =
6L 12 6L 12
and
gL3 Mg gL3 Mg
b1 = + (L )(2L ) and b2 = + (L2 2 ).
24 6L 24 6L

Solution for Exercise 10.37


Here F = y 2 + 2yy + y 2 so that Fy = 2(y + y). The Weierstrass-Erdmann
(corner) conditions, equations 10.53 and 10.54, show that this expression is continuous
at any corner. Since y(x) is continuous it follows that y (x) is also continuous. The
Euler-Lagrange equation is second-order, so it follows (by differentiation) that all higher
derivatives are continuous. Hence there are no corners.

Solution for Exercise 10.38


The general solution of the Euler-Lagrange equation is y = mx + d. Let m1 and m2 be
the gradients to the left and right, respectively, of the corner. The Weierstrass-Erdmann
corner conditions, equations 10.53 and 10.54, give, since F y Fy = 2y 3 ,

m31 = m32 , m21 = m22 .

The second equation gives m2 = m1 and the first shows that the only solution is
m1 = m2 . Hence the functional has no corners.

Solution for Exercise 10.39


Here F = y 4 6y 2 and the general solution of the associated Euler-Lagrange equation
is y = mx + d. If there is a corner and if the gradients on the left and right-hand sides
are m1 and m2 then since

Fy = 4y y 2 3 and F y Fy = 3y 2 2 y 2
 

the Weierstrass-Erdmann corner conditions, equations 10.53 and 10.54, give

m1 m21 3 = m2 m22 3 and m21 m21 2 = m22 m22 2 .


   

The first equation gives m2 = m1 (we ignore the solution m2 = m1 ), and then the
second equation gives

m1 = m, m2 = m with m = 3.

The stationary path that satisfies the boundary conditions is y = mx (for 0 x c)


and y = A + m(a x) (for c x a) and continuity at x = c gives mc = A + m(a c).
Thus the two solutions are

mx, 0 x c,
y(x) =
A + m(a x), c x a, where c = A + ma , m = 3
2m
Since 0 < c < a we also need, ma < A < ma.
15.10. SOLUTIONS FOR CHAPTER 10 573

Solution for Exercise 10.40


The equation of the circle is most conveniently expressed in the parametric form
x = b + r cos , y = r sin (circle)
and the equation of the cycloid, found in section 5.2.3, is
1 2
x= c (2 sin 2), y = A c2 sin2 (cycloid).
2
The geometric interpretation of is shown in figure 15.16, where we have set A = b = 2
and r = 1/2 for the purpose of illustration.
2

1.5
Cycloid
1

Boundary circle
0.5

r
0
L
0 0.5 1 1.5 2 2.5 3 R

Figure 15.16 Example of a cycloid starting at (0, 2) and terminating


on the circumference of the circle of radius r = 1/2 with centre at
(2, 0).

The boundary conditions, equation 10.37, is therefore


2c2 r sin2 sin 2c2 sin cos cos = 0
which gives cos( ) = 0 (we ignore the solution sin = 0). This equation has many
solutions and we determine which is appropriate by considering the limiting cases where
= /2 (and c2 = A) so at the terminus the cycloid is tangential to the x-axis. Then
= , so the required solution is = /2.
The equations for c and the value of at the terminus, which we denote by , are
1 2
c (2 sin 2) = b r sin and c2 sin2 = A r cos .
2
An equation for is therefore
2 sin 2 b r sin
2 = , (b > r).
2 sin A r cos
This equation has one real root in the interval 0 < < , as may be seen by sketching
the graphs of
2 sin 2 b r sin
f1 () = 2 and f2 () = .
2 sin A r cos
Observe that f1 () = 34 + O( 3 ), that f1 () as and that f1 is monotonc
increasing for 0 < < . Note also that the behaviour of f2 depends upon whether
A > r or A < r, but in either case sketches of these functions show that there is one
real root for 0 < < .
574 CHAPTER 15. SOLUTIONS TO EXERCISES

15.11 Solutions for chapter 11


Solution for Exercise 11.1
Differentiating equation 11.2 gives h (x) = (3x + 1)h(x) so that h (x) = 0 at x = 1/3,
y = 1 x = 2/3, and here h = 3 exp(1/3). Since h > 0, the sign change of h follows
that of 3x + 1, so this stationary point is a maximum, as is clear from figure 11.1.

Solution for Exercise 11.2 p


The constraint can be written in the form y = b 1 x2 /a2 ; substituting this into the
area function, A = 4xy, gives the expression quoted. Differentiation gives

x2 4b a2 2x2
 
dA 4b p 2
= a x2 = ,
dx a a2 x2 a a2 x2

so that A (x) = 0 when x = a/ 2 (since x > 0). If x = a/ 2 r2 , A 2 , so that
4b a a2
A(x) has a local maximum at this point, with value A = a2 = 2ab.
a 2 2

Solution for Exercise 11.3


Using Pythagoras theorem the distance is given by D2 = (x A)2 + (y B)2 , and
since y = b(1 x/a) this becomes
 x 2  x
D2 = x2 2Ax + A2 b2 1 2bB 1 + B2
a a
a2 + b 2 2 b2
 
Bb
= x 2x A + + A2 + (b B)2 .
a2 a a

This is a quadratic equation in x and since the coefficient of x is positive it has a single
minimum, which is most easily seen by writing it in the form
2
a2 + b 2 (Aa + b(b B))2

a
D2 = 2
x 2 2
(Aa + b(b B)) + A2 + (b B)2 .
a a +b a2 + b 2

Hence D has its minimum value at


a
x= (Aa + b(b B))
a2 + b 2
and here
(Aa + b(b B))2 (ab Ab Ba)2
D2 = A2 + (b B)2 = .
a2 + b 2 a2 + b 2
For this example the method of Lagrange multipliers is easier, see exercise 11.6.

Solution for Exercise 11.4


Eliminate C to give f = sin A sin B sin(A + B), so that
 
fA = sin B cos A sin(A + B) + sin A cos(A + B) = sin B sin(2A + B),
 
fB = sin A cos B sin(A + B) + sin B cos(A + B) = sin A sin(A + 2B).
15.11. SOLUTIONS FOR CHAPTER 11 575

Since sin A 6= 0 and sin B 6= 0 (because 0 < A, B < ) we have sin(2A + B) = 0 and
sin(A+2B) = 0, that is 2A+B = n and A+2B = m, with n and m positive integers,
both smaller than 3. Hence 3A = (2nm) and 3B = (2mn), and 3C = (3nm).
The bounds on A and B give n = m = 1 and hence A = B = C = /3.

Solution for Exercise 11.5


The contours of f are the curves f (x, y) = c, which we assume can be expressed in as
the function y(x); the gradient on these contours are given by fx + fy y (x) = 0, that is
y (x) = fx /fy .
Suppose the constraint g(x, y) = 0 (which is a particular contour of g) defines the
function yg (x), so that on this curve the original function has the values
dz
z(x) = f (x, yg (x)) and = fx + fy yg (x).
dx
fx
Thus z(x) is stationary when yg (x) = = y (x), that is when the contour of f is
fy
tangential to the contour defined by g = 0.

Solution for Exercise 11.6


(a) For the walker on the hill we define
F (x, y, ) = h(x, y) (x + y 1) where h(x, y) = 3 exp(x2 y 2 /2),
so that
Fx = 2xh(x, y) = 0 and Fy = yh(x, y) = 0.
These give = yh(x, y) and = 2xh(x, y), and since h 6= 0, y = 2x, then the
equation of constraint gives 3x = 1, hence the result.
(b) For the area of the rectangle inscribed inside the ellipse we define
 2
y2

x
F (x, y, ) = 4xy + 1 ,
a2 b2
where is the Lagrange multiplier. Thus
2 2
Fx = 4y x = 0, Fy = 4x y = 0,
a2 b2
so that y = sx, for some s. Divide these equations to see that = 2ab and hence
s = b/a. The constraint equation gives 2x2 = a2 and then 2y 2 = b2 , so that the
stationary value of the area is A = 2ab.
(c) The auxiliary function is
x y 
F = (x A)2 + (y B)2 + 1
a b
so we require the solutions of

Fx = 2(x A) = 0 = x A =
a 2a

Fy = 2(y B) = 0 = y B = .
a 2b
576 CHAPTER 15. SOLUTIONS TO EXERCISES

Hence the constraint equation gives


   
A B 1 1 1 1 ab Ab Ba
+ + + = 1 that is + 2 =
a b 2 a2 b2 2 a 2 b ab

which gives
2 (ab Ab Ba)2
 
2 1 1
D = + 2 = .
4 a2 b a2 + b 2

Solution for Exercise 11.7


Let (x, y, z) be the point on the plane and D the distance, given by Pythagoras theorem,
D2 = x2 + y 2 + z 2 . Then F = D2 2(ax + by + cz d), with Lagrange multiplier 2.
Hence
Fx = 2(x a) = 0, Fy = 2(y b) = 0, Fz = 2(z c) = 0.
Now use the constraint equation to give (a2 + b2 + c2 ) = d and then

d2 |d|
D2 = 2 (a2 + b2 + c2 ) = that is D= .
a2 + b 2 + c2 a2 + b 2 + c2

Solution for Exercise 11.8


Let the coordinates of the corner opposite the origin be y
(u, v), as shown in the diagram. The area is A = uv and if
is a Lagrange multiplier the auxiliary function is b
u v  (u,v)
F = uv + 1
a b x/a+y/b=1
then Fu = v /a = 0 and Fv = u /b = 0. Solving these x
equation for u and v, and substituting into the constraint
equation gives 2 = ab and hence u = a/2, v = b/2 so the
a
stationary area is A = ab/4.

Solution for Exercise 11.9


We use the constraints to express x and y in terms of z, which is trivial because the
constraint equations are both linear; they give

x+y =1z and x + 2y = 2 3z

and have the solution x = z and y = 1 2z. Hence the expression for f (x, y, z) becomes

f (z) = az 2 + b(1 2z)2 + cz 2


= (a + 4b + c)z 2 4bz + b.

This is a quadratic in z (provided a + 4b + c 6= 0): if a + 4b + c > 0 it has a single


minimum at the root of
2b
f (z) = 2(a + 4b + c)z 4b = 0 that is z= .
a + 4b + c
15.11. SOLUTIONS FOR CHAPTER 11 577

If a + 4b + c < 0 this stationary point is a maximum. Using the expressions for x(z)
and y(z) the results quoted in equation 11.19 are obtained.

Solution for Exercise 11.10


The analysis is a minor generalisation of that given in section 11.2.1. Suppose that
a is the stationary point, and consider a nearby point a + u, that also satisfies the
constraint, so Taylors theorem gives
n
X
g(a + u) = g(a) + uk gk (a) + O( 2 )
k=1
Pn
and since g(a + u) = g(a) = 0 wePhave k=1 uk gk (a) = 0. Also, by definition,
n
f (a + u) f (a) = O( 2 ) and hence k=1 uk fk (a) = 0.
Pn  
If is a Lagrange multiplier we have, for all , k=1 uk fk (a) gk (a) = 0. It
follows, by the same reasoning as used in the text that fk (a) gk (a) = 0 for all k.
But these equations are just those that determine the stationary points of the aux-
iliary function F (x, ) = f (x) g(x).

Solution for Exercise 11.11


In this case the auxiliary function is
 
F = x + y 1 h(x, y) c , h(x, y) = 3 exp(x2 y 2 /2),

so that Fx = 1 + 2xh = 0 and Fy = 1 + yh = 0, giving y = 2x (as in the dual


problem). The constraint equation, h(x, y) = c, then gives c = 3 exp(3x2 ), that is
3x2 = ln(3/c).

Solution for Exercise 11.12


The volume of the box is Vb = xyz. The volume of the material is proportional to the
area of the sides and is Vm = (xy + 4yz + 2xz)d, so the auxiliary function can be taken
to be
F = xyz (xy + 4yz + 2xz)
where we have absorbed the thickness, d, into the Lagrange multiplier, and ignored an
irrelevant constant. The equation for the stationary values are

Fx = yz (y + 2z) = 0,
Fy = xz (x + 4z) = 0,
Fz = xy (4y + 2x) = 0.

Put y = sx and z = tx, so these become (assuming that x 6= 0)

stx = (s + 2t), tx = (1 + 4t), sx = (2 + 4s).

The second two equations can be rearranged to give t(x 4) = and s(x 4) = 2,
so that s = 2t. Also t = /(x 4), so the first equation gives x = 8; hence t = 1/4
and s = 1/2, giving 2y = x and 4z = x.
578 CHAPTER 15. SOLUTIONS TO EXERCISES

Solution for Exercise 11.13


The volume and surface area of the cylinder are, respectively, r2 h and 2rh. The
volume and surface area of a right circular cone of base radius r, height hc and slant
height l are, respectively, r2 hc /3 and rl; if the semi-vertical angle is we have
tan = r/hc and sin = r/l. Adding the volumes and areas in the appropriate
proportions gives the quoted results.
The auxiliary function can be taken to be

2r2 2r3
 
2
F = 2rh + r h + .
sin 3 tan

Differentiation with respect to h gives Fh = r(2 r) = 0, so that r = 2. Differenti-


ation with respect to gives

2r3 2r2 r
 
cos
F = 2r2 2 + = cos =0
sin 3 sin2 sin2 3

Since r = 2, this gives cos = 2/3.


Finally, differentiation with respect to r gives
 
4r 2r
Fr = 2h + r 2h +
sin tan
4r 2r
= 2h + (1 cos ) = 0 hence h= .
sin 3 sin
The volume is
!1/3
2r3 2r3 10r3 3 5V
V = + = and hence r= .
3 sin 3 tan 3 5 10

Solution for Exercise 11.14


The first equation can be rearranged to give tan x = sinh y and then 2 tan x = ey ey .
The second of these can be expressed as a quadratic in ey ,
p
e2y + 2ey tan x 1 = 0 with solutions ey = tan x 1 + tan2 x.

When x = 0, y = 0, so the upper sign gives the required solution, that is


p
ey = 1 + tan2 x tan x
s
4t2 2t
= 1+ , where t = tan(x/2),
(1 t2 )2 1 t2
1t
= ,
1+t

where we have used the identity tan x = 2t/(1 t2 ). Adding unity to each side of the
last equation gives the second of equations 11.5.
15.11. SOLUTIONS FOR CHAPTER 11 579

Solution for Exercise 11.15


If is the Lagrange multiplier, F = x2 + y 2 + z 2 + w2 (xyzw)2 and
Fx = 2x 1 (yzw)2 = x = 0 or (yzw)2 = 1,


Fy = 2y 1 (xzw)2 = y = 0 or (xzw)2 = 1,


Fz = 2z 1 (xyw)2 = z = 0 or (xyw)2 = 1,


Fw = 2w 1 (xyz)2 = w = 0 or (xyz)2 = 1.


If x = 0 then the three remaining equations for have no solution: hence we discard
the solution x = y = z = w = 0. The equation (yzw)2 = 1 gives = x2 (on
multiplying by x2 and using the constraint equation). Similarly, = y 2 = z 2 = w2 ,
hence (xyzw)2 = 4 = 1 and = 1 ( = 1 is not allowed); so there are 16 stationary
points, x = 1, y = 1, z = 1 and w = 1, all of which give f = 4.

Solution for Exercise 11.16


If is the Lagrange multiplier, F = xyzw9 (4x4 + 2y 8 + z 16 + 9w16 1) and
Fx = yzw9 16x3 = f = 16x4 ,
Fy = xzw9 16y 7 = f = 16y 8 ,
Fz = xyw9 16z 15 = f = 16z 16 ,
Fw = 9xyzw8 16 9w15 = f = 16w16 .
Hence x4 = y 8 = z 16 = w16 and since all variables are positive we can put w = a > 0
to give z = a, y = a2 , x = a4 and the constraint equation becomes
g = 4a16 + 2a16 + a16 + 9a16 = 1 that is 16a16 = 1.
1 1 1
Thus the stationary point is x = , y = , z = w = 1/4 .
2 2 2

Solution for Exercise 11.17


a2 b2 c2
If is the Lagrange multiplier, F = + + (x + y + z d) so that
x y z
F a2 F b2 F c2
= 2 = 0, = 2 =0 and = 2 = 0.
x x y y z z
a2 2 b2 c2
Hence x2 = , y = and z 2 = and the equation of constraint gives

1 a+b+c
(a + b + c) = d that is = ,
d
(a + b + c)2
and at this point f = (a + b + c) = .
d
Solution for Exercise 11.18
The distance D is given by D2 = (x A)2 + (y B)2 + (z C)2 , so the auxiliary
function is
F = (x A)2 + (y B)2 + (z C)2 2(ax + by + cz d)
580 CHAPTER 15. SOLUTIONS TO EXERCISES

with Lagrange multiplier 2. The derivatives are

Fx = 2(x A) 2a = x = a + A,
Fy = 2(y B) 2b = y = b + B,
Fz = 2(z C) 2c = z = c + C.

Thus the constraint gives

Aa + Bb + Cc d
(a2 + b2 + c2 ) + Aa + Bb + Cc = d = = .
a2 + b 2 + c2
But at the stationary point

(Aa + Bb + Cc d)2 |Aa + Bb + Cc d|


D2 = 2 (a2 + b2 + c2 ) = = D= .
a2 + b 2 + c2 a2 + b 2 + c2

Solution for Exercise 11.19


pq
Since f = pq/(p + q) the auxiliary equation is F (p, q, ) = (p + q 4c) and
p+q
we require the roots of

F q2 F p2
= = 0 and = = 0.
p (p + q)2 q (p + q)2

Clearly since both p and q are positive, p = q and then = 1/4. The constraint
equation then gives p = q = 2c.

Solution for Exercise 11.20


The auxiliary function is

F = ax2 + by 2 + cz 2 1 (x2 + y 2 1) 2 (x + y + z 1)

and so we require the solutions of

2
Fx = 2x(a 1 ) 2 = 0 = x =
2(a 1 )
2
Fy = 2y(b 1 ) 2 = 0 = y =
2(b 1 )
2
Fz = 2cz 2 = 0 = x = .
2c
The constraints now give the following equations for 1 and 2 : on the plane,
 
2 1 1 1
+ + = 1. (15.15)
2 a 1 b 1 c

And on the cylinder


2
 
1 1
+ = 1.
4 (a 1 )2 (b 1 )2
15.11. SOLUTIONS FOR CHAPTER 11 581

The first of these equations gives


 2
1 1 1 4 4 1 1
+ + = and the second gives = + .
a 1 b 1 c 22 22 (a 1 )2 (b 1 )2

These two equations give the following quadratic equation for 1

21 (a + b + 4c)1 + ab + 2c(a + b) + 2c2 = 0

which has the two real solutions


1 1 p
1 = (a + b + 4c) , = (a b)2 + 8c2 .
2 2
The quadratic equation for 1 can be rewritten in the form

(1 a)(1 b) = 2c(21 a b) 2c2

and also the solution for 1 gives a + b 21 = (4c ). Hence

1 1 4c
+ = .
a 1 b 1 2c(3c )

Hence, equation 15.15 becomes


 
2 4c 2c(6c 2)
1= 1 giving 2 = .
2c 6c 2 2c

Solution for Exercise 11.21


Let hc be the height of the right crcular cylinder, so its volume and surface area are

Vcy = r2 hc and Scy = 2rhc .

The volume and surface area of the cone are


1 2 p
Vcn = r h and Scn = r r 2 + h2
3
and since the material of the cone is double thickness the total volume and surface areas
are
1 p
V = r2 hc + r2 h and S = 2rhc + 2r r2 + h2 .
3
Eliminate hc to give

2V 2 p
S(r, h) = rh + 2r r2 + h2 .
r 3
Thus
S 2 2rh p
= r + = 0 and hence r2 + h2 = 3h or r = 2 2 h,
h 3 r 2 + h2
582 CHAPTER 15. SOLUTIONS TO EXERCISES

which is the first result. Also


S 2V 2 p 2r2
= 2 h + 2 r2 + h2 + = 0.
r r 3 r 2 + h2
3
Substituting for r = 2 2h and r2 + h2 = r gives
2 2

2V 2 r 3r 2 2 8
2
= + 2 + 2r = r 2
r 3 2 2 2 2 3 3

and hence
4 3 2  3
V = 2r = r 2 .
3 3
15.12. SOLUTIONS FOR CHAPTER 12 583

15.12 Solutions for chapter 12


Solution for Exercise 12.1
Put x = y + , so the integral becomes

1 1
 
1 2 1 3
Z Z
2 2 2 2
 
dy f (+y) y = dy y f () + yf () + y f () + y f ( + )
2 2 2 6

for some in (, ), where we have used Taylors series, (section 1.3.8). Since

4 4 5
Z Z Z
2 2
= 3, 2 2
dy 2 y 2 y 2 =
  
dy y dy y y = 0, ,
3 15

we see that
b
4
Z
dx f (x)g(x ) = f () + O( 3 ), = .
a 3

Solution for Exercise 12.2


If is the Lagrange multiplier,
Z 1
dx y 2 y ,

S[y] = y(0) = y(1) = 0,
0

and the associated Euler-Lagrange equation is 2y + = 0. This has the general solution

y(x) = x2 + ax + b,
4
where a and b are constants. The boundary condition at x = 0 gives b = 0: that at

x = 1 gives a = /4, so the solution is y(x) = x(1 x). The constraint gives
4
Z 1

A= dx x(1 x) = giving y(x) = 6Ax(1 x).
4 0 24

Solution for Exercise 12.3


If is the Lagrange multiplier,
Z 2
dx xy 2 y ,

S[y] = y(1) = y(2) = 0,
1

and the associated Euler-Lagrange equation is 2(xy ) + = 0, which has the general
solution

y(x) = x + a ln x + b,
2
where a and b are constants. The boundary condition at x = 1 gives b = /2: that at
x = 2 gives 0 = a ln 2 /2, so the solution is

ln x
y(x) = (1 x) + .
2 2 ln 2
584 CHAPTER 15. SOLUTIONS TO EXERCISES

The constraint gives


2
2
  
ln x 1 x ln x x (3 ln 2 2)
Z
1= dx 1 x + = x x2 + =
2 1 ln 2 2 2 ln 2 1 4 ln 2

and hence  
2 ln 2 ln x
y(x) = 1x+ .
3 ln 2 2 ln 2

Solution for Exercise 12.4


If is the Lagrange multiplier,
 
F = y 2 + z 2 2xz 4z y 2 xy z 2
= (1 )y 2 + (1 + )z 2 2xz + xy 4z

so the Euler-Lagrange equations for y and z are, respectively,


d  
2(1 )y + x = 0
dx
d  
2(1 + )z 2x + 4 = 0.
dx
The first equation gives
1
2(1 )y + x = A and hence 2(1 )y = Ax + B x2
2
where A and B are constants. The boundary condition at x = 0 give B = 0 and that
at x = 1 gives A = (4 3)/2.
Similarly the second Euler-Lagrange equation gives

2(1 + )z 2x = 4x + C and hence 2(1 + )z = x2 + Cx + D

and the boundary condition at x = 0 gives D = 0 and that at x = 1 gives C = 3 + 2.


Hence the solution is
(4 3)x x2 (3 + 2)x x2
y= and z = .
4(1 ) 2(1 + )
The values of the Lagrange multiplier are obtained by substituting these functions into
the constraint. The component integrals are
Z 1 Z 1
2 1
c1 = dx y = dx (4 3 2x)2
0 16(1 )2 0
 
1 49 2
= 16 32 +
16(1 )2 3
Z 1 Z 1
1
dx xy = dx (4 3)x 2x2

c2 =
0 4(1 ) 0
12 13
= .
24(1 )
15.12. SOLUTIONS FOR CHAPTER 12 585

Hence
24 46 + 232
c1 c2 = .
48(1 )2
Also
1 1
1 1
Z Z
2
c3 = dx z 2 = dx (3 + 2 2x) = 1 + .
0 4(1 + )2 0 12(1 + )2
Thus the constraint becomes
24 46 + 232 1
c= 1 .
48(1 )2 12(1 + )2

Solution for Exercise 12.5


If is the Lagrange multiplier then
Z 1
dx y y 2 ,

S[y] = y(0) = y(1) = 0,
0

and the Euler-Lagrange equation is 2y + 1 = 0, with the general solution


x2
y(x) = + Ax + B.
4
The boundary condition at x = 0 gives B = 0 and that at x = 1 gives A = 1/(4), so
the stationary path is
1
y(x) = x(1 x).
4
The constraint gives
 2 Z 1
1 1 1
c= dx (1 2x)2 = and hence = 3c,
4 0 3(4)2 4
which gives the required result.

Solution for Exercise 12.6


The function sinh passes through the origin with unitpgradient, and for > 0 its
gradient is a monotonic increasing function. Hence if L2 (A B)2 > a, equa-
tion 12.24 has a unique, positive solution. This condition can be written in the form
L2 > a2 + (A B)2 , which is simply the condition that the cable is longer than the
distance between the end points.
In the limit L2 2 2
p a + (A B) the root of equation 12.24 lend to zero, so c .
More precisely, if L (A B)2 = a(1 + ), for some small positive quantity ,
2

equation 12.24 becomes sinh = (1 + ) and using the approximation sinh = +


a
3 /6 + O( 5 ) gives 2 = 6, so c .
2 6
Solution for Exercise 12.7
Since y (x) = sinh((x + d)/c), y (0) = 0 if d = 0. Then equations 12.22 and 12.23 give
a a
L = c sinh = c sinh 2, = , and A B = c (cosh 2 1)
c 2c
586 CHAPTER 15. SOLUTIONS TO EXERCISES

and we may eliminate c by dividing these two equations,


AB cosh 2 1
= = tanh .
L sinh 2
Substitute for A B in equation 12.24 to obtain
L L
q
sinh = 1 tanh2 = giving a sinh 2 = L2.
a a cosh
Given L and a, provided L > a, this equation gives a unique real, positive value of .
Then the height difference, A B, is obtained from the equation A B = L tanh .

Solution for Exercise 12.8


On the path defined in equation 12.20 the energy functional is
Z a      Z a  
x+d x+d 2 x+d
E[y] = dx + c cosh cosh = L + c dx cosh
0 c c 0 c
c2
    
1 2a + 2d 2d
= L + ac + sinh sinh
2 4 c c

where = c cosh(d/c). Hence


     
d 1 1 2 a + 2d a
E[y ] = Lc0 cosh ac0 c0 cosh sinh ,
c0 2 2 c0 c0
     
d+ 1 1 2 a + 2d+ a
E[y+ ] = Lc0 cosh + ac0 + c0 cosh sinh .
c0 2 2 c0 c0

Define = E[y ] E[y+ ], so we need to show that > 0. Write in the form
    
d d+
= ac0 + Lc cosh + cosh
c0 c0
      
1 2 a + 2d a + 2d+ a
c0 cosh + cosh sinh
2 c0 c0 c0
   
d + d d+ d
= ac0 + 2Lc0 cosh cosh
2c0 2c0
     
2 a a + d+ + d d+ d
c0 sinh cosh cosh .
c0 c0 c0

But d = D0 0 , so (d+ + d )/c0 = 20 and (d+ d )/c0 = 2D0 , with 20 = a/c0 .


Hence
= ac0 + 2Lc0 cosh 0 cosh D0 c20 sinh 20 cosh 2D0 .
But L = 2c0 cosh D0 sinh 0 , and hence

c20 sinh 20 2 cosh2 D0 cosh 2D0 ac0



=
= c20 (sinh 20 20 ) > 0 if 0 > 0.
15.12. SOLUTIONS FOR CHAPTER 12 587

Solution for Exercise 12.9


(a) The Euler-Lagrange equation is
!
d (y )y p
p 1 + y 2 = 0 y(0) = 0, y(a) = A,
dx 1+y 2

and this expands to


(y )y = 1 + y 2 , y(0) = 0, y(a) = A.

(b) If y(x) = A w(u), u = a x then


dy dw du dw d2 y d2 w
= = and similarly 2
= 2.
dx du dx du dx du
Hence the Euler-Lagrange equation becomes
d2 w
(A w(u)) = 1 + w (u)2 , w(0) = 0, w(a) = A,
du2
and this can be rewritten in the form
(w )w (u) = 1 + w (u)2 , + = A.
Thus if y(x) is a solution of the Euler-Lagrange equation, so is w(x) if is replaced by
A .
For the minimum surface problem there is no Lagrange multiplier, that is = 0, so we
do not have sufficient flexibility for w(x) to be a solution.

Solution for Exercise 12.10


The area, A[y], and the constraint, C[y] = L, are
Z v Z v p
A[y] = dx y and C[y] = dx 1 + y 2 = L, y(0) = 0.
0 0

If is a Lagrange multiplier the auxiliary functional is


Z v  p 
A[y] = dx y 1 + y 2 , y(0) = 0.
0

The boundary condition at x = v is given by equation 12.31, with (x, y) = y, that is


p
y F y F = 0 where F = y 1 + y 2 .
But the functional has the first integral y F y F = c, for some constant c, and the
boundary condition shows that c = 0. Hence the equation for the stationary path is
p
dy 2 y 2
y= p or = with solution 2 = (x A)2 + y 2 ,
1 + y 2 dx y
for some constant A. The boundary condition at x = 0 gives = A, so the stationary
path is a semicircle of radius with centre at (, 0) and hence v = 2. The length of
the arc is therefore L = , which gives .
588 CHAPTER 15. SOLUTIONS TO EXERCISES

Solution for Exercise 12.11


(a) The surface area is derived in section 5.3. For the volume consider a thin disc centre
at x, and width x: its volume approximately is the product of the area of one surface,
y(x)2 , and the width. Hence
Z v Z v p
V [y] = dx y 2 and A[y] = 2 dx y 1 + y 2.
0 0

(b) If /2 is the Lagrange multiplier the modified functional is


Z v p
V [y] = dx F (y, y ) where F = y 2 y 1 + y 2, y(0) = 0.
0

The first integral of the associated Euler-Lagrange equation is

y
y F y F = p y 2 = c,
1+y 2

where c is a constant.
The boundary condition at x = v is given by equation 12.31 with (x, y) = y: hence
as in exercise 12.10, c = 0 and,the equation for the stationary path is exactly the same
as in exercise 12.10, that is 2 = (x B)2 + y 2 , for some constant B. The boundary
condition at x = 0 gives = B, so the stationary path is a semicircle of radius with
centre at (, 0) and hence v = 2. Since the shape created is a sphere of radius , its
area and volume are A = 42 and V = 43 3 = A3/2 /(6 ).

Solution for Exercise 12.12


The solution of the Euler-Lagrange equation is given in equation 12.20, and is
 
xd
y = + c cosh .
c

The left-hand end of the cable is constrained to the curve = x = 0, so the boundary
condition at x = 0 is, from equation 12.31,

(y )y
0 = F y = p , (x = 0), which gives d = 0.
1 + y 2

The boundary condition at x = a and the length constraint give


a a
A = + c cosh and L = c sinh .
c c
The second of these equations can be written in the form L/a = sinh , = a/c, so
give a unique positive value for c. Then the first equation can be used to write the
solution in the form a x
y = A c cosh + c cosh .
c c
15.12. SOLUTIONS FOR CHAPTER 12 589

Solution for Exercise 12.13


If is the Lagrange multiplier, the functional is given by equation 12.19 and the asso-
ciated Euler-Lagrange equation has the general solution
 
xd
y(x) = + c cosh .
c

There
p is a natural boundary condition at x = 0, so here F y = 0, where F = (y

2
) 1 + y , so

y (y )
p = 0, that is y(0) = or y (0) = 0.
1 + y 2

The first equation gives cosh(d/c) = 0, which cannot be satisfied, and the second gives
sinh(d/c) = 0, which gives d = 0.
The transversality condition, equation 12.31, with = x/a + y/b 1, gives
 
y y 1 v a
p = 0 at x = v, and hence sinh = .
1+y 2 a b c b

This is one equation relating v and c. The other is given by the length constraint
Z v p
v ac bL
L= dx 1 + y 2 = c sinh = hence c = .
0 c b a
Thus the required solution is
Lb  ax  Lb a
y = + cosh , 0x sinh1 .
a bL a b
Finally, at x = v we have v/a + y(v)/b = 1 and since
  
Lb 1 b Lp 2
y(v) = + cosh sinh =+ a + b2 ,
a a a

this gives

Lp 2 vb Lp 2 Lb2 a
=b a + b2 =b a + b2 2 sinh1 .
a a a a b

Solution for Exercise 12.14


Put m1 = tan 1 and m2 = tan 2 so that equations 12.35 become
     
tan2 1 tan2 2 = cos 2 cos 1 and 2 tan 1 tan 2 = sin 1 sin 2 .

Eliminate by dividing these equations: assuming that tan 1 6= tan 2 , a solution of


no interest, we obtain

sin 1 + sin 1
 
sin 1 cos 2 + sin 2 cos 1 2
2
2
2

= .
cos 1 + sin 1

2 cos 1 cos 2 2
2
2
2
590 CHAPTER 15. SOLUTIONS TO EXERCISES

Assuming that 1 6= 2 , because we have already dealt with this solution, gives
 
1 + 2
cos2 = cos 1 cos 2
2

which simplifies to 1 = cos 1 cos 2 + sin 1 sin 2 = cos(1 2 ), the only solution of
which is 1 = 2 + 2n.

Solution for Exercise 12.15


(a) The energy of the hanging mass is M gy() and the energy of the two portions of
the cable either side of it are
Z p Z a p
E1 [y] = g dx y 1 + y 2 and E2 [y] = g dx y 1 + y 2 .
0

Since the total energy is the sum of these three components we obtain the given result.
The constrains are just the lengths along each portion of the cable.

(b) If g1 and g2 are the Lagrange multipliers the modified functional is


Z q Z a q
E[y] = M gy() + g dx (y1 1 ) 1 + y1 2 + g dx (y2 2 ) 1 + y2 2 ,
0

where 
y1 (x), 0 x ,
y(x) = with y1 () = y2 ().
y2 (x), x a,
Now evaluate the functional on the varied path y + h, using the method described in
section 10.5.2. The corner moves to the point ( + u, y() + v), where u and v are
independent variables. Thus
  Z +u Z a
E[y+h] = M g y() + v +g dx F (y1 +h1 , y1 +h1 )+g dx F (y2 +h2 , y2 +h2 )
0 +u

p (15.16)
where F (y, y ) = (y ) 1 + y 2 .
We have, as in section 10.5.2 (but with notation changes)

y() + v = yk ( + u) + hk ( + u), k = 1 and 2,


yk () + uyk () + hk () + O(2 ).

=

Differentiate equation 15.16 with respect to and then set = 0 to obtain the Gateaux
differential,
h i
E[y] = M gv + gu F (y1 , y1 ) F (y2 , y2 )
x=
Z Z a
dx h1 Fy1 + h1 Fy1 + g dx h2 Fy2 + h2 Fy2 .
 
+g
0
15.12. SOLUTIONS FOR CHAPTER 12 591

The usual integration by parts gives, since h1 (0) = h2 (a) = 0,


h i
E[y] = M gv + gu F (y1 , y1 ) F (y2 , y2 ) + g h1 Fy1 + g h2 Fy2

x= x= x=
Z   Z a  
d  d 
= +g dx Fy1 Fy1 + g dx Fy2 Fy2 .
0 dx dx

First consider the subset of variations for which u = h() = 0, to obtain the Euler-
Lagrange equations satisfied by y1 and y2 :
d  
Fyk Fyk = 0, y1 (0) = B y2 (a) = A.
dx
p
Since F = (y ) 1 + y 2 is independent of x we the first integrals
yk k
p = ck = constant, k = 1 and 2. (15.17)
1 + yk 2

The general solution of these equations are


 
x dk
yk = k + ck cosh k = 1 and 2, (15.18)
ck

where c1 , c2 , d1 , d2 ) are constants to be determined.


(c) Now we need Weierstrass-Erdmann conditions at x = . From equation 15.16 we
have v = uyk () + hk (), k = 1 2, to replace hk (). Thus
h i h i
E[y] = M gv+gu F (y1 , y1 ) F (y2 , y2 ) +g (v uy1 ) Fy1 (v uy2 ) Fy2 .
x= x=

Collecting the coefficients of u and v together gives

E[y] = gv M Fy2 Fy1 + gu F (y1 , y1 ) y1 Fy1 F (y2 , y2 ) y2 Fy2 .


 

This expression must be zero for all u and v and hence we have the conditions

lim (F y Fy ) = lim (F y Fy ) .

M = Fy2 Fy1 and (15.19)
x+ x

The first of these equations represents the resolution of forces in the vertical direction
at x = : the second equation is the resolution of forces in the horizontal direction.
In addition the first integral, equation 15.18, represents the fact that the horizontal
component of the tension in the cable is constant. Since F y Fy is c1 or c2 we see
that the second of these conditions gives c1 = c2 = c. Using the actual expression for
F the first condition becomes
( )
(y2 2 )y2 (y1 1 )y1
M = p p = c (y2 y1 ) . (15.20)
1 + y2 2 1 + y1 2

Now we have sufficient conditions to solve the problem, as may be seen by substituting
the solutions 15.19 into these equations.
592 CHAPTER 15. SOLUTIONS TO EXERCISES

First we have the length constraints


    
d1 d1
Z q
L1 = 1 + y1 2 = c sinh
dx + sinh (15.21)
0 c c
Z a q     
a d2 d2
L2 = dx 1 + y2 2 = c sinh sinh . (15.22)
c c

The boundary conditions give


   
d1 a d2
B = 1 + c cosh and A = 2 + c cosh . (15.23)
c c

Equation 15.20 gives


    
d2 d1
M = c sinh sinh . (15.24)
c c

Finally the solution is continuous at x = ,


   
d1 d2
1 + c cosh = 2 + c cosh . (15.25)
c c

Thus we have six equations for the six constants (1 , 2 , , c, d1 , d2 ).

Solution for Exercise 12.16


df
(a) Since dt = f (s)s equation 12.42 becomes

d2 x d2 y
s 2
+ sy (s) = 0 and s 2 sx (s) = 0.
ds ds
On putting t = s, so s = 1 these integrate to

x + y = and y x = .

Differentiate the second with respect to s and use the first to substitute for x to obtain
2 y + y = which has the general solution y = + a cos(s/ + ), where a and are
constants. From this we obtain x = y = a sin(s/ + ).
(b) The curve is closed and has length L, that is x(0) = x(L) and y(0) = y(L), so
that L/ = 2. Further (x + )2 + (y a)2 = a2 , so a is the radius of the circle of
circumference L, that is 2a = L.

Solution for Exercise 12.17


This is the dual of the problem dealt with in the text, so the stationary curve is a circle.
It is not necessary to do any calculations to prove this but here we provide the details.
The auxiliary functional is
Z 2 p

L[x, y] = dt x2 + y 2 (xy xy .
0 2
15.12. SOLUTIONS FOR CHAPTER 12 593

where is the lagrange multiplier The Euler-Lagrange equations are


! !
d x d y
p + y = 0 and p x = 0.
dt x2 + y 2 dt x2 + y 2

Integrating these and using s, the arc length, for the independent variables gives as in
exercise 12.16
dx dy
= y and = + x,
ds ds
with solutions

x = a sin(s + ) and y = + a cos(s + )

where is a constant.

Solution for Exercise 12.18


(a) If 2(x) is the Lagrange multiplier the auxiliary functional is
Z b
dx y 2 + z 2 y 2 2(z y )

S[y, z] =
a

so the Euler-Lagrange equations for y and z, respectively, are

d d
(y + ) + y = 0 and (z ) + = 0,
dx dx
and the natural boundary condition is z (b) = 0. These equations simplify to

y + y + = 0 and = z ,

so eliminating z and gives

d4 y d2 y
y = 0, y(a) = A1 , z(a) = A2 , y(b) = B1 , z (b) = y (b) = 0.
dx4 dx2

(b) The Euler-Lagrange equation for the functional J[y] is given using the general result
given in section 10.2.1, but see also exercise 4.33 (page 143),

d2
   
F d F F
+ = 0,
dx2 y dx y y

with F = y 2 + y 2 y 2 this gives y (4) y (2) y = 0, with y(a) = A1 , y (a) = A2 ,


y(b) = B1 . The natural boundary condition for y is given by Fy = 0, that is y (b) = 0.

Solution for Exercise 12.19


Since y = s, dy/ds = 1 and the equation of motion is

dv
v = v 2 + g, v(0) = 0.
ds
594 CHAPTER 15. SOLUTIONS TO EXERCISES

Integration gives
v iv
v
Z h
dv =s that is ln(g v 2 = 2s
0 g v 2 0

which simplifies to the quoted result.

Solution for Exercise 12.20


(a) The Euler-Lagrange equation for is
 p
d 1
v = + ( )R (v) x 2 + y 2 .
d v2
p
Since (1) = 0, (1) = x 2 + y 2 /v 3 > 0.

(b) If (1 ) = 0 then by the same arguments as used above (1 ) > 0.


Thus ( ) can only increase through a zero and, since is continuous there cannot be
adjacent zeros in the interval [0, 1]. Since (1) = 0 and (1) > 0 we must have ( ) < 0
for 0 < 1.
Since H(, v) = v 1 ( )R(v) and R > 0 it follows that H(, v) > 0.

Solution for Exercise 12.21


(a) When R = 0 equation 12.83 becomes

1 v 1 2 1 2
Z
y =A dv v that is g(A y) = v v0 .
g v0 2 2

Multiplying by the mass gives the energy equation: the left-hand side is the loss in
potential energy as the particle falls through a distance A y; the right-hand side is
the gain in kinetic energy.

(b) If R = 0

2 g2
 
1
f (v)2 = + 2 2
g 2 2

v2 v2
p
2 + 2
 
2 2 2
 1 2 v
= g + hence =
v2 f (v) g 1 2 v 2

so that equation 12.81 for x(v) becomes

v v2
Z
x(v) = dv p .
g v0 1 (v)2

(c) Putting av = sin and v0 = 0 gives, with g + > 0,



1 1
Z
x(v) = 2 d sin2 = (2 sin 2) .
g 0 42 g
15.12. SOLUTIONS FOR CHAPTER 12 595

The energy equation, found in part (a), gives

1 1
y =A 2
sin2 = A 2 (1 cos 2) .
2 g 4 g

Putting c2 = 1/(22 g) gives the required result.

(d) Equation 12.76 becomes, with R = 0,



f (v) 1 2 v 2
g = p =
g 2 + 2 v
cos
= hence g = .
sin tan

At the terminus = 0, so = / tan b . Since tan > 0 for (0, /2) and tan < 0
for (/2, ) the result follows.
If b = /2 the cycloid is tangent to the x-axis at the terminus. If b > /2 it crosses
the x-axis and reaches a point lower than the end point. Thus type A motion has
b < /2 and type B motion has b > /2.

Solution for Exercise 12.22


In this example we are interested in the limit where the gravitational force is negligible
by comparison to the resistive force, so R > g. The quadratic equation 12.74 for is
therefore most conveniently written in the form,
   
2 2 2
 R 2 2 1
R g 2 g + 2 = 0.
v v
p
At the terminus (1) = 0 and 1/Vt = 2 + 2 and we assume that v(t) > Vt , so
v 2 Vt2 < 0 throughout the motion and the third term of this quadratic is negative.
The solution is
  s 2  
R R 1
R2 g 2 =

g g + (R2 g 2 ) 2 + 2 2
g g v

and using the condition v = Vt when = 0 we see that the lower sign gives the required
solution. Hence  
R f (v)
R2 g 2

g = p
g 2 + 2
where f (v) is defined in the question. Then, as in the text,
 2
2 2 2 1
H = + (g ) = R = 2 + (g )2
v

and  
dH d d d
H = g(g ) and H Hv R = ( g)
dv dv dv dv
596 CHAPTER 15. SOLUTIONS TO EXERCISES

so that    
d 1 d f (v)
HHv = R R g( g) = p
dv v dv 2 + 2
As in the text, equation 12.77
dx v d dy ( g)v d
= and =
dv HHv dv d HHv dv
and hence
dx p v dy p v
= 2 + 2 and = ( g) 2 + 2 .
dv f (v) dv f (v)

If g = 0, f = (2 + 2 )R and since v decreases along the path


Z v0 Z v0
p
2 2
v p
2 2
v
x(v) = + dv and y = A + dv = A x.
v R(v) v R(v)

With / = A/b this gives the straight line y = A(1x/b) through the terminal points.

Solution for Exercise 12.23


If tan is the gradient of the wire, < 0 and
A b
sin = , cos = and = 0
A + b2
2 A + b2
2

so the equation of motion 12.100 becomes


dv g(A b)
= .
dt A2 + b2
If A > b, v > 0 and the bead reaches the end at x = b. If A < b the bead decellerates
and it reaches the end only of the initial speed is sufficiently large: in this case the
equation of motion is valid only until v(t) = 0.
Integration gives

ds g(A b)t g(A b)t2


= v(t) = v0 + and s = v0 t + ,
dt A2 + b2 2 A2 + b2
where
s is the distance travelled along the wire. If A > b the end is reached when
s = A2 + b2 , that is at the positive root of
g(A b) 2 p
t + v0 t A2 + b2 = 0
2 A2 + b2
that is
p
v02 + 2g(A b) v0 p 2 2 A2 + b2
t= A + b2 = p .
g(A b) v0 + v02 + 2g(A b)

If A < b the above expression for s(t) is valid only t < t0 where v(t0 )= 0: for t t0
the bead is stationary. The equation for the time to reach the end, s = A2 + b2 is the
15.12. SOLUTIONS FOR CHAPTER 12 597

same but now both roots are positive and only one satisfies t < t0 and this gives the
above expression for t. If v02 < 2g(b A) this time is complex and the bead does not
reach the point x = b.

Solution for Exercise 12.24


(a) There are two ways of doing this. The easiest is by using elementary geometry. The
harder method is to note that
dy dy . dx sin
tan = = = hence tan tan = 1,
dx d d cos
so that tan( ) = . This equation has many solutions, but = /2 when = 0,
so the appropriate solution is = /2.
(b) Multiply equation 12.100 by dt/d to obtain
 
dv 2 d dy dx
v + v +g + = 0,
d d d d
and hence
dv
v + v 2 = gR(cos sin ).
d
(c) Observe that the equation of motion is homogeneous of degree two in v, which
suggests using the variable w defined by v 2 = gRw2 : the equation for w is
dw
w + w2 = (cos sin ), w(0) = 0.
d
The solution of this equation, w(, ) depends only upon and , so the condition
w(/2, ) = 0 gives an equation involving only.
(d) Write the equation for v in the form
 
2 d 1 2 2
e v e = gR(cos sin )
d 2
and integrate to give

1 2 2
Z
v e = gR d e2 (cos sin ).
2 0

But
1
Z
dx eax+ibx = eax+ibx
a + ib
eax
= (a cos bx + b sin bx + i(a sin bx b cos bx))
a + b2
2

so that
e2
Z
d e2 (cos sin ) = 3 cos + (1 22 ) sin )

1 + 42
598 CHAPTER 15. SOLUTIONS TO EXERCISES

and hence
1 2 gR h 2 2
i
v = 3 cos + (1 2 ) sin 3e
2 1 + 42
so that v(/2) = 0 if 22 + 3e = 1.
The speed is zero when and satisfy

g(, ) = 3 cos + (1 22 ) sin 3e2 = 0.

This equation defines a function (), the angle at which the bead stops. If > 1 ,
where 1 is the solution of this equation when = /2, the implicit function theorem
gives the rate of change of (1 ), d/d = g /g with the derivatives evaluated at
= /2 and = 1 . Thus

4 + 3(1 )e
(1 ) = = 1.36
3(1 2e )

where we have used the result 1 = 0.603. As expected () decreases as increases


past 1 .

Solution for Exercise 12.25


At the start = /2 and h(/2) = 1. At the terminus = 1 and since C = tan 1 ,

h(1 ) = 1 + 2 sin 1 cos 1 2 tan 1 cos2 1 = 1.

The derivative is h () = 2(cos 2C sin 2) and h () = 0 when cos 2 = tan 1 sin 2,


that is tan 1 tan 2 = 1. But
tan 2 tan 1
tan(2 1 ) =
1 + tan 1 tan 2

so the stationary points are at 2 1 = /2, 3/2, . But /2 1 < /2


and the only physically significant solution is 2 1 = /2.

Solution for Exercise 12.26


If = 0, h = 1 and

1 1
Z Z
x() = d (1 + cos 2) and y() = A + d sin 2.
2gB 2 /2 2gB 2 /2

Putting = /2 + gives

1 1
Z
x() = d (1 cos 2) = (2 sin 2)
2gB 2 0 4gB 2

1 1
Z
y() = A d sin 2 = A (1 cos 2).
2gB 2 0 4gB 2

At the terminus x = b, y = 0 and put = 1 so

4gB 2 b = 21 sin 21 and 4gB 2 A = 1 cos 21

and division gives the required equation for 1 .


15.12. SOLUTIONS FOR CHAPTER 12 599

Solution for Exercise 12.27


If is the Lagrange multiplier the functional is
Z
dx y 2 y 2 ,

S[y] = y(0) = y() = 0,
0

and the associated Euler-Lagrange equation is y + y = 0. If 0 there are no


solutions that satisfy the boundary conditions. Thus we set = 2 , to give the solution
y = A sin x that fits the boundary condition at x = 0. The condition at x = then
gives = 1, 2, 3, , so there are infinitely many solutions. The constraint gives
Z r
2 2 1 2 2
1=A dx sin nx = A giving A = ,
0 2

and the Lagrange multiplier = n2 .

Solution for Exercise 12.28


(a) If is the Lagrange multiplier the functional is
Z b
dx py 2 (q + w)y 2 ,

S[y] = y(a) = y(b) = 0,
a

with associated Euler-Lagrange equation


 
d dy
p + (q + w)y = 0, y(a) = y(b) = 0.
dx dx
Z b
(b) If the constraint is dx w(x)f (y) = 1 the functional becomes
a
Z b
dx py 2 qy 2 wf (y) ,

S[y] = y(a) = y(b) = 0,
a

with associated Euler-Lagrange equation


 
d dy 1
p + qy + wf (y) = 0, y(a) = y(b) = 0.
dx dx 2

If f (y) is linear in y, f = y, this equation becomes


 
d dy 1
p qy = w, y(a) = y(b) = 0,
dx dx 2

which is a linear inhomogeneuos equation. Otherwise f (y) is not linear and the Euler-
Lagrange equation is a nonlinear equation.

Solution for Exercise 12.29


If is the Lagrange multiplier
Z 1
dx y 2 y

S[y] =
0

and the Euler-Lagrange equation gives y = /2. The constraint then gives = 2a, so
y = a.
600 CHAPTER 15. SOLUTIONS TO EXERCISES

Solution for Exercise 12.30


If 1 and 2 are the Lagrange multipliers then
Z
dx y ln y + 1 y + 2 x2 y ,

P [y] =

so the Euler-Lagrange equation is simply

1 + ln y + 1 + 2 x2 = 0 y = exp 1 1 2 x2 .

that is

The first constraint gives


r

Z
2
1 = e11 dx e2 x = e11
2

and then the second constraint gives


r Z
2 2 2 1
= dx x2 e2 x =
22

x2
 
1
Hence y(x) = exp 2 .
2 2

Solution for Exercise 12.31


The auxiliary functional is
Z
dx y 2 y sin x ,

S[y] = y(0) = y() = 0
0

where is the Lagrange multiplier. The Euler-Lagrange equation 2y + sin x = 0,


with the general solution

y(x) = sin x + Ax + B.
2
The boundary condition at Zx = 0 gives B = 0 and that at x = gives A = 0. The
2a
constraint then gives a = dx sin2 x = hence y(x) = sin x.
2 0 4

Solution for Exercise 12.32


On the ellipse the relation between , see figure 12.8 and is

dy a cos a
tan = = =
dx b sin b tan

hence
1 d a d ab
= that is = 2 .
cos2 d b sin2 d a cos + b2 sin2
2

Thus the equation of motion is

dv abv 2
v + 2 = g(a cos b sin ).
d a cos2 + b2 sin2
15.12. SOLUTIONS FOR CHAPTER 12 601

If z = v 2 /2 this gives

dz 2abz
+ 2 = g(a cos b sin ).
d a cos + b2 sin2
2

Now define a function f () by


 
1 b
Z
ln f = 2ab dw = 2 tan1 tan ,
0 a2 cos2 w + b2 sin2 w a

so the equation can be written in the form

d(zf )
= g(a cos b sin )f (), z(0) = 0,
d
and integration gives

g
Z
z() = dw (a cos w b sin w)f (w)
f () 0

which is the required result. If z(/2) = 0 the equation for = b/a is


Z /2
dw (cos w sin w)f (w) = 0.
0
602 CHAPTER 15. SOLUTIONS TO EXERCISES

15.13 Solutions for chapter 13


Solution for Exercise 13.1
(a) With the Lagrange multiplier, the auxiliary functional is
Z b
dx py 2 (q + w)y 2

S[y] =
a

with the Euler-Lagrange equation (py ) + (q + w)y = 0.


(b) With (x) = 1/p(x) this functional is
2 !
b 
dx dy d
Z
2
S[y] = d p (q + w)y
0 d d dx
Z b  2 !
dy 2
= d p(q + w)y ,
0 d
x
1
Z
where (x) = dt and b = (b). The associated Euler-Lagrange equation is
a p(t)

y () + p(q + w)y = 0.

(c) With y = uv the functional S[u], for u, is


Z b    
1
S[u] = dx p u 2 v 2 + (u2 ) (v 2 ) + u2 v 2 qu2 v 2
a 2
but
b b
d
Z Z
b
dx p(u2 ) (v 2 ) = pu2 (v 2 ) a dx u2 (v 2 ) p
 
a a dx
so, since u(a) = u(b) = 0, the functional becomes
Z b    
2 2 2 2 1d 2
 2
S[u] = dx pv u qv pv + p(v ) u .
a 2 dx

Now set pv 2 = 1 to give


b
p 2 p
   
q
Z
S[u] = dx u 2 + 2 u2 .
a p 4p 2p
The constraint then becomes
b
w 2
Z
C[u] = dx u .
a p
so that the Euler-Lagrange equation is
d2 u p 2 p
 
q
+ + u = 0, u = y p,
dx2 p 4p2 2p
which is the same as equation 2.32, if in that equation, q is replaced by q + w.
15.13. SOLUTIONS FOR CHAPTER 13 603

Solution for Exercise 13.2


(a) In terms of the functional is
Z b  2 !
d dy 2
S[y] =
p (x) (q + w)y
a (x) d
Z b  
1
= d p (x)y ()2 (q + w)y 2 ,
a (x)
where a = (a) and b = (b). Now put y = A()v() to give
Z b    2 
2 2 1 2 2 A 2
S[v] =
d p (x) A v + (A ) (v )
(q + w) p (x)A v2 .
a 2 (x)
But, integration by parts gives
Z b b
d
Z
2 2

2 2 b

d v 2 p (x)(A2 )
 
d p (x)(A ) (v ) = p (x)(A ) v a
a a d
so that
ib
1h
S[v] = p (x)(A2 ) v 2
2 a
Z b   2  
2 2 A 2 1d  2
+ d A p (x)v (q + w) p A + p (x)(A ) v2 .
a (x) 2 d
Now define (x) with the equation A2 p (x) = 1 to put S[v] in the simpler form
b Z b
1 (A2 ) v 2

d v 2 F ()v 2

S[v] = 2
+
2 A a a

where
A 2 (A2 )
 
41d
F () = (q + w)A p 2 + .
A 2 d A2
But
A 2 (A2 ) A 2 d A A 2A 2 d2
   
1d 1
2
+ = + = = A ,
A 2 d A2 A2 d A A A2 d 2 A
and hence
d2
 
1
F () = (q + w)pA4 A .
d 2 A
(b) The coefficient of is made unity by defining wpA4 = 1 and then
d2
 
q 1
F () = + A 2 , where A = (wp)1/4
w d A
and the Euler-Lagrange equation is
s
x
d2 v v() w(t)
Z
+ F ()v = 0, y= , (x) = dt .
d 2 (wp)1/4 a p(t)
604 CHAPTER 15. SOLUTIONS TO EXERCISES

Solution for Exercise 13.3


Substituting = XY into the equation and dividing by gives
1 d2 X 1 d2 Y
+ + k 2 = 0.
X dx2 Y dy 2
Thus defining the two constants 1 and 2 by the equations
1 d2 X 1 d2 Y
= 12 and = 22
X dx2 Y dy 2
gives the two quoted equations if 12 + 22 = k 2 .
Since = 0 on the boundary we have X(0)Y (y) = X(a)Y (y) = 0 which, for
nontrivial Y (y), gives X(0) = X(a) = 0. Similarly Y (0) = Y (b) = 0.

Solution for Exercise 13.4


We need to express xx and yy in terms of the differentials of r and , using the chain
rule which gives
r r
= + and = + .
x r x x y r y y
But, since r2 = x2 + y 2 we have r/x = x/r = cos and r/y = y/r = sin . Also
by differentiating x = r cos and y = r sin with respect to x we obtain
r
1 = cos r sin
x x = r = sin
r x
0 = sin + r cos
x x
and, by differentiating with respect to y
r
0 = cos r sin
y y
= r = cos .
r y
1 = sin + r cos
y y
Hence
   
1 1
= cos sin and = sin + cos .
x r r y r r
Applying the first formula twice gives
2
  
1 1
= cos sin cos sin
x2 r r r r
2
     
2 1 1 1
= cos 2 sin cos sin cos + 2 sin sin
r r r r r r
and similarly
2 2
     
2 1 1 1
= sin cos cos + cos sin + 2 cos cos .
y 2 r2 r r r r r
15.13. SOLUTIONS FOR CHAPTER 13 605

Adding these two expressions gives


2 1 1 2
2 = + + .
r2 r r r2 2
If = R(r)() the equation 2 + k 2 = 0 becomes, on division by and multipli-
cation by r2 ,
r2 2 R r R 2 2 1 2
+ + k r + = 0.
R r2 R r 2
Putting = 2 , where is a positive constant, gives
d2
+ 2 = 0,
d2
d2 R dR
r2 2 + r + k 2 r2 2 R =

0.
dr dr
The constant is chosen to ensure that () is 2-periodic.
Since (rR ) = rR + R we can write the equation for R in the self-adjoint form
2
   
d dR 2
r + k r R = 0.
dr dr r

Solution for Exercise 13.5


(a) If < 0, put = 2 ( > 0) to give y 2 y = 0 with the general solution
y = A cosh x + B sinh x. The boundary condition at x = 0 gives A = 0 and at x = ,
B sinh = 0, which can be satisfied only if B = 0 (since is real).
If = 0 the general solution is y = A + Bx, which satisfies the boundary conditions
only if A = B = 0.
(b) If > 0, put = 2 ( > 0) to give y + 2 y = 0 with the general solution
y = A cos x + B sin x. The boundary condition at x = 0 gives A = 0 and at x = ,
B sin = 0, which is satisfied if = n, n = 1, 2, . Hence the eigenvalues and
eigenfunctions are n = n2 , yn (x) = B sin nx, n = 1, 2, .

Solution for Exercise 13.6


(a) We have
     
d dy X d dk X
p + qy = yk p + qk = k yk w(x)k .
dx dx dx dx
k=1 k=1

(b) If y(x) is a solution of the inhomogeneous equation 13.27 this gives



X
F (x) = k yk w(x)k .
k=1

Now multiply this equation by p (x) , integrate and use the orthogonality relation 13.23
to obtain Z b
du p (u) F (u) = p yp hp ,
a
606 CHAPTER 15. SOLUTIONS TO EXERCISES

which gives a value for yp . Substituting this value for yk into the original sum for y(x)
gives a solution of the inhomogeneous equation in the form
Z b Z b
X 1
y(x) = du F (u)k (u) k (x) = du G(x, u)F (u)
k hk a a
k=1


X k (u) k (x)
where G(x, u) = .
hk k
k=1

Solution for Exercise 13.7


(a) If = 0 the solution is y = constant. Otherwise the general solution that fits the
boundary condition at x = 0 is

A cosh x, < 0,
y=
A cos x, > 0.

Only if > 0 can the boundary condition at x = be satisfied, so put = 2 ( > 0)


and then sin = 0, that is = n, n = 0, 1, 2, and = n2 . Note that n = 0 gives
the = 0 eigenvalue. Hence the eigenfunctions and eigenvalues are

yn = cos nx, n = n2 , n = 0, 1, 2, .

For the orthogonality condition the integral needed is


(
Z
1
Z   0, n 6= m,
dx cos nx cos mx = dx cos(n m)x + cos(n + m)x =
0 2 0
, n = m.
2

(b) The general solution that fits the boundary condition at x = 0 is



A sinh x, < 0,
y= Ax, = 0,
A sin x, > 0.

Only if > 0 can the boundary condition at x = be satisfied, so put = 2 ( > 0)


and then cos = 0, that is = n + 1/2, n = 0, 1, 2, and = (n + 1/2)2 . Hence
the eigenfunctions and eigenvalues are

yn = sin(n + 1/2)x, n = (n + 1/2)2 , n = 0, 1, 2, .

For the orthogonality condition the integral needed is


(
Z
1
Z   0, n 6= m,
dx sin(n+ 12 )x sin(m+ 12 )x = dx cos(n m)x cos(n + m + 1)x =
0 2 0 , n = m.
2

(c) The general solution that fits the boundary condition at x = 0 is



A sinh x, < 0,
y= Ax, = 0,
A sin x, > 0.

15.13. SOLUTIONS FOR CHAPTER 13 607

If = 0 the boundary condition at x = cannot be satisfied. If < 0, put = 2


( > 0) to give tanh = and this has one real (positive) solution (as can be seen
by sketching the graphs of either side of the equation). Hence there is one negative
eigenvalue.
If > 0, put = 2 ( > 0) and the boundary condition at x = gives tan = ,
which has an infinity of positive solutions, k , k = 1, 2, , with k < k < k + 12 and
k k + 21 as k , as can be seen by sketching the graphs of either side of the
equation.
Hence the eigenfunctions and eigenvalues are

y0 (x) = sinh 0 x, 0 = 02 , tanh 0 = 0 , 0 > 0,


yn (x) = sin n x, n = n2 , tan n = n , n > 0.

The orthogonality condition is more difficult to establish in this case. First consider I0n ,
Z Z
I0n = dx sinh 0 x sin n x = i dx sin i0 x sin n x
0
Z  0  
sin(n i0 )
= dx cos(n i0 )x =
0 n i0
 
1 
= ( n + i 0 ) sin n cos i 0 cos n sin i 0 ,
n2 + 02

where (z) is the imaginary part of z. Using the definitions of k we see that the term
in the outer brackets is real, and hence I0n = 0.
For n, m 6= 0 we have
Z
1
Z
Inm = dx sin n x sin m x = dx (cos(n m )x cos(n + m )x)
0 2 0
 
1 sin(n m ) sin(n + m )
=
2 n m n + m
m sin n cos m n cos n sin m
=
n2 m2

n m  
= cos n cos m cos n cos m .
n2 m
2

If n 6= m this is zero. The case n = m can be obtained from this using LHospitals
rule. Alternatively,
Z  
2 1 2
 1 1
Inn = dx sin n x = cos n =
0 2 2 1 + n2

so Inn /2 as n .
Finally
 
1  1 1
Z
I00 = dx sinh2 0 x = cosh2 0 = .
0 2 2 1 02
608 CHAPTER 15. SOLUTIONS TO EXERCISES

Solution for Exercise 13.8


In all cases we use the formula 13.26 (page 351) with f (x) = x and w(x) = 1.
(a) The Fourier components are a0 = /2 and

R
dx x cos nx n 0, n even,
2((1) 1)
an = R0 = = 4 n = 1, 2, ,
0
dx cos2 nx n2 , n odd,
n2

4 X cos(2k + 1)x
giving x = .
2 (2k + 1)2
k=0

(b) The Fourier components are


R
dx x sin(n + 12 )x 8 (1)n
an = R0 2 1
= , n = 0, 1, 2 ,
0
dx sin (n + 2 )x (2n + 1)2

2 X (1)k
sin k + 21 x.

giving x = 1 2
(k + 2 )
k=0

(c) The Fourier components for n = 0 is


R
dx x sinh 0 x 2( 1) cosh 0
a0 = R0 2 =
0 dx sinh 0 x 0 ( cosh2 0 )

since
Z
1 1
Z
dx sinh2 0 x = cosh2 0 ,

dx x sinh 0 x = ( 1) cosh 0 ,
0 0 0 2

where the definition sinh 0 = 0 cosh 0 has been used. For n 1, we use the
results
Z Z
1 1
dx sin2 n x = cos2 n

dx x sin n x = ( 1) cos n ,
0 n 0 2

to obtain
R
dx x sin n x 2( 1) cos n
an = R0 2 = n = 1, 2, 3 ,
0 dx sin n x n ( cos2 n )

giving

2( 1) cosh 0 X cos k sin k x
x= sinh 0 x 2( 1) .
0 ( cosh2 0 ) k=1
k ( cos2 k )
15.13. SOLUTIONS FOR CHAPTER 13 609

Solution for Exercise 13.9

(a) If < 0 the nontrivial solution cannot be made to satisfy the boundary conditions.
If = 0 the solution y = 1 satisfies the boundary condition.
If > 0, put = 2 , ( > 0) giving the general solution (note it is easier to use the
complex form here)
y = Aeix + Beix .

The boundary conditions give

A+B = Ae2ia + Be2ia


and therefore A = Ae2ia and B = Be2ia .
AB = Ae2ia Be2ia

If a = n, n = 0, 1, 2, , both equations are satisfied otherwise they are not. Hence


there are two linearly independent solution (except if n = 0), y = exp(inx/a) with
the eigenvalue n = 2 = (n/a)2 .
Alternatively we may use the linear combinations,
n  nx   nx o  n 2
yn = cos , sin , n = , n = 0, 1, 2, .
a a a

(b) Consider the integral


Z 2a Z 2 
dx u1 u2 = a dw A1 A2 cos2 nw + B1 B2 sin2 nw
0 0

+ (A1 B2 + A2 B1 ) sin nw cos nw
= a (A1 A2 + B1 B2 ) ,

which is zero only if A1 A2 +B1 B2 = 0, that is the vectors a = (A1 , A2 ) and b = (B1 , B2 )
are orthogonal.

Solution for Exercise 13.10


First assume < 0 and put = 2 , > 0, giving y 2 y = 0 with the general
solution y = A cosh x + B sinh x. The boundary condition at x = 0 gives A = 0 and
then the other boundary condition gives sinh = a. This equation has no positive
roots if a < , and one if a > , as may be seen by sketching the graphs of a and
sinh .
If = 0 the solution satisfying the boundary conditon at x = 0 is y = Ax and the
second boundary condition gives a = .
Hence all eigenvalues are positive. Put = 2 , > 0, giving y + 2 y = 0 with the
solution satisfying the condition at x = 0 being y = B sin x. The second boundary
condition gives sin = a.
In figure 15.17 the graphs of u = sin and u = a, for some representative values
of a are shown.
610 CHAPTER 15. SOLUTIONS TO EXERCISES

1 a=1/5
a=1/10
0.5

0
1 2 3 4 5 6 7 8 9 10 11

-0.5

-1
Figure 15.17 Graphs of the functions u = sin and u = a for a = 1/5
and 1/10.

For > 0 these curves intersect if < c 1/a, giving real zeros; for > c the zeros
are complex. There are about N 1/a zeros because there is one zero every time
passes through an integer. Hence there are a finite number of real zeros.
Consider the inner product of two distinct eigenfunctions, yi and yj with j > i.

1
Z Z
Iij = dx sin i x sin j x = dx (cos(j i )x cos(j + i )x)
0 2 0
 
1 sin(j i ) sin(j + i )
= sin k = ak , k = i, j
2 j i j + i
 
a j cos i i cos j j cos i + i cos j
=
2 j i j + i
aj i
= (cos i cos j ) .
j2 i2

It is obvious that cos j + cos i 6= 0, but this is easily proved. We note that

a(j i ) = 2 sin(j i ) 2 cos(j + i ) 2 > 0


a(j + i ) = 2 sin((j + i ) 2 cos((j i ) 2 > 0


and since cos j cos i = 2 sin(j i ) sin(j + i ) , it follows that Iji 6= 0.
2 2
Solution for Exercise 13.11
Z 
dx
(a) Comparing with the equation in exercise 2.31 (page 74) we see that p = exp =x
x
and that the self-adjoint form is

2
   
d dy
x + x y = 0.
dx dx x


(b) In this example p = x, q = x 2 /x so v = 1/ p = 1/ x and

2 2 41
  
1
I(x) = 1 + 4x x =1 .
4x2 x x2
15.13. SOLUTIONS FOR CHAPTER 13 611

13.2 we put p = x, q = x, = 2 and


(c) Comparing with the equations in exercisep
1/4
w = 1/x, so A = (pw) = 1 and (x) = w/p = 1/x, giving = ln x. Then the
transformed equation is

d2 v
+ e2 2 v = 0,

2
y(x()) = v().
d

The solution of this equation is J (e ).

Solution for Exercise 13.12


(a) (i) Put n = m in equation 13.29

X
eiz sin t = Jm (z)eimt
m=

now put t = s, so sin t = sin s to put this in the form



X
eiz sin s = Jm (z)eim eims .
m=

Compare the nth coefficient of this and the original series to obtain the first result.
P
(ii) Put z = x in equation 13.29 eix sin t = n= Jn (x)eint , and now set t = +s,
so sin t = sin s to obtain eix sin t = in int
P
n= Jn (x)e e . Compare the nth
coefficient of this and the original series to obtain the second result.

(iii) Put t = 0

X 
X 
X
1= Jn (z) = J0 (z) + Jn (z) + Jn (z) = J0 (z) + 2 J2n (z),
n= n=1 n=1

since the terms with odd n cancel.

(b) Putting z = 0 gives



1
Z
int 1, n = 0,
Jn (0) = dt e =
2 0, otherwise.

(c) Differentiate equation 13.31 (page 354) with respect to x,


Z
dJn 1
= dt i sin t exp i (nt x sin t)
dx 2
Z
1 1
dt eit eit ei(ntx sin t) = (Jn1 (x) Jn+1 (x)) ,

=
4 2

that is 2Jn (x) = Jn1 (x) Jn+1 (x).


612 CHAPTER 15. SOLUTIONS TO EXERCISES

(d) The sum is


Z
1  
Jn1 (x) + Jn+1 (x) = dt ei((n1)tx sin t) + ei((n+1)tx sin t)
2
Z
1 1
Z
dt ei(ntx sin t) eit + eit = dt cos t ei(ntx sin t)

=
2
But
d i(ntx sin t)
e = i(n x cos t)ei(ntx sin t)
dt
so
n i(ntx sin t) 1 d i(ntx sin t)
cos t ei(ntx sin t) = e e ,
x ix dt
and

n 1 d i(ntx sin t) 2n
Z Z
i(ntx sin t)
Jn1 (x) + Jn+1 (x) = dt e dt e = Jn (x).
x ix dt x

Solution for Exercise 13.13


(a) Put x = et so
d2 y
 
dy dy dt dy dy d dy
= giving x = , similarly x x = 2
dx dt dx dx dt dx dx dt
and the equation becomes
d2 y dy
+ y = 0.
dt2 dt

It we put y = ept the equation for p is p2 p + = 0 giving 2p = 1 1 4. Thus
the general solution is
1
y = et/2 Aeqt + Beqt ,

q= 1 4,
2
or, in terms of x
1 1 1
Axq + Bxq , q =

y = 1 4, < ,
x 2 4
1 1 1
Aei ln x + Bei ln x , =

= 4 1, > ,
x 2 4
and if = 1/4, the general solution is y = et/2 (A + Bt) giving
1 1
y = (A B ln x) , = .
x 4
1
These solutions are bounded as x 0 only if < 4 and then only if B = 0 and
q 12 > 0, that is < 0. Thus if c > 0 the solution is
1
y = cxq1/2 , q= 1 4, for all < 0.
2
If c = 0 there are no nontrivial solutions.
15.13. SOLUTIONS FOR CHAPTER 13 613

(b) With the boundary conditions y(a) = y(1) = 0 the system is regular and we have:

< 1/4: the boundary conditions give

Aaq + Baq = 0 and A + B = 0

which have no real solutions for A and B.


= 1/4: the boundary conditions give A = B ln a and A = 0, so there is no nontrivial
solution.
> 1/4: the boundary conditions give

Aei ln a = Bei ln a and A + B = 0

hence = n/ ln a giving the eigenfuctions and eigenvalues


 
1 ln x 1  n 2
yn (x) = sin n , n = + , n = 1, 2, .
x ln a 4 ln a

Solution for Exercise 13.14


If w(x) > 0, p(x) > 0 and both have continuous second derivatives, the function

d2
 
q 1
f () = A 2
w d A
is continuous and hence has a minimum value Qm . If < Qm then f () + < 0 for
all and the result proved in the text shows that any solution v() has at most one
zero, so cannot satisfy the boundary conditions. Hence there are no eigenvalues smaller
than Qm .

Solution for Exercise 13.15


(a) For x < 0 the equation is like equation 13.41 with Q(x) = x < 0. Hence all solutions
have at most one zero for x < 0.
For x 1 use the comparison theorem 13.2 with Q1 = x and Q2 = 1 Q1 . The
comparison equation has a solution sin x with infinitely many zeros in between which
there is at least one zero of the equation y +xy = 0. Let these zeros be rn , n = 1, 2, .
(b) If = ax the equation becomes

1 d2 y
+ ay = 0 that is v () + a3 v() = 0.
a2 d 2
Thus if y(x) is a solution of y + xy = 0, v() = y(a) is a solution of v + a3 v = 0.
(c) Suppose the solution of y + xy = 0 with the condition y(0) = 0 is v(x); then
v(x) = y(1/3 x) where y(x) is a solution of y + xy = 0.
If = rn3 then v(1) = y(rn ) = 0, so y(rn x) is an eigenfunction with eigenvalue n = rn3 ,
and there are infinitely many of these.
There are no negative eigenvalues because y(0) = 0 and there can be no other zeros.
614 CHAPTER 15. SOLUTIONS TO EXERCISES

Solution for Exercise 13.16


We have
du
       
d d dv
v(Lu) u (Lv) = v p + qu u p + qv
dx dx dx dx
dp du d2 u d2 v
   
dp dv
= v +p 2 u +p 2
dx dx dx dx dx dx
2 2
du
   
dp dv d u d v
= v u +p v 2 u
dx dx dx dx dx2

   
dp du dv d du dv
= v u +p v u
dx dx dx dx dx dx
du
  
d dv
= p v u .
dx dx dx

Solution for Exercise 13.17


The boundary term, B, of equation 13.46 is
   
B = p(b) v(b)u (b) u(b) v (b) p(a) v(a)u (a) u(a) v (a)
  
= p(b) p(a) v(b)u (b) u(b) v (b) ,

which is zero, for all u and v, only if p(a) = p(b).

Solution for Exercise 13.18


For Lu = du/dx we have

dv du
Z h i Z

(u, Lv) = dx u = u v dx v = (Lu, v),
dx dx

since u and v tend to zero as |x| .


For Lu = idu/dx we have, similarly

du

dv du
Z Z Z
(u, Lv) = i dx u = i dx v= dx i v = (Lu, v),
dx dx dx

so that L is self-adjoint, but L is not.

Solution for Exercise 13.19


For this operator equation 13.46, with p = 1, holds so, with u and v real and satisfying
the boundary conditions we have
   
(Lu, v) = (u, Lv) = v()u () u()v () v(0)u (0) u(0)v (0)
   
= B v() u() A u (0) v (0) .

The right hand side is zero for all u and v only if A = B = 0.


15.13. SOLUTIONS FOR CHAPTER 13 615

Solution for Exercise 13.20


In this case p = 1 and for real functions
   
(Lu, v) (u, Lv) = v(0)u (0) u(0)v (0) v()u () u()v ()
 
= a u (0)v () v (0)u () ,

which is zero for all u and v only if a = 0.

Solution for Exercise 13.21


(a) Equation 13.56 gives tan 0 = 0, so 0 = 0. Since y = r sin , y(x) = 0 when = n,
n = 1, 2, . For the nth eigenfunction y() = 0 and this is the nth zero, so is given
implicitly by
Z n Z /2
1 1 n
= d 2 2 = 2n d 2 2 = .
0 cos + sin 0 cos + sin
showing that the nth eigenvalue is given by = n2 .
As increases from 0 to n, it passes through , 2, , (n 1), at which points
y(x) = 0, so there are precisely n 1 zeros.
(b) If () = (, ) it is defined implicitly by equation 13.58,
Z
1
= d .
0 cos + sin2
2

Differentiating with respect to gives



1 d sin2
Z
0= 2 d ,
cos + sin d
2
0 (cos2 + sin2 )2
which gives the required result, and shows that increases with if > 0.
(c) If = n2 , (, ) = n, so as n , (, n2 ) = n . But (, ) is a
continuous, monotonic increasing function of , hence (, ) for all .

Solution for Exercise 13.22


Since y = r sin (x), with (a) = 0, the kth zero of y(x) occurs at (x) = k. But
dx 1
= 2 = F (x, )
d Q(x) sin + p(x)1 cos2
and we have
1 1
2 1 = G1 () F (x, ) G2 () = 2
2
Q2 sin + p1 cos Q1 sin + p1 2
2 cos

If x1 () and x2 () are the solutions of xi = Gi (), i = 1, 2, we have x1 () x()


x2 (). The kth zero of the comparison equations are at = k so we have
Z k Z k
1 1
d 2 1 xk a d 2 .
0 Q 2 sin + p 1 cos2
0 Q 1 sin + p1 2
2 cos
616 CHAPTER 15. SOLUTIONS TO EXERCISES

But if A > 0 and B > 0 we have


Z k Z /2
1 1 k
d 2 2
= 2k d 2 2
= ,
0 A sin + B cos 0 A sin + B cos AB
and hence
p1 xk a p2
r r
.
q2 + w2 k q1 + w1
For the nth eigenfunction the nth zero is at x = b, so
 2  2
ba p2 ba
= q1 + n w1 p2
n q1 + w1 n
and  2  2
ba p1 ba
= q2 + n w2 p1
n q2 + w2 n
and hence  2  2
p1 ba q2 p2 ba q1
n .
w2 n w2 w1 n w1

Solution for Exercise 13.23



If = the equation becomes
1 d
= 1 x2 that is = 1 x2 .
dx
Substituting the series into this gives
0 + 2 1 + 3 2 + = 1 x 0 + 1 + 2 2 +


1 x20 2x0 1 2 x 21 + 20 2 + O(3 ),



=
and this rearranges to
     
1 x20 + 0 + 2x0 1 + 2 1 + x 21 + 20 1 = O(3 ).

Equating the coefficients of k , k = 0, 1, , to zero gives


1 0 1 1 + x21 7
0 = , 1 = = 2, 2 = = ,
x 2x0 4x 20 32x7/2
which gives the series quoted.

Solution for Exercise 13.24


Since (x) = F (, x; ) = (q + w) sin2 + p1 cos2 , if 2 > 1 , F (, x; 2 )
F (, x; 1 ) and the comparison theorem shows that (x, 2 ) (x, 1 ). Setting x = b
gives the required result.
If q(x) q1 , w(x) w1 and p(x) p2 for x (a, b) then
F (, x; ) (q1 + w1 ) sin2 + p1 2
2 cos = G()

and if = G() the comparison theorem gives (x, 2 ) (x, 1 ).


15.13. SOLUTIONS FOR CHAPTER 13 617

Solution for Exercise 13.25


(a) Since Q1/4 v = R cos and Q1/4 v = R sin , we have
v
Q1/2 v 2 + Q1/2 v 2 = R2 and tan = .
Q1/2 v
Hence
1 d v v 2 v Q
= , but v = Qv
cos2 d Q1/2 v Q1/2 v 2 2Q3/2 v
v 2 + v2 Q v Q Q1/2 1 Q sin
= 2 1/2 = and hence
v Q 2vQ3/2 cos2 2 Q cos

d 1Q
= Q1/2 sin 2.
d 4Q
Also
dR 1 Q 2 1 Q 2
2R = 2vv Q1/2 + 1/2
v + 2v v Q1/2 v
d 2Q 2 Q3/2
1 Q 2
cos2 sin2

= R and hence
2Q
d 1 Q
ln R = cos 2.
d 4Q

(b) Write Q(, ) = Q0 () + where Q0 () = q/w A(1/A) , is independent of , so


d 1 Q0
= ( + Q0 )1/2 sin 2
d 4 + Q0
d Q0
ln R = cos 2.
d 4( + Q0 )
If max(Q0 ) we may expand in powers of 1 ,
1 Q0
   
d Q0 Q0
= 1+ + 1 + sin 2
d 2 4
1/2
= + O( ),
and
d Q
ln R = 0 cos 2 + O(2 ).
d 4

Hence an approximation accurate to the lowest order is () = and R = r, for
some constants and r. Hence
r  
v() = cos
( + Q0 ())1/4
and
since y(a) = y(b) = 0, we set = /2 to satisfy the condition at x = a ( = 0) and
(b) = n to satisfy the condition at x = b, to obtain the approximate eigenvalue
 2 Z b r  
n w r n
n = , (b) = dx , with eigenfunction vn () = sin .
(b) a p Q(, n )1/4 (b)
618 CHAPTER 15. SOLUTIONS TO EXERCISES

Solution for Exercise 13.26


If = f (u)g(v) the equation can be written in the form
   
f 1 2 g 1 2
+ (k) cosh 2u + (k) cos 2v = 0.
f 2 g 2
The terms in curly braces are functions of u and v only, so
f g
+ 2(k)2 cosh 2u = a and 2(k)2 cos 2v = a
f g
where a is a constant. Hence the quoted equations. Since the points with coordinates
(u, v) and (u, v +2) are physically identical, g(v) must be 2-periodic, g(v +2) = g(v)
for all v.

Solution for Exercise 13.27


Increasing u by 2 increases by 2, so we define u() = + P () where P () is an
odd 2-periodic function of . Thus we can write u() in the form

X
u() = + ak sin k,
k=1

where
1 1 d
Z Z
ak = d (u() ) sin k = du sin u sin k (u sin u)
du
1
Z
= du sin u(1 cos u) sin k (u sin u)

Z
1 d
= du sin u cos k (u sin u)
k du
h i Z 
1
= sin u cos k (u sin u) du cos u cos k (u sin u)
k
Z

= du cos u cos k (u sin u)
k
Z
h    i
= du cos (k + 1)u k sin u + cos (k 1)u k sin u
2k
2
= (Jk+1 (k) + Jk1 (k)) = Jk (k).
k k

Solution for Exercise 13.28


We have, by differentiating under the integral sign,
Z Z
1 1
Jn (x) = dt (i sin t)ei(ntx sin t) and Jn (x) = dt ( sin2 t)ei(ntx sin t) ,
2 2
so that Bessels equation becomes
Z
n2
  
1 i
dt sin2 t sin t + 1 2 ei(ntx sin t) .
2 x x
15.13. SOLUTIONS FOR CHAPTER 13 619

If the integrand of this integral can be expressed as a differential of a periodic function,


the integral is zero and we have proved the required result. Consider the integral

1 d 
Z 
dt g(t)ei(ntx sin t) .
2 dt

By expanding this and comparing with the previous integrand we obtain the differential
equation,
n2
 
2 i
g + i(n x cos t)g = sin t sin t + 1 2 .
x x
Consider a solution g = A + B cos t; by substituting this in the left hand side we see
that if B = i/x and A = in/x2 a solution is obtained. This solution is periodic, hence
the result.

Solution for Exercise 13.29


If = 0 the general solution is y = A+Bx: the boundary condition at x = 0 gives A = 0
and the boundary condition at x = gives B = B. Hence there is no nontrivial
solution if = 0 (except possibly if = , a case we return to later).

<0
If < 0, put = 2 , ( > 0), the general solution is y = A cosh x + B sinh x: the
boundary condition at x = 0 gives A = 0 and the boundary condition at x = gives

tanh = , > 0.

If > there are no real solutions of this equation the gradient of the left and right
hand sides at = 0 are, respectively, and , so if > , > tanh for > 0.
If < , the same reasoning shows that there is one real positive solution which we
denote by 0 .

>0
If > 0, put = 2 , ( > 0), the general solution is y = A cos x + B sin x: the
boundary condition at x = 0 gives A = 0 and the boundary condition at x = gives

tan = , > 0.

If > , the first positive solution, 0 is in (0, /2) and the solution k , is in the
interval (k, (k + 1/2)), k = 0, 1, .
If < , the first positive solution, 1 is in (/2, 3/2) with kk < (k + 1/2),
k = 1, 2, .
Thus we have the following,

if > the eigenvalues are k = k2 with k < < (k + 1/2), k = 0, 1, :

if = the function y = Bx is a solution for = 0 and all B:

if < then 0 = 02 and k = k2 with k < < (k + 1/2), k = 1, 2, .


620 CHAPTER 15. SOLUTIONS TO EXERCISES

Solution for Exercise 13.30


In this example p /p = tan x, so that p(x) = 1/ cos x and the self-adjoinf form is
 
d 1 dy
cos x = 0.
dx cos x dx

Solution for Exercise 13.31


Since x/(1 + x) 1/2 for x 1, put Q2 = 1/2 and Q1 = x/(1 + x) in the comparison

theorem 13.2, to see that the given equation has at least one zero in (n 2, (n+ 1) 2)
for every n = 1, 2, .

Solution for Exercise 13.32


If < 0 there are no periodic solutions. If = 0 the general solution is y = A + Bx,
which is periodic if B = 0. If > 0, put = 2 , > 0, to hive the solutions cos x
and sin x, which are 2-periodic if n = 1, 2, .
Hence 2-periodic solutions are
yn = {cos nx, sin nx}, n = 0, 1, , with n = n2 .
Alternatively the functions zn = einx , n = 0, 1, , satisfy the equation and are
2-periodic, with n = n2 . These are linear combinations of the first set of functions.

Solution for Exercise 13.33


(a) If x = et then t (0, ) if x (1, ), and
d2 y
 
dy dy d dy
x = and x x = ,
dx dt dx dx dt2
pt 2
and the equation
becomes y y + By = 0. Putting y = e gives p p + B = 0 so
that 2p = 1 1 4B. If 4B > 1 this gives the general solution
h i
y = x A cos( ln x) + B sin( ln x) , = 4B 1,

which has infinitely many zeros for x > 1. If 4B < 1 the general solutions is

y = x Axq + Bxq , q = 1 4B,


which has at most one zero.


(b) If q(x) > 1/4 use the equation defined in part (a) as a comparison equation.

Solution for Exercise 13.34


(a) Since (xy ) = xy + y the first result follows directly. Since p(x) = x, the system
is regular provided the interval does not include the origin.
(b) With p = x, q = /x the normal form, exercise 2.31 (page 74) is
d2 u
 
1 1 + 4
I(x) = 2 (1 + 4) giving + u = 0 with y = u/ x.
4x dx2 x2
Comparing with equation 13.34 (page 357) we see that q and w are continuous only if
x 6= 0, so this system is regular provided the interval does not contain the origin.
15.13. SOLUTIONS FOR CHAPTER 13 621

(c) Put x = et , so 0 < t < and

du du d2 u d2 u du
x = and x2 2
= 2 +
dx dt dx dt dt
and the equation for u becomes u (t) + u (t) + ( + 1/4)u = 0. Putting u = ept gives
p2 + p + ( + 1/4) = 0 and hence the general solution is
 
u = et/2 Ae t + Be t ,

= 2 , > 0,

(Ax + Bx )

y = (A B ln x) , = 0,

i ln x i ln x
= 2 , > 0.

Ae + Be

(i) < 0: the solution is bound at the origin only if B = 0, so y = Ax giving y(0) = 0
and y(1) = A. Hence there are no nontrivial solutions.
(ii) = 0: In the case the bound solutions are y = A: if c 6= 0, the solution is y = c,
with eigenvalue = 0.
(iii) > 0: the solution is not defined at the origin for any A or B, except A = B = 0.

(d) (i) < 0: the boundary conditions give

Aa + Ba = 0
= A = B = 0.
Ab + Bb = 0

(ii) = 0: the boundary conditions give

A B ln a = 0, A B ln b = 0 = A = B = 0.

(iii) > 0: the boundary conditions give

Aei ln a + Aei ln a = 0
= e2i ln(b/a) = 1,
Aei ln b + Aei ln b = 0
n
hence n = n2 , n = , and yn = c sin (n ln(x/a)), for some constant c.
ln(b/a)

Solution for Exercise 13.35


In this example q(x) = xa , so for x [0, 1], q1 = 0 and q2 = 1; since p = w = 1, the
inequality of exercise 13.22 is (n)2 1 n (n)2 . This shows that for large n, n
is relatively close to (n)2 .
622 CHAPTER 15. SOLUTIONS TO EXERCISES

15.14 Solutions for chapter 14


Solution for Exercise 14.1
Z 1
dx y 2 xy 2 , y(0) = 1, y(1) = 0. The trial function

The functional is S[y] =
0
y = 1 ax (1 a)x2 satisfies the boundary conditions, so we need the integrals
Z 1  2 Z 1  
S1 = dx a + 2(1 a)x = dx a2 + 4a(1 a)x + 4(1 a)2 x2
0 0
4 2 1
= a + a2 ,
3 3 3
Z 1  
S2 = dx x 1 2ax + (a2 + 2a 2)x2 + 2a(1 a)x3 + (1 a)2 x4
0
1 1 1
= a + a2
6 10 60
so that
19 2 17 7 19 17
a a+
S(a) = and S (x) = a .
60 30 6 30 30
The stationary point is at a = 17/19 and hence the approximate solution is
17 2
z =1 x x2 .
19 19
In the interval [0, 1] the largest difference between this approximation and the numeri-
cally generated solution is 0.0012.

Solution for Exercise 14.2


(a) The Euler-Lagrange of the functional is y + y 3 = 0, y(0) = 0, and the natural
boundary condition at x = X is, equation 10.7 (page 260), y (X) = 0.
 x  a  x 
(b) The trial function y(x) = a sin , y (x) = cos , satisfies both bound-
2X 2X 2X
ary conditions. Substituting it into the functional gives
1  a 2 X  x  a4 Z X  x 
Z
S(a) = dx cos2 dx sin4 .
2 2X 0 2X 4 0 2X
But
X  x  1
X X
Z Z
2
dx cos = dz (1 + cos z) = , x = Xz,
0 2X 2 0 2
and
X  x  1  
3 1 1 3X
Z Z
dx sin4 =X dz cos z + cos 2z = ,
0 2X 0 8 2 8 8
so that
2 a2 3a4 X 2 a 3a3 X
S(a) =
and S (a) =
16X 32 8X 8


so S (a) = 0 when aX = / 3, giving the approximate solution
 x 
y = sin .
X 3 2X
15.14. SOLUTIONS FOR CHAPTER 14 623

This result suggests that the solution with amplitude A = /(X 3) has the period
1
T = 4X. The exact period of this solution is obtained from the first-integral y 2 +
2
1 4 1 4
y = A : the period is given by
4 4
Z T /4 Z A Z
dy 1
T = 4 dx = 4
= 4 2 dy p
0 0 y A y4
4
Z 1
4 2 1 2 3/2 2 3
= dz = . With A = , T = X 4.09X.
A 0 1 z5 A(3/4)2 X 3 (3/4)2

Thus this simple variational estimate provides a fairly good approximation to the period.

Solution for Exercise 14.3


Multiply equation 14.9 by y and integrate,
Z 1 Z 1    
d dy
dx y 2 = dx y p qy 2
0 0 dx dx
 1 Z 1  2 !
dy dy 2
= py + dx p qy = S[y],
dx 0 0 dx

since y(0) = y(1) = 0 the boundary term vanishes and C[y] = S[y]. Putting y = yn
gives the result.

Solution for Exercise 14.4


We have
S[zn ] = S[yn + u] = S[yn ] + S[yn , u] + O(2 ).
But the Gateaux differential S is
Z 1
S[yn , u] = dx ((pyn ) + qyn ) u
0
Z 1
= n dx yn u from the Euler-Lagrange equation.
0

But both yn and zn satisfy the constraint,


Z 1 Z 1
1 = C[yn + u] = C[yn ] + 2 dx yn u + O(2 ) = dx yn u = O(),
0 0

from which it follows that S[zn ] = n + O(2 ).

Solution for Exercise 14.5


(a) In this example p(x) = w(x) = 1 and q(x) = x2 , and a simple trial function is
z = a sin x having only one free variable, which is determined by the constraint,
Z 1
2 1
C[z] = a dx sin2 x = a2 = 1.
0 2
624 CHAPTER 15. SOLUTIONS TO EXERCISES

The functional is
Z 1 Z 1 Z 1
2 2 2 2 2 2
dx x2 sin2 x.

S(a) = dx pz qz = a dx cos x a
0 0 0
R1 R1
But 0 dx x2 sin2 x = 1
2 0 dx x2 (1 cos 2x) and
1 1
x2 1 1
Z  Z
2
dx x cos 2x = sin 2x dx x sin 2x
0 2 0 0
 Z 1 
1 h x i1 1 1
= cos 2x + dx cos 2x = .
2 0 2 0 2 2
1 1
Hence 1 S(a) = 2 3 + 2 2 9.587.
(b) For the trial function z = ax(1 x), the constraint gives
Z 1
a2
 
2 2 2 2 1 2 1
C[z] = a dx x (1 x) = a + = = 1.
0 3 4 5 30

The functional therefore has the value


Z 1 Z 1
2
S[z] = 30 dx (1 2x) 30 dx x4 (1 x)2
0 0
 
1 1 1 68
= 10 30 + = 9.714.
5 3 7 7

Hence 1 < 9.71. The first bound is smaller, so is the better approximation.

Solution for Exercise 14.6


With the one parameter trial function z = a1 sin(x/2) the constraint gives
1
1 1
Z
a21 dx sin2 x = a21 = 1.
0 2 2
The functional is
1
2
   2 
1 1 1 1
Z
S(a1 ) = a21 dx cos2 x x sin2 x = a21 2 .
0 4 2 2 8 4

Hence
2 1 2
1 S(a1 ) =
2 = 1.76476.
4 2
Note that to 10 significant figures the value of the first eigenvalue is 1.762682254 it
can be shown to be the first zero of Ai(u)Bi (1 u) Bi(u)Ai (1 u), where Ai
and Bi are Airy functions.
With the two parameter trial function z = a1 sin x/2 + a2 sin 3x/2 the constraint
gives
Z 1 Z 1
2 2 1 2 3 1 1
a1 dx sin x + a2 dx sin2 x = a21 + a22 = 1.
0 2 0 2 2 2
15.14. SOLUTIONS FOR CHAPTER 14 625

The functional is
Z 1  2 Z 1  2
x 3 3x x 3x
S(a) = dx a1 cos + a2 cos dx x a1 sin + a2 sin
0 2 2 2 2 0 2 2
2 2
    
1 1 1 1 2
a1 + 9a22 a21 + a22 2 a1 a2

= + +
8 4 2 4 9 2
 2   2 
1 1 9 1 1 2
= 2 a21 + 2 a22 + 2 a1 a2 ,
8 4 8 4 9
where we have used the integrals quoted in the question. Thus the equation is
2
1 2 2

4
2 2 2
a = a
2
2 9 1 2
2
2 4 2 9
and the eigevalues are given by the quadratic equation 2 23.4489 + 38.2261 = 0
and the smallest root is 1.7627.

Solution for Exercise 14.7


(a) Substituting the series into the solution gives
n
X n
X
ak k 2 2 + xak sin kx =

ak sin kx.
k=1 k=1

Now multiply by sin px and integrate to obtain


Xn Z 1
p2 2 ap + 2 ak dx x sin kx sin px = ap , p = 1, 2, , n.
k=1 0

These n linear equations for a can be written in the matrix form M a = a where Mij
is defined in the question.
(b) If n = 1 we have
Z 1
2
M11 = 2 dx x sin2 x
0
Z 1 Z 1
2
= dx x(1 cos 2x), but since dx x cos 2x = 0,
0 0

this gives 1 M11 = 2 1/2.


If n = 2 the other matrix elements are
Z 1 Z 1
M12 = 2 dx x sin x sin 2x = dx x (cos x cos 3x))
0 0

and since
1
1 (1)k 16
Z
dx x cos kx = , M12 =
0 (k)2 9 2
626 CHAPTER 15. SOLUTIONS TO EXERCISES

and
1
1
Z
M22 = 4 2 dx x (1 cos 4x) = 4 2 ,
0 2
giving the eigenvalue problem
1 16

2
2 9 2
1 a = a,

16
2
4
9 2 2
which is just equation 14.21.
(c) If p = 1, q = x and k = sin kx we have
Z 1 Z 1
(H 1 S)ij = 2 2 ij dx cos ix cos jx 2 dx x sin ix sin jx
0 0
Z 1
= 2 ijij 2 dx x sin ix sin jx = Mij .
0

Solution for Exercise 14.8


(a) If is the Lagrange multiplier, the Gateaux differential is
Z b  
S = 2p(a)y(a)h(a) + 2p(b)y(b)h(b) + 2 dx py h qyh wyh
a
   
= 2h(a)p(a) y(a) + y (a) + 2h(b)p(b) y(b) + y (b)

Z b  
2 dx (py ) + (q + w)y h.
a

Using the class of variations with h(a) = h(b) = 0, we see that the Euler-Lagrange
equation,  
d dy
p + (q + w)y = 0,
dx dx
must be satisfied by a stationary path. Further, since S = 0 for all admissible paths
the given boundary conditions must also be satisfied.
(b) Since wy = qy + (py ) we have
Z b Z b Z b
2
k dx wyk = dx qyk2 + dx yk (pyk )
a a a
h ib Z b  
= yk pyk dx pyk 2 qyk2 .
a a

Using the constraint condition and the boundary conditions to replace y (a) with y(a)
and y (b) with y(b), this becomes
Z b  
k = dx pyk 2 qyk2 + p(b)y(b)2 p(a)y(a)2 = S[yk ].
a
15.14. SOLUTIONS FOR CHAPTER 14 627

Solution for Exercise 14.9



(a) For this differential equation, p = 1, q = 0, w = x and (a, b) = (0, 1), so
Z 1 Z 1
2
S[y] = dx y and C[y] = dx x y 2 = 1.
0 0

(b) Substituting z = ax(1 x) into the constraint gives


1
16
Z
1 = a2 dx x5/2 (1 x)2 = a2
0 693
1
a2 231
Z
so that 1 S[z] = a2 dx (1 2x)2 = = .
0 3 16

Solution for Exercise 14.10


In this case p(x) = w(x) = 1 and q(x) = x2p . With the trial function z = a(1 x2 ) the
constraint gives
1 1
16 2
Z Z
C[z] = a2 dx (1 x2 )2 = 2a2 dx (1 2x2 + x4 ) = a .
1 0 15

The functional is
Z 1 Z 1
S[a] = dx (2ax)2 a2 dx x2p (1 x2 )2
1 1
Z 1 Z 1
= 8a2 dx x2 2a2 dx x2p (1 2x2 + x4 )
0  0

2 8 1 2 1
= a 2 + .
3 2p + 1 2p + 3 2p + 5
 
5 6
Hence 1 S(a) = 1 .
2 (2p + 1)(2p + 3)(2p + 5)

Solution for Exercise 14.11


(a) If = 0 the general solution is y = A+Bx; the boundary conditions give A = B = 0.
If < 0, put = 2 , ( > 0), so the general solution is y = A cosh x + B sinh x;
the boundary condition at x = 0 gives B = 0, and that at x = 1 gives A cosh = 0, so
A = 0.
If > 0, put = 2 , ( > 0), so the general solution is y = A cos x + B sin x; the
boundary condition at x = 0 gives B = 0, and that at x = 1 gives A cos = 0, so
= (n 1/2), n = 1, 2, , giving n = (n 1/2)2 2 .

(b) For this problem the functional and constraint are


Z 1   x  Z 1
S[y] = dx y 2 by 2 sin and C[y] = dx y 2 = 1.
0 2 0
628 CHAPTER 15. SOLUTIONS TO EXERCISES

Taking the lowest eigenfunction of the simpler problem, z = a cos(x/2), to be the trial
function the constraint gives
Z 1  x  a2
a2 dx cos2 = = 1,
0 2 2
and the functional becomes
Z 1   x 
S(a) = dx y 2 by 2 sin
0 2
1 2 2 1
Z  x  Z 1  x   x 
= a dx sin2 a2 b dx sin cos2
4 0 2 0 2 2
2 2 2b 2 2 4b
= a a and hence 1 .
8 3 4 3

(c) With the trial function z = a cos(n 1/2)x, the constraint gives a2 = 2 and the
functional becomes
Z 1 Z 1
S(a) = a2 2 (n 1/2)2 dx sin2 (n 1/2)x a2 b dx sin(x/2) cos2 (n 1/2)x.
0 0

But
1 1 
sin(x/2) cos2 (n 1/2)x = sin(x/2) + sin(2n 1/2)x sin(2n 3/2)x ,
2 4
so
1  
1 1 1 1
Z
dx sin(x/2) cos2 (n 1/2)x = +
0 4 2n 1/2 2n 3/2
1 1
=
(4n 1)(4n 3)
and hence
2
a2 b
  
1 2 2 1 1
S(a) = a n 1 , and since a2 = 2
2 2 (4n 1)(4n 3)
 
2b 1 1
n n2 2 1 2 , n=n .
16n 1 2

Solution for Exercise 14.12


(a) By comparison with the systems considered in exercise 13.1 (page 341) we see that
p = 1/x, q = 0 and w = x; since the system is defined on the interval (1, 2), where
p > 0 and w > 0, and has separable boundary conditions it is a regular Sturm-Liouville
system (section 13.4).
(b) The functional associated with this system is given in exercise 14.8 (page 387),
Z 2 Z 2
1 2 1 2
S[y] = y(2) + dx y with the constraint C[y] = dx xy 2 = 1,
2 1 x 1
15.14. SOLUTIONS FOR CHAPTER 14 629

and the general theory shows that


2
1 1
Z
k = S[yk ] = yk (2)2 + dx yk 2 ,
2 1 x

and for the lowest eigenvalue 1 , 1 S[y], where y is any admissible function.
Alternatively this Sturm-Liouville equation can be written in the form
  Z 2 Z 2  
d 1 dyk 2 d 1 dyk
k xyk = and hence k dx xyk = dx yk .
dx x dx 1 1 dx x dx
Z 2
Since, by definition, hk = dx xyk2 , putting hk = 1 gives
1

2    2 2  2
d 1 dyk 1 1 dyk
Z Z
k = dx yk = yk yk + dx .
1 dx x dx x 1 1 x dx

With k = 1 the boundary conditions y1 (1) = 0 and y1 (2) y1 (2) = 0 then give,
2  2
1 1 dy1
Z
1 = y1 (2)2 + dx .
2 1 x dx

The trial function z = (x 1)(Ax + B) satisfies the boundary condition at x = 1 and


at x = 2, z(2) z (2) = A = 0 if z = B(x 1). The constraint gives
Z 2 Z 2
B2 dx x(x 1)2 = B 2 dx x x3 2x2 + x

1 =
1 1
 2
1 4 2 3 1 2 7 2
= A2 x x + x = B .
4 3 2 1 12

The value of the functional is


2
1 dx 1
Z
S[z] = B 2 + B 2 = B 2 + B 2 ln 2.
2 1 x 2

Hence, since the functional S[y] has a minimum the Rayleigh-Ritz method shows that

6 12
1 S[z] = + ln 2 = 0.331.
7 7
This is an upper bound because S[y] is a positive quadratic functional having a minimum
value which is attained in the allowed space of functions.

The exact solution


In this example the equation can be solved by putting x = w. This gives

d2 y
+ y=0
dw2 4
630 CHAPTER 15. SOLUTIONS TO EXERCISES

and the solution fitting the boundary condition at x = 1 is


!
2
y(x) = A sin (x 1) .
2

3
 
The boundary condition at x = 2 gives the equation tan = 2 . The smallest
2
positive solution of this is = 0.317.

Anda mungkin juga menyukai